People, Development and Environment UGC NET Paper 1 MCQ

The total CO2 emissions from various sectors are 5 mmt. In the Pie Chart given below, the percentage contribution to CO2 emissions from various sectors is indicated.What is the absolute CO2 emission from domestic sector?

(A) 1.5 mmt
(B) 2.5 mmt
(C) 1.75 mmt
(D) 0.75 mmt

Answer: D

What is the absolute CO2 emission for combined thermal power and transport sectors?

(A) 3.25 mmt
(B) 1.5 mmt
(C) 2.5 mmt
(D) 4 mmt

Answer: A

Which of the following pollutants affects the respiratory tract in humans?

(A) Carbon monoxide
(B) Nitric oxide
(C) Sulphur di-oxide
(D) Aerosols

Answer: C

Which of the following pollutants is not emitted from the transport sector?

(A) Oxides of nitrogen
(B) Chlorofluorocarbons
(C) Carbon monoxide
(D) Poly aromatic hydrocarbons

Answer: B

Which of the following sources of energy has the maximum potential in India?

(A) Solar energy
(B) Wind energy
(C) Ocean thermal energy
(D) Tidal energy

Answer: A

Ecological footprint represents

(A) area of productive land and water to meet the resources requirement
(B) energy consumption
(C) CO2 emissions per person
(D) forest cover

Answer: A

The Ganga Action Plan was initiated during the year

(A) 1986
(B) 1988
(C) 1990
(D) 1992

Answer: A

Identify the correct sequence of energy sources in order of their share in the power sector in India:

(A) Thermal > nuclear > hydro > wind
(B) Thermal > hydro > nuclear > wind
(C) Hydro > nuclear > thermal > wind
(D) Nuclear > hydro > wind > thermal

Answer: B

Chromium as a contaminant in drinking water in excess of permissible levels, causes

(A) Skeletal damage
(B) Gastrointestinal problem
(C) Dermal and nervous problems
(D) Liver/Kidney problems

Answer: D

The main precursors of winter smog are

(A) N2O and hydrocarbons
(B) NOx and hydrocarbons
(C) SO2 and hydrocarbons
(D) SO2 and ozone

Answer: C

Flash floods are caused when

(A) the atmosphere is convectively unstable and there is considerable vertical wind shear
(B) the atmosphere is stable
(C) the atmosphere is convectively unstable with no vertical windshear
(D) winds are catabatic

Answer: A

Which of the following is a source of methane?

(A) Wetlands
(B) Foam Industry
(C) Thermal Power Plants
(D) Cement Industry

Answer: A

โ€˜Minamata disasterโ€™ in Japan was caused by pollution due to

(A) Lead
(B) Mercury
(C) Cadmium
(D) Zinc

Answer: B

Biomagnification means increase in the

(A) concentration of pollutants in living organisms
(B) number of species
(C) size of living organisms
(D) biomass

Answer: A

Nagoya Protocol is related to

(A) Climate change
(B) Ozone depletion
(C) Hazardous waste
(D) Biodiversity

Answer: D

The second most important source after fossil fuels contributing to Indiaโ€™s energy needs is

(A) Solar energy
(B) Nuclear energy
(C) Hydropower
(D) Wind energy

Answer: C

In case of earthquakes, an increase of magnitude 1 on Richter Scale implies

(A) a ten-fold increase in the amplitude of seismic waves.
(B) a ten-fold increase in the energy of the seismic waves.
(C) two-fold increase in the amplitude of seismic waves.
(D) two-fold increase in the energy of seismic waves.

Answer: A

Irritation in eyes is caused by the pollutant

(A) Sulphur di-oxide
(B) Ozone
(C) PAN
(D) Nitrous oxide

Answer: C

Which is the source of chlorofluorocarbons?

(A) Thermal power plants
(B) Automobiles
(C) Refrigeration and Air-conditioning
(D) Fertilizers

Answer: C

Which of the following is not a renewable natural resource?

(A) Clean air
(B) Fertile soil
(C) Fresh water
(D) Salt

Answer: D

Which of the following parameters is not used as a pollution indicator in water?

(A) Total dissolved solids
(B) Coliform count
(C) Dissolved oxygen
(D) Density

Answer: D

S and P waves are associated with

(A) floods
(B) wind energy
(C) earthquakes
(D) tidal energy

Answer: C

Match Lists I and II and select the correct answer from the codes given below :

List โ€“ I
(i) Ozone hole
(ii) Greenhouse effect
(iii) Natural hazards
(iv) Sustainable development
List โ€“ II
(a)Tsunami
(b) UV radiations
(c) Methane
(d) Eco-centrism

Codes : (i) (ii) (iii) (iv)
(A) (b) (c) (a) (d)
(B) (c) (b) (a) (d)
(C) (d) (c) (a) (b)
(D) (d) (b) (c) (a)

Answer: A

Which one of the following continents is at a greater risk of desertification ?

(A) Africa
(B) Asia
(C) South America
(D) North America

Answer: A

โ€œWomen are closer to nature than men.โ€ What kind of perspective is this ?

(A) Realist
(B) Essentialist
(C) Feminist
(D) Deep ecology

Answer: B

Which one of the following is not a matter a global concern in the removal of tropical forests ?

(A) Their ability to absorb the chemicals that contribute to depletion of ozone layer.
(B) Their role in maintaining the oxygen and carbon balance of the earth.
(C) Their ability to regulate surface and air temperatures, moisture content and reflectivity.
(D) Their contribution to the biological diversity of the planet.

Answer: A

The most comprehensive approach to address the problems of manenvironment interaction is one of the following :

(A) Natural Resource Conservation Approach
(B) Urban-industrial Growth Oriented Approach
(C) Rural-agricultural Growth Oriented Approach
(D) Watershed Development Approach

Answer: D

The major source of the pollutant gas, carbon mono-oxide (CO), in urban areas is

(A) Thermal power sector
(B) Transport sector
(C) Industrial sector
(D) Domestic sector

Answer: B

In a fuel cell driven vehicle, the energy is obtained from the combustion of

(A) Methane
(B) Hydrogen
(C) LPG
(D) CNG

Answer: B

By the year 2022, the Climate Change Action Plan of Government of India aims at installing

(A) 20,000 MW of wind power
(B) 25,000 MW of wind power
(C) 20,000 MW of solar power
(D) 10,000 MW of solar power

Answer: C

Which one of the following biosphere reserves has UNESCO recognition ?

(A) Manas
(B) Kanchenjunga
(C) Seshachalam Hills
(D) Greater Nicobar

Answer: D

Which activity contributes to water pollution more than any other throughout world ?

(A) Agriculture
(B) Hydroelectric power generation
(C) Industry
(D) Urbanisation

Answer: A

Match List โ€“ I with List โ€“ II and find the correct answer from the codes given below :

List โ€“ I (Biosphere Reserve)
a. Nilgiri
b. Manas
c. Similipal
d. Achankmar-Amarkantak
List โ€“ II (Area of Location)
i. Deccan Peninsula
ii. Chhattisgarh
iii. Eastern Himalaya
iv. Western Ghat

Codes : a b c d
(A) i ii iii iv
(B) ii iii iv i
(C) iii iv ii i
(D) iv iii i ii

Answer: D

The phrase โ€˜tragedy of commonsโ€™ is in the context of

(A) tragic event related to damage caused by release of poisonous gases.
(B) tragic conditions of poor people.
(C) degradation of renewable free access resources
(D) climate change

Answer: C

Kyoto Protocol is related to

(A) Ozone depletion
(B) Hazardous waste
(C) Climate change
(D) Nuclear energy

Answer: C

Which of the following is a source of emissions leading to the eventual formation of surface ozone as a pollutant?

(A) Transport sector
(B) Refrigeration and Air conditioning
(C) Wetlands
(D) Fertilizers

Answer: A

The smog in cities in India mainly consists of

(A) Oxides Of sulphur
(B) Oxides of nitrogen and unburnt hydrocarbons
(C) Carbon monoxide and SPM
(D) Oxides of sulphur and ozone

Answer: B

Which of the following types of natural hazards have the highest potential to cause damage to humans?

(A) Earth quakes
(B) Forest fires
(C) Volcanic eruptions
(D) Droughts and Floods

Answer: D

The percentage share of renewable energy sources in the power production in India is around

(A) 2-3 %
(B) 22-25%
(C) l0-12%
(D) Less than 1%

Answer: C

In terms of total CO2 emissions from a country, identity the correct sequence:

(A) U.S.A. > China > India > Russia
(B) China > U.S.A. > India > Russia
(C) China > U.S.A. > Russia > India
(D) U.S.A. > China > Russia > India

Answer: B

Match List โ€“ I and List โ€“ II and identify the correct code:

List I
a. World Health Day
b. World Population Day
c. World Ozone Day
d. World AIDS Day
List II
i. 16th September
ii. 1st December
iii. 11th July
iv. 7th April

(A) i ii iii iv
(B) iv iii i ii
(C) ii iii iv i
(D) iii iv ii i

Answer: B

Which of the anthropogenic activity accounts for more than 2/3rd of global water consumption?

(A) Agriculture
(B) Hydropower generation
(C) Industry
(D) Domestic and Municipal usage

Answer: A

One of the anthropogenic sources of gaseous pollutants chlorofluorocarbons (CFCs) in air is

(A) Cement Industry
(B) Fertiliser industry
(C) Foam industry
(D) Pesticide industry

Answer: C

The cyclone โ€œHudhudโ€ hit the coast of which State?

(A) Andhra Pradesh
(B) Karnataka
(C) Kerala
(D) Gujarat

Answer: A

Which of the following is not a renewable natural resource?

(A) Clean air
(B) Fresh water
(C) Fertile soil
(D) Salt

Answer: D

Which of the following cities has been worst affected by urban smog in recent times?

(A) Paris
(B) London
(C) Los Angeles
(D) Beijing

Answer: D

The primary source of organic pollution in fresh water bodies is

(A) run-off urban areas
(B) run-off from agricultural forms
(C) sewage effluents
(D) industrial effluents

Answer: C

โ€˜Laharโ€™ is a natural disaster involving

(A) eruption of large amount of material
(B) strong winds
(C) strong water waves
(D) strong winds and water waves

Answer: A

The population of India is about 1.2 billion. Take the average consumption of energy per person per year in India as 30 Mega Joules. If this consumption is met by carbon based fuels and the rate of carbon emissions per kilojoule is l5 x I06 kgs, the total carbon emissions per year from India will be

(A) 54 million metric tons
(B) 540 million metric tons
(C) 5400 million metric tons
(D) 2400 million metric tons

Answer: Marks given to all

The National Disaster Management Authority functions under the Union Ministry of

(A) Environment
(B) Water Resources
(C) Home Affairs
(D) Defence

Answer: C

Match List โ€“ I and List โ€“ II and select the correct answer from the codes given below:

List โ€“ I
a. Flood
b. Drought
c. Earthquake
d. Volcano
List โ€“ II
1. Lack of rainfall of sufficient duration
2. Tremors produced by the passage of vibratory waves through the rocks of the earth
3. A vent through which molted substances come out
4. Excess rain and uneven distribution of water

Codes: a b c d
(A) 4 1 2 3
(B) 2 3 4 1
(C) 3 4 2 1
(D) 4 3 1 2

Answer: A

Which one of the following green house gases has the shortest residence time in the atmosphere?

(A) Chlorofluorocarbon
(B) Carbon dioxide
(C) Methane
(D) Nitrous oxide

Answer: C

In order to avoid catastrophic consequences of climate change, there is general agreement among the countries of the world to limit the rise in average surface temperature of earth compared to that of pre-industrial times by

(A) 1.5 oC to 2 oC
(B) 2.0 oC to 3.5 oC
(C) 0.5 oC to 1.0 oC
(D) 0.25 oC to 0.5 oC

Answer: A

Consider the following statements and select the correct answer from the code given below:

i. Rajasthan receives the highest solar radiation in the country.
ii. India has the fifth largest installed wind power in the world.
iii. The maximum amount of wind power is contributed by Tamil Nadu.
iv. The primary source of uranium in India is Jaduguda

Codes:
(A) i and ii
(B) i, ii and iii
(C) ii and iii
(D) i and iv

Answer: Marks given to all

Inside rural homes, the source/sources of Nitrogen Oxide Pollution may be:

(a) Unvented gas stoves
(b) Wood stoves
(c) Kerosene heaters

Choose the correct code:
(A) (a) and (b) only
(B) (b) and (c) only
(C) (b) only
(D) (a), (b) and (c)

Answer: D

Which of the following pollutants can cause cancer in humans ?

(A) Pesticides
(B) Mercury
(C) Lead
(D) Ozone

Answer: B

Assertion (A): People population control measures do not necessarily help in checking environmental degradation. Reason (R): The relationship between population growth and environmental degradation is rather complex.

Choose the correct answer from the following
(A) Both (A) and (R) are true and (R) is the correct explanation of (A).
(B) Both (A) and (R) are true but (R) is not the correct explanation of (A).
(C) (A) is true but (R) is false.
(D) (A) is false but (R) is true.

Answer: A

Which of the following phenomena is not a natural hazard?

(A) Wildfire
(B) Lightning
(C) Landslide
(D) Chemical contamination

Answer: D

As part of National Climate Change Policy, Indian government is planning to raise the installed capacity of renewable energy by the year 2030 to

(A) 175 GW
(B) 200 GW
(C) 250 GW
(D) 350 GW

Answer: D

At present, in terms of per capita energy consumption (kWh/year), identify the correct sequence.

(A) Brazil > Russia > China > India
(B) Russia > China > India > Brazil
(C) Russia > China > Brazil > India
(D) China > Russia > Brazil > India

Answer: C

The factors which are most important in determining the impact of anthropogenic activities on environment are:

(A) Population, forest cover and land available per person
(B) Population, affluence per person, land available per person
(C) Population, affluence per person and the technology used for exploiting resources
(D) Atmospheric conditions, population and forest cover

Answer: C

Indiaโ€™s contribution to total global carbon dioxide emissions is about:

(A) ~15%
(B) ~3%
(C) ~6%
(D) ~10%

Answer: C

In which of the countries per capita use of water is maximum?

(A) India
(B) USA
(C) European Union
(D) China

Answer: C

In the recently launched Air Quality Index in India, which of the following pollutants is not included?

(A) Chlorofluorocarbons
(B) Carbon monoxide
(C) Fine particulate matter
(D) Ozone

Answer: A

Which of the following combinations represent renewable natural resources?

(A) Oil, forests and tides
(B) Fertile soil, fresh water and natural gas
(C) Clean air, phosphates and biological diversity
(D) Fishes, fertile soil and fresh water

Answer: D

Inductive reasoning is grounded on:

(A) Harmony of nature
(B) Integrity of nature
(C) Unity of nature
(D) Uniformity of nature

Answer: D

If we want to seek new knowledge of facts about the world, we must rely on reason of the type:

(A) Physiological
(B) Inductive
(C) Deductive
(D) Demonstrative

Answer: B

The arsenic pollution in ground water in Bangladesh and parts of India is due to

(A) industrial waste
(B) thermal power plants
(C) natural sources
(D) agricultural practices

Answer: C

Among the following air pollutants which directly impacts human health as well as climate ?

(A) Soot
(B) Sulphur dioxide
(C) Oxides of nitrogen
(D) Chlorofluoro carbons

Answer: A

Assertion (A): The problems of environmental sustainability are difficult to resolve. Reason (R): Our understanding how environment works and how different human choices affect environment is inadequate.

Choose the correct code:
(A) Both (A) and (R) are correct and (R) is the correct explanation of (A).
(B) Both (A) and (R) are correct, but (R) is not the correct explanation of (A).
(C) (A) is true and (R) is false.
(D) (A) is false and (R) is true.

Answer: A

The percentage share of electrical power from solar energy in the total electric power generation from all renewable energy sources sought to be achieved by Indian government by the year 2022, is

(A) ~ 57.1%
(B) ~ 65.5%
(C) ~ 47.5%
(D) ~ 75%

Answer: A

As part of Intended Nationally Determined contributions, Indian government through additional forest and tree cover has resolved to create an additional carbon sink (in terms of carbon dioxide equivalent) by year 2030 to the extent of

(A) 3.5 to 4 billion tons
(B) 2.5 to 3 billion tons
(C) 1.5 to 2 billion tons
(D) 1 to 1.5 billion tons

Answer: B

Which of the following is a meteorological hazard ?

(A) Avalanche
(B) Coastal erosion
(C) Landslide
(D) Blizzard

Answer: D

Which of the following is the largest source of water pollution in major rivers of India?

(A) Untreated sewage
(B) Agriculture run-off
(C) Unregulated small scale industries
(D) Religious practices

Answer: A

Sustainable development goals have specific targets to be achieved by

(A) 2022
(B) 2030
(C) 2040
(D) 2050

Answer: B

Indian governmentโ€™s target of producing power from biomass by the year 2022, is

(A) 50 MW
(B) 25 MW
(C) 15 MW
(D) 10 MW

Answer: Marks given to all

Assertion (A): Conserving our soil resources is critical to human survival. Reason (R): Soil is home to many micro-organisms and contains minerals.

Choose the correct code:
(A) Both (A) and (R) are correct and (R) is the correct explanation of (A).
(B) Both (A) and (R) are correct but (R) is not the correct explanation of (A).
(C) (A) is true and (R) is false. (D) (A) is false and (R) is true.

Answer: B

World Meteorological Organizationโ€™s (WMO) objective has been to reduce the number of deaths due to hydro meteorological disasters over the decade 2010-2019 by (with reference to the decade 1994-2003)

(A) 25%
(B) 50%
(C) 75%
(D) 80%

Answer: B

Identify the air pollutant in urban areas which irritates eyes and also respiratory tract of human beings.

(A) Particulate matter
(B) Oxides of nitrogen
(C) Surface ozone
(D) Carbon monoxide

Answer: C

Consider the following statement and select the correct code stating the nature of the argument involved in it: To suppose that the earth is the only populated world in the infinite space is as absurd as to assert that in an entire field of millet only one grain will grow.

(A) Astronomical
(B) Anthropological
(C) Deductive
(D) Analogical

Answer: D

Among the following fuels of energy, which is the most environment friendly?

(1) Hydrogen
(2) Ethanol
(3) Biogas
(4) CNG

Answer: 1

Occurrence of natural hazards is affected by:

(a) Land use changes
(b) Drainage and construction
(c) Ozone depletion
(d) Climate change

Choose the correct answer from the codiven below :
(1) (b), (c) and (d)
(2) (a), (c) and (d)
(3) (a), (b) and (c)
(4) (a), (b) and (d)

Answer: 4

Which of the following pollutant gases is not produced both naturally and as a result of industrial activity?

(1) Carbon dioxide
(2) Chlorofluoro carbons
(3) Nitrous oxide
(4) Methane

Answer: 2

Assertion (A): In urban areas, smog episodes occur frequently in winters. Reason (R): In winters, a lot of biomass is burnt by people for heating purposes or to keep themselves warm.

Choose the correct answer from the code given below :
(1) Both (A) and (R) are false
(2) Both (A) and (R) are true and (R) is the correct explanation of (A)
(3) Both (A) and (R) are true but (R) is not the correct explanation of (A)
(4) (A) is true and (R) is false

Answer: 3

Assertion(A):Sustainable development is critical to well being of human society. Reason (R): Environmentally sound policies do not harm the environment or deplete the natural resources.

(1) Both (A) and (R) are correct and (R) is the correct explanation of (A).
(2) Both (A) and (R) are correct, but (R) is not the correct explanation of (A).
(3) (A) is true and (R) is false.
(4) (A) is false and (R) is true.

Answer: 2

The dominant source of pollution due to oxides of nitrogen (NOx ) in urban areas is

(1) Road transport
(2) Commercial sector
(3) Energy use in industry
(4) Power plants

Answer: 1

Which of the following is not a water-brone disease?

(1) Typhoid
(2) Hepatitis
(3) Cholera
(4) Dengue

Answer: 4

Which of the following natural hazards is not hydro-meteorological?

(1) Snow avalanche
(2) Sea erosion
(3) Tropical Cyclone
(4) Tsunami

Answer: 4

โ€˜Fly ashโ€™ produced in thermal power plants is an ecofriendly resource for use in:

(a) agriculture as micro-nutrient
(b) wasteland development
(c) dam and water holding structures
(d) brick industry

Choose the correct answer from the code given below:
(1) (a), (b) and (d) only
(2) (b), (c) and (d) Only
(3) (a), (c) and (d) Only
(4) (a), (b), (c) and (d)

Answer: 4

Which of the following types of natural disasters has no definite beginning and end?

(1) Earthquakes
(2) Landsides
(3) Hurricanes
(4) Droughts

Answer: 4

Assertion (A): Indoor air pollution is serious hazard. Reason (R): The dispersal of air pollutants is rather limited in indoor environment.

Choose the correct answer from the code given below:

(1) Both (A) and (R) are true and (R) is the correct explanation of (A)
(2) Both (A) and (R) are true and (R) is not the correct explanation of (A)
(3) (A) is true and (R) is false.
(4) Both (A) and (R) are false.

Answer: 1

In terms of their contribution to the total power generation in India, Identify the correct sequence of energy sources โ€“ Thermal Power Plants (TPP), Large Hydropower Projects (LHP), Nuclear Energy (NE) and Renewable Energy (RE) which includes solar energy, wind energy, biomass and small hydropower projects.

(1) TPP > RE > LHP > NE
(2) TPP > LHP > RE > NE
(3) LHP > TPP > RE > NE
(4) LHP > TPP > NE > RE

Answer: 2

Which of the following is considered as major source of pollution in rivers of India?

(1) Unregulated small scale industry
(2) Untreated sewage
(3) Agricultural run-off
(4) Thermal power plants

Answer: 2

The annual range of temperature in the interior of the continents is high compared to coastal areas. What is/are the reason(s)?

(a) Thermal difference between land and water.
(b) Variations in altitude between continents and oceans.
(c) Presence of strong winds in the interior.
(d) Heavy rains in the interior as compared to coastal region.

1. (a) and (b) only
2. (b) and (c) only
3. (a) only
4. (a), (b), (c) and (d)

Answer: 3

A source of renewable power which can be developed with minimum cost is :

1. Wind mills
2. Tidal power
3. Biomass power
4. Geothermal energy

Answer: 3

How much carbon is being added to the atmosphere by burning of fossil fuels ?

1. 6-9 billion tons
2. 9-12 billion tons
3. 4-6 billion tons
4. 2-6 billion tons

Answer: 1

Natural vegetation of an area needs to be preserved for the future, mainly for the purpose of :

1. Providing habitat for birds
2. Preventing soil erosion
3. Providing material for breeding new species
4. Study by scientists

Answer: 3

In large parts of Eastern India, the ground water is contaminated by

(1) Arsenic
(2) Lead
(3) Mercury
(4) Nickel

Answer: 1

Taj Mahal is mainly threatened by the deleterious effects of

(1) Sulphur dioxide
(2) Chlorine
(3) Oxygen
(4) Hydrogen

Answer: 1

Efficiency of electrical power generation is least amongst the following sources of energy in case of

(1) Solar
(2) Wind
(3) Geothermal
(4) Nuclear

Answer: 3

Which among the following natural hazards has relatively slow onset?

1. Volcanic eruptions
2. Droughts
3. Wild free
4. Land and slides

Answer: 2

Under Kyoto protocol, the parties, in the second commitment period, have to reduce their greenhouse gas emissions with reference to 1990 levels by atleast

1. 10%
2. 12%
3. 15%
4. 18%

Answer: 4

Major Sources of oxygen demanding waste in fresh water systems are

1. Agricultural run-off
2. Food Processing
3. Ore Mining
4. Sewage effluent
5. Landfill sites

Choose the correct options from these given below:
1. (a), (d) and (e)
2. (b), (d) and (e)
3. (a), (b) and (d)
4. (a), (b), (c) and (d)

Answer: 3

Which of the following is renewable natural resources?

1. Biological University
2. Tinโ€™
3. Natural Gas
4. Salt

Answer: 1

Biomass fuels have the potential to form a sustainable carbon neutral energy source because they

1. Produce carbon dioxide in combustion as much as they consume when they grow
2. Produce less amount of carbon dioxide on combustion compared to the amount they use during their growth
3. Have carbon dioxide content same as fossil fuels
4. Do not produce hazardous emissions on combustion

Answer: 1

Dissolved oxygen (DO), an important parameter of water quality, is essential for survival of

1. Humans
2. Animals
3. Fish
4. Crops and vegetables

Answer: 3

Among the following elements which is typically the most abundant in dried sewage sludge?

1. Total nitrogen
2. Total Sulphur
3. Calcium
4. Total phosphorous

Answer: 3

When did the United Nations Environment Program (UNEP) and the International Program in Environmental Education (IEEP) come into existence?

1. 1972
2. 1975
3. 1982
4. 1992

Answer: None

Identify the correct sequence of sectoral global CO2 emissions in increasing order as per IPCC (2014) report

A. Electricity and heat production
B. Buildings
C. Transportation
D. Industry

Choose the correct answer from the options given below
1. B, C, D, A
2. B, C, A, D
3. B, D, C, A
4. B, D, A, C

Answer: 1

Maximum soot is released from

(A) Petrol vehicles
(B) CNG vehicles
(C) Diesel vehicles
(D) Thermal Power Plants

Answer: C

Surface Ozone is produced from

(A) Transport sector
(B) Cement plants
(C) Textile industry
(D) Chemical industry

Answer: A

Which one of the following non-conventional energy sources can be exploited most economically ?

(A) Solar
(B) Wind
(C) Geo-thermal
(D) Ocean Thermal Energy Conversion (OTEC)

Answer: B

The most recurring natural hazard in India is

(A) Earthquakes
(B) Floods
(C) Landslides
(D) Volcanoes

Answer: B

Which type of natural hazards cause maximum damage to property and lives ?

(A) Hydrological
(B) Hydro-meteorological
(C) Geological
(D) Geo-chemical

Answer: B

Dioxins are produced from

(A) Wastelands
(B) Power plants
(C) Sugar factories
(D) Combustion of plastics

Answer: D

The slogan โ€œA tree for each childโ€ was coined for

(A) Social forestry programme
(B) Clean Air programme
(C) Soil conservation programme
(D) Environmental protection programme

Answer: A

The main constituents of biogas are

(A) Methane and Carbon di-oxide
(B) Methane and Nitric oxide
(C) Methane, Hydrogen and Nitric oxide
(D) Methane and Sulphur di-oxide

Answer: A

Assertion (A) : In the world as a whole, the environment has degraded during past several decades. Reason (R) : The population of the world has been growing significantly.

(A) (A) is correct, (R) is correct and (R) is the correct explanation of (A).
(B) (A) is correct, (R) is correct and (R) is not the correct explanation of (A).
(C) (A) is correct, but (R) is false.
(D) (A) is false, but (R) is correct.

Answer: B

Climate change has implications for

1. soil moisture
2. forest fires
3. biodiversity
4. ground water

Identify the correct combination according to the code : Codes :
(A) 1 and 3
(B) 1, 2 and 3
(C) 1, 3 and 4
(D) 1, 2, 3 and 4

Answer: B

Which of the following is not covered in 8 missions under the Climate Action Plan of Government of India ?

(A) Solar power
(B) Waste to energy conversion
(C) Afforestation
(D) Nuclear energy

Answer: D

The concentration of Total Dissolved Solids (TDS) in drinking water should not exceed

(A) 500 mg/L
(B) 400 mg/L
(C) 300 mg/L
(D) 200 mg/L

Answer: A

โ€˜Chipkoโ€™ movement was first started by

(A) Arundhati Roy
(B) Medha Patkar
(C) Ila Bhatt
(D) Sunderlal Bahuguna

Answer: D

The constituents of photochemical smog responsible for eye irritation are

(A) SO2 and O3
(B) SO2 and NO2
(C) HCHO and PAN
(D) SO2 and SPM

Answer: C

Assertion (A) : Some carbonaceous aerosols may be carcinogenic.
Reason (R) : They may contain polycyclic aromatic hydrocarbons (PAHs).

(A) Both (A) and (R) are correct and (R) is the correct explanation of (A).
(B) Both (A) and (R) are correct but (R) is not the correct explanation of (A).
(C) (A) is correct, but (R) is false.
(D) (A) is false, but (R) is correct.

Answer: A

The two major processes involved in the carbon cycle are: Options:-

(A) Photosynthesis and respiration
(B) Deposition and erosion
(C) Fixation and denitrification
(D) Evaporation and transpiration

Answer: A

Which of the following methods would be most appropriate in reducing acid rain and acid deposition problems? Options:-

(A) Adding lime to the acidic lakes
(B) Promotion of acid-resistant crops
(C) Increasing the height of smokestacks
(D) Reducing the use of fossil fuels

Answer: D

Which of the following best describes the mechanism of the greenhouse effect in earthโ€™s atmosphere? Options:-

(A) Ultraviolet radiation from the sun is absorbed by Ozone layer in the stratosphere.
(B) Gamma radiation from the sun is absorbed at ground level by dust particles in the atmosphere.
(C) Infrared radiation from earthโ€™s surface is absorbed by gases in the atmosphere.
(D) Cosmic radiation from the space is absorbed by gases in the atmosphere.

Answer: C

Assertion โ€œAโ€: Mapping of landslide prone areas and construction of houses, falling of trees and grazing in landslide prone areas should be prohibited.
Reason โ€œRโ€: Afforestation in the vulnerable areas is an effective way of durable landslides control. Choose the correct code:

Options:-
(A) โ€œAโ€ is false and โ€œRโ€ is true.
(B) โ€œAโ€ is true and โ€œRโ€ is false.
(C) Both โ€œAโ€ and โ€œRโ€ are correct, but โ€œRโ€ is not the correct explanation of โ€œAโ€.
(D) Both โ€œAโ€ and โ€œRโ€ are correct, and โ€œRโ€ is the correct explanation of โ€œAโ€.

Answer: D

Human Populations have historically settled in flood plains due to which of the following reasons?

(a) Flood plains are close to rivers for transportation.
(b) The soils in the flood plains are usually fertile.
(c) The terrain in flood plains is found as flat.

Select the correct answer form the code given below: Options:-
(A) (a) and (b) only
(B) (b) and (c) only
(C) (a) only
(D) (a), (b) and (c)

Answer: D

Choose the word which is different from the rest: Options:-

(A) Aravali Hills
(B) Nilgiri Hills
(C) Shivalik Hills
(D) Mount Kilimanjaro

Answer: D

A source of renewable power which can be developed with minimum cost is: Options:-

(A) Geothermal energy
(B) Tidal power
(C) Wind mills
(D) Biomass power

Answer: D

Natural vegetation of an area needs to be preserved for the future, mainly for the purpose of: Options:-

(A) Providing material for breeding new species
(B) Study by scientists
(C) Preventing soil erosion
(D) Providing habitat for birds

Answer: A

How much carbon is being added to the atmosphere by burning of fossil fuels? Options:-

(A) 6 โ€” 9 billion tons
(B) 2 โ€” 6 billion tons
(C) 9 โ€” 12 billion tons
(D) 4 โ€” 6 billion tons

Answer: A

The annual range of temperature in the interior of the continents is high compared to coastal areas. What is/are the reason(s)?

(a) Thermal difference between land and water.
(b) Variations in altitude between continents and oceans.
(c) Presence of strong winds in the interior.
(d) Heavy rains in the interior as compared to coastal region.

Options:-
(A) (a) only
(B) (a), (b), (c) and (d)
(C) (b) and (c) only
(D) (a) and (b) only

Answer: A

In which part of India, the tropical western Ghats are situated? Options:-

(A) Punjab
(B) Kerala
(C) Rajasthan
(D) Assam

Answer: B

International Drinking Water Supply and Sanitation Decade (IDWSSD) was observed between: Options:-

(A) 1991-2000
(B) 1971-1980
(C) 1965-1975
(D) 1981-1990

Answer: D

Consider the following statements:

(a) The radiation reflected back to the atmosphere is called albedo.
(b) The atmosphere is essential in maintaining the heat balance of the body.
(c) The heat and sunlight both pass through the atmosphere.

Which of the statements(s) given above is/are correct? Options:-
(A) (a) and (b)
(B) (a), (b) and (ยข)
(C) Only (a)
(D) (b) and (c)

Answer: B

Assertion โ€œAโ€: Environmental and social impact assessments never overlap with each other.
Reason โ€œRโ€: Environmental and social impact assessments are effectively opposite ends of the same spectrum.

Options:
(A) โ€œAโ€ is false, but โ€œRโ€ is true.
(B) Both โ€œAโ„ข and โ€œRโ€ is not the correct explanation of โ€œAโ€.
(C) Both โ€œAโ€ and โ€œRโ€ are true and โ€œRโ€ is the correct explanation of โ€œAโ€.
(D) โ€œAโ€ is true, but โ€œRโ€ is false.

Answer: A

What is the long term effect of cutting down larger arcas of rain forests? Options:-

(A) Increased rate of soil erosion
(B) Increased rainfall in these areas
(C) Decreased carbon dioxide content in the air
(D) Decreased flooding of low-laying land

Answer: A

Recent study showed that, River Ganga will not flow cleaner in the near future. Which of the following reasons is false? Options:-

(A) Climatic change
(B) Increased sewage
(C) Failure of treatment plants
(D) Thinning of Flow rate of the river

Answer: A

Assertion โ€œAโ€: Hails damage the standing crops.
Reason โ€œRโ€: In India, hails occur only during the winter season

Options:-
(A) โ€œAโ€ is false, but โ€œRโ€ is true.
(B) โ€œAโ€ is true, but โ€œRโ€ is false.
(C) Both โ€œAโ€ and โ€œRโ€ are true but โ€œRโ€ is not the correct explanation of โ€œAโ€.
(D) Both โ€œAโ€ and โ€œRโ€ are true and โ€œRโ€ is the correct explanation of โ€œAโ€.

Answer: C

Least polluting energy generating technique among the following is: Options:-

(A) Fission based nuclear energy
(B) Thermal power
(C) Photovoltaic
(D) Magneto-hydrodynamic

Answer: C

Largest amount of freshwater is present in : Options:-

(A) Glaciers
(B) Rivers
(C) Lakes
(D) Polar ice

Answer: A

Which of the following statements are indicative of the vision of Blue- Revolution? Code:

(a) Promoting development of Botanical Gardens.
(b) Creating an enabling environment for the full potential of fisheries in the country.
(c) Improving the income status of fish farmers keeping in view the sustainability concerns.
(d) Encouraging scientific research in forestry to increase the vegetation cover.
(e) Promoting integrated development of fisheries of the country.
(f) Undertaking research in new areas like marine finfish.

Options:
(A) (b), (c) and (e)
(B) (b), (c) and (a)
(C) (a), (d) and (ยฉ)
(D) (c), (d) and (e)

Answer: A

What percentage of energy coming from the Sun is re-radiated towards the space?

(A) 30%
(B) 15%
(C) 50%
(D) 25%

Answer: A

The most harmful types of environmental pollutants are:

(A) Natural nutrients present in excess
(B) Wastes from faecal matter
(C) Non-biodegradable chemicals
(D) Human organic wastes

Answer: C

As per Central Forestry Commission (CFC), how many forest types are found in our country ?

(A) 28
(B) 15
(C) 16
(D) 20

Answer: C

Assertion โ€œAโ€: Natural gas is a very attractive eco-friendly fuel.
Reason โ€œRโ„ข: It produces few pollutants and less carbon dioxide per unit energy than any other fossil fuel on combustion.

Code :
(A) โ€œAโ€ is false, but โ€œRโ€ is true.
(B) Both โ€œAโ€ and โ€œRโ€ are true and โ€œRโ€ is the correct explanation of โ€œAโ€.
(C) โ€œAโ€ is true, but โ€œRโ€ is false.
(D) Both โ€œAโ€ and โ€œRโ€ are true and โ€œRโ€ is not the correct explanation of โ€œAโ€.

Answer: B

A study was carried out on the drought resilience in India. Which of the following States show highest level of handling drought ?

(A) Sikkim, Punjab, Arunachal Pradesh
(B) Jharkhand, Chhattisgarh, Jammu and Kashmir
(C) Odisha, Karnataka, Kerala
(D) Telangana, Andhra Pradesh, Tamil Nadu

Answer: A

An earthquake is rated as โ€˜majorโ€™ id its magnitude in Richter Scale is in the range of: Options:-

(A) 6.0-6.9
(B) 4.0-4.9
(C)7.0-7.9
(D) 5.0-5.9

Answer: C

Assertion โ€œAโ€: Climate change is going to increase social tension in India.
Reason โ€œRโ€: The frequency and intensity of the extreme weather events will have serious consequences for food security. Options:-

(A) Both โ€œAโ€ and โ€œRโ€ are true and โ€œRโ€ is not the correct explanation of โ€œAโ€.
(B) โ€œAโ€ is false, but โ€œRโ€ is true.
(C) โ€œAโ€ is true, but โ€œRโ€ is false.
(D) Both โ€œAโ€ and โ€œRโ€ are true and โ€œRโ€ is the correct explanation of โ€œAโ€.

Answer: D

Which among the following industries, consumes maximum water in India? Options:-

(A) Thermal power plants
(B) Engineering
(C) Paper and pulp
(D) Textiles

Answer: A

The biggest hindrance in using biomass as a major energy source is: Options:-

(A) Air pollution due to combustion.
(B) Technology not well developed for commercialization.
(C) Large amount of land required to grow energy crops.
(D) Energy yield of low level.

Answer: A

Plants suitable for biomonitoring of Sulphur Dioxide pollution are: Options:-

(A) White pine, moss and lichens
(B) Tobacco, grapes and garden bean
(C) Apricot, peach and gladiolus
(D) Tomato and lettuce

Answer: A

One of the following has a positive human influence on the environment: Option:-

(A) Pollution
(B) Deforestation
(C) Conservation
(D) Over population

Answer: C

Assertion โ€œAโ€: Hydropower is an inexhaustible energy resource.
Reason โ€œRโ€: The power of falling water would neither diminish nor get exhausted.

Options:-
(A) Both โ€œAโ€ and โ€œRโ€ are true but โ€œRโ€ is not, the correct explanation of โ€œAโ€
(B) Both โ€œAโ€ and โ€œRโ€ are false
(C) โ€œAโ€ is true, but โ€œRโ€ is false
(D) Both โ€œAโ€ and โ€œRโ€ are true and โ€œRโ€ is the correct explanation of โ€œAโ€

Answer: D

Assertion โ€œAโ€: The environmental impact of soil erosion can best be mitigated by removing vegetative cover only from the specific site on which construction is to take place.
Reason โ€œRโ„ข: The environmental impact of soil erosion can also be mitigated by disturbing the vegetation in adjacent areas as little as possible.

Options:-
(A) Both โ€œAโ€ and โ€œRโ€ are true but โ€œRโ€ is not, the correct explanation of โ€œAโ€
(B) Both โ€œAโ€ and โ€œRโ€ are true and โ€œRโ€ is the correct explanation of โ€œAโ€
(C) โ€œAโ€ is true, but โ€œRโ€ is false
(D) โ€œAโ€ is false, but โ€œRโ€ is true

Answer: B

In the scheme of life, soil is an essential resource of life, as it plays the following roles.

(a) It is a reservoir of carbon.
(b) It is the medium for growth of food and energy for the animal world.
(c) It provides oxygen to the biotic community.
(d) It is a natural reservoir for the huge amont of water.

Select the correct answer from the above: Options:-
(A) (b), (c) and (d)
(B) (a), (b) and (d)
(C) (a), (c) and (d)
(D) (a), (b) and (c)

Answer: B

Below are given a number of roles and functions at the government/non- government level indicating the specific responsibilities of the Ministry of Drinking Water and Sanitation. Identify those statements which indicate the areas of responsibility of this ministry. Select from the code to give your answer: Options:

(a) Overall policy and planning in respect programmes of drinking water and sanitation.
(b) Funding in respect of the programmes listed in (a).
(c) Coordination of programmes floated.
(d) Supply of food grains at reasonable price to the consumers.
(e) Achieving the goal of zero hunger,
(f) Implementation of Swachh Bharat Abhiyan in rural India.
(g) Monitoring of prices and availability of essential commodities.

Options:-
(A) (a), (c), (d) and (e)
(B) (a), (b), (d) and (e)
(C) (a). (b). (c) and (f)
(D) (a). (b), (c) and (g)

Answer: C

Following are two sets of items. Set-I lists different types of washes, while Set-II indicates method of their disposal. Match the two sets and given your answer from the code given below:

Set-I (Types of Waste)
(a) Dry Waste
(b) Wet Waste
(c) Toxic Waste
(d) Soiled Waste
Set-II (Methods of Disposal)
(i) By composting
(ii) By burying in landfills
(iii) By dropping them in corporation bins.
(iv) By recycling

Options:-
(A) (a)-(v), (b)-(i), (c)-(ii), (d)-(iii)
(B) (a)-(ii), (b)-(iii), (c)-(iv), (d)-(i)
(C) (a)-(i). (b)-(ii). (c)-(iii). (d)-(iv)
(D) (a)-(iii). (b)-(iv). (c)-(i). (d)-(ii)

Answer: A

The purpose underlying the Renewable Energy Education programmes are: Options:-

(A) Preventive and ameliorative
(B) Educative and investigative
(C) Corrective and predictive
(D) Diagnostic and evaluative

Answer: B

In the two sets given below, set-I specifies level in a water sample, while set โ€” II provides observations. Match the two sets and indicate your answer from the code given below.

Set โ€” I (H:S level in water sample)
(a) No HsS is produced
(b) H,S is produced and is small in quantity
(c) HsS is produced and is medium in quantity
(d) HS is produced and is high in quantity
Set โ€” II (Observation)
(1) Dark black โ€” high water pollution
(ii) No colour โ€” clean water
(iii) Light black โ€“ low water pollution
(iv) Black โ€” moderate water pollution

Options: (a) (b) (c) (d)
(A) (i) (ii) (iii) (iv)
(B) (iv) (i) (ii) (iii)
(C) (iii) (iv) (i) (ii)
(D) (ii) (iii) (iv) (i)

Answer: D

Which of the following statements are indicative of the vision of Blueโ€”Revolution? Code:

(a) Promoting development of Botanical Gardens.
(b) Creating an enabling environment for the full potential of fisheries in the country.
(c) Improving the income status of fish farmers keeping in view the sustainability concerns.
(d) Encouraging scientific research in forestry to increase the vegetation cover.
(e) Promoting integrated development of fisheries of the country.
(f) Undertaking research in new areas like marine finfish.

Options:
(A) (b), (c) and (e)
(B) (b), (c) and (a)
(C) (a), (d) and (f)
(D) (c), (d) and (e)

Answer: A

Below are given two sets. Set โ€” I mentions the types of pollution, while Set โ€” II indicates their source. Match the two sets and selects your answer from the code.

Set โ€” I (Pollution Type)
(a) Air
(b) Land
(c) Water
(d) Noise
Set โ€” II (Source)
(i) Point and non โ€” point source such as discharges from industries etc.
(ii) Industries thermal power plants and motor vehicles emissions.
(iii) Roadway, aircraft, industrial as well as high intensity sonar.
(iv) Over use of chemical fertilizers.

Options: (a) (b) (c) (d)
(A) (iii) (i) (ii) (iv)
(B) (i) (ii) (iii) (iv)
(ยฉ) (ii) (iv) (i) (iii)
(D) (iv) (iii) (ii) (i)

Answer: B

Which of the following provisions are directly covered in the Environment Protection Act, 1986?

(A) Protection and conservation of forest and matters connected therewith
(B) Protection and safety of wild animals, birds, and plants and for matters connected therewith
(C) Protection and improvement of quality environment and preventing, controlling, and abating environmental pollution
(D) Prevention and control of water pollution

Answer: B

Which of the following gases initiates a chain reaction that breaks-down ozone in the upper atmosphere? Options:-

(A) Carbon dioxide
(B) Nitrogen Dioxide
(C) Hydrogen sulphide
(D) Chlorine

Answer: D

Assertion โ€œAโ€: The ecosystem surrounding a river gets damaged due to construction of a dam.
Reason โ€œRโ€: The area gets inundated with large volume of water.

Choose the correct code: Options:-
(A) โ€œAโ€ is false, but โ€œRโ€ is true.
(B) Both โ€œAโ€ and โ€œRโ€ are true, but โ€œRโ€ is not the correct explanation of โ€œAโ€.
(C) โ€œAโ€ is true, but โ€œRโ€ is false.
(D) Both โ€œAโ€ and โ€œRโ€ are true, and โ€œRโ€ is the correct explanation of โ€œAโ€.

Answer: D

High level lead (Pb) exposures in humans through inhalations or food consumption may cause:

(a) Mental retardation
(b) High blood pressure
(c) Disorder of central nervous system

Select the correct answer from the code given below: Options:-
(A) (a), (b) and (c)
(B) (a) and (b) only
(C) (c) and (a) only
(D) (b) and (c) only

Answer: A

The tsunami that occurred in south and south-east Asia in December, 2004 was caused due to: Options:-

(A) A hurricane
(B) An earthquake
(C) A volcanic eruption
(D) A tropical cyclone

Answer: B

In a human population, which is undergoing the demographic transition, which of the following generally decreases first? Options:-

(A) Life expectancy
(B) Death rate
(C) Level of education
(D) Birth rate

Answer: B

What is SWAYAM?

(A) Non-Governmental Organization
(B) Digital Programme to achieve the principles of education
(C) On-line platform
(D) Name of a web site

Answer: B

Choose the set of non-renewable resources.

(A) Uranium, natural gas, soil
(B) Natural gas, uranium, phosphates
(C) only A
(D) Both A and B

Answer: D

One of the parameters used to characterize the air quality at a location is PM2.5. Here, the suffix 2.5 refers to:

(A) average number of suspected particles in 10 cm3 of air
(B) size of suspended particles in certain units
(C) concentration of oxides of sulphur and nitrogen
(D) concentration of suspended particles in 2.5 m3 of air

Answer: B

Which of the following diseases are due to polluted water?

(a) Giardia
(b) Dengue
(c) Amoebiasis
(d) Typhoid

Choose the correct answer from the options given below:
(A) (a), (b) and (d)
(B) (b), (c) and (d)
(C) (a), (b), (c) arid (d)
(D) (a), (c) and (d)

Answer: D

Which of the following is non-vehicular pollutant?

(A) chlorofluorocarbon
(B) carbon-monoxide
(C) hydrocarbon
(D) particulate matter

Answer: A

Which of the following is the largest sink of CO, gas?

(A) Forests
(B) Oceans
(C) Ice sheets
(D) Grasslands

Answer: B

Montreal protocol aims at:

(A) Reduction in emissions of greenhouse gases.
(B) Phasing out ozone depleting substances.
(C) Prohibiting transboundary movement of hazardous waste.
(D) Enhancing cooperation among UN member states for peaceful uses of nuclear energy.

Answer: B

Statement I: Earthquake โ€˜Aโ€™ is 5 on Richter scale and Earthquake โ€˜Bโ€™ is 8 on the same scale. Earthquake โ€˜Bโ€ has 1000 times the wave amplitude compared to โ€˜Aโ€™.
Statement II: The energy released in โ€˜Bโ€™ is three times that of โ€œA.

Choose the correct option:
(A) Statement I is correct and Statement II is incorrect
(B) Statement I and II are correct
(C) Statement II is correct and I is incorrect
(D) Statement I and II are incorrect

Answer: A

Taj Mahal is mainly threatened by the deleterious effects of:

(A) Sulphur dioxide
(B) Chlorine
(C) Oxygen
(D) Hydrogen

Answer: A

In large parts of Eastern India, the ground water is contaminated by:

(A) Arsenic
(B) Lead
(C) Mercury
(D) Nickel

Answer: A

Name the scheme which facilitates academic and research collaboration between Indian Institution and the best institution in the world.

(A) ICSSR โ€“ IMPRESS
(B) Staff Development Scheme
(C) SPARC
(D) Institutional Development Scheme

Answer: C

Efficiency of electrical power generation is least amongst the following sources of energy in case of:

(A) Solar
(B) Wind
(C) Geothermal
(D) Nuclear

Answer: A

Which of the following are features of Paris Agreement?

(i) The agreement focuses on mitigation measures for environmental pollution.
(ii) The agreement recognizes the principles of equity and common but differentiated responsibilities.
(iii) Paris agreement pertains to 2020 climate actions.

Select the correct answer using the codes given below:
(A) (1) and (ii) only
(B) (i), (ii) and (iii)
(C) (ii) and (iii) only
(D) (i) and (iii) only

Answer: C

Which of the following is considered as a secondary Air Pollutant?

(A) Nitric oxide
(B) Ozone
(C) Sulphur dioxide
(D) Carbon monoxide

Answer: B

Indian Governmentโ€™s target for share of renewable energy in the total installed capacity of electric power in India by the year 2030 is:

(A) 20%
(B) 30%
(C) 40%
(D) 50%

Answer: C

Amongst the following, which is better suited for dumping of nuclear waste?

(A) Salt mines
(B) Deserts
(C) Forests
(D) Oceans

Answer: D

The long term effect of cutting down of trees in large arcas of rain forests is:

(A) Increased rain fall in those areas
(B) Decreased rain fall in those arcas
(C) Increase in wind speeds in those areas
(D) Increased rate of soil erosion in those areas

Answer: D

Which of the following statements explains the concept of Gross Enrolment Ratio?

(A) Total number of students divided by the total population.
(B) Total number of enrolled students of a particular age group divided by total population of that age group. (C) Total number of students divided by the total number of children not admitted to the institution.
(D) Total number of students not admitted to the institution divided by the total number children of that age-group

Answer: B

Which of the following pollutants is included in the Air Quality Index in India?

(A) Carbon Dioxide
(B) Chlorofluoro carbons
(C) Sulphur dioxide
(D) Methane

Answer: C

The initiative to establish International Solar Alliance was jointly taken by:

(A) USA and India
(B) France and India
(C) Sweden and India
(D) China and India

Answer: B

Which of the following is a biological method of disposal of Municipal Solid Waste?

(A) Land filling
(B) Pulverization
(C) Composting
(D) Shredding

Answer: C

Which of the following is/are missions under the National Action Plan on Climate Change?

(1) Sustaining the Himalaya Ecosystem
(ii) Sustainable Forest Management
(iii) Strategic Knowledge for Climate Change

Select the correct answer using the codes given below:
(A) (i) and (i1) only
(B) (ii) only
(C) (ii) and (iii) only
(D) (i) and (iii) only

Answer: D

Blue- baby syndrome can happen due to excess of which component in water?

(A) Calcium
(B) Magnesium
(C) Nitrates
(D) Pesticides

Answer: C

Which pollutant is the major source of marine pollution?

(A) Sewage
(B) Oil spill
(C) Industrial waste water
(D) None of the above

Answer: A

In Audio and Video compression, the term RGB means:

(A) Red, Grey, Blue
(B) Red, Green, Blue
(C) Red, Green, Black
(D) Red, Grey, Black

Answer: B

Permafrost is defining characteristic of which biome?

(A) Taiga
(B) Tundra
(C) Grassland
(D) Desert

Answer: B

The target set by Indian government for generating power from wind energy by the year 2022 is:

(A) 60 GW
(B) 50 GW
(C) 40 GW
(D) 15 GW

Answer: A

The mangroves exist in:

(A) Fresh water system
(B) Temperate arcas
(C) Semi-arid areas
(D) Saline waters

Answer: D

Which of the following statements best explains the concept of sustainable development?

(A) Use of natural resources minimally.
(B) Use of natural resources for our benefit.
(C) Use of natural resources judiciously so that they are available for further generations.
(D) Preserve our natural resources for our future generation.

Answer: C

Chemical oxygen demand in eutrophic water is:

(A) Low
(B) High
(C) Medium
(D) Extremely low

Answer: B

In the last few years, India has been affected by which of the following tropical cyclones?

(A) Gaja, Hudhud, Bhima
(B) Hudhud, Bhima, Ockhi
(C) Ggja, Hudhud, Ockhi
(D) Gaja, Bhima, Ockhi

Answer: C

Assertion (A): High concentration of ozone in the lower troposphere is desirable.
Reason (R): Ozone present in the atmosphere protects the living organisms on the surface of earth from the harmful ultra-violet radiation of the sun.

Choose the correct answer from the options given below:
(A) Both (A) and (R) are true and (R) is the correct explanation of (A).
(B) Both (A) and (R) are true but (R) is not the correct explanation of (A).
(C) (A) is true but (R) is false
(D) (A) is false but (R) is true

Answer: D

Algal blooms in oligotrophic lakes are

(A) very frequent
(B) frequent
(C) very rare
(D) widespread

Answer: C

Assertion (A): Mcthemoglobinemia is a condition in which blood is not able to carry and deliver enough oxygen to the body.
Reason (R): Consuming drinking water with high nitrate levels may cause methemoglobinemia. Choose the correct answer from the options given below:

(A) Both (A) and (R) are true and (R) is the correct explanation of (A).
(B) Both (A) and (R) are true but (R) is not the correct explanation of (A).
(C) (A) 1s true but (R) is false
(D) (A) is false but (R) is true

Answer: A

Which of the following are priority areas in relation to the Sustainable Development Goals?

(a) No poverty
(b) Zero Hunger
(c) Reducing urbanization
(d) Peace, justice and strong institutions

Choose the correct answer from the options given below:
(A) (a), (b), (c)
(B) (a), (c), (d)
(C) (b), (c), (d)
(D) (a), (b). (d)

Answer: D

Which of the following types of energy sources employ direct energy conversion process to produce electricity?

(A) Biomass
(B) Wind
(C) Geothermal
(D) Nuclear

Answer: B

The target set by Indian government for power from roof-top solar installations by the year 2022 is

(A) 40.0 GW
(B) 50.0 GW
(ยฉ) 60.0 GW
(D) 100.0 GW

Answer: A

A geothermal reserve has the stored heat of 5ร—10โ€ณ J. If the overall efficiency of a geothermal power plant based on this reserve is 0.04, how much power can be produced by the plant in 1 year?

(A) ~032GW
(B) ~ 0.64 GW
(C) ~2.62GW
(D) ~ 63.41 GW

Answer: D

How many National Missions of the Indian Government are there to address the problem of climate change?

(A) 6
(B) 38
(C) 12
(D) 17

Answer: B

Which of the following materials is widely used in solar cell fabrication?

(A) Nubidium
(B) Silicon
(C) Nickel
(D) Chromium

Answer: B

Which one of the following conferences/summits is also known as the UN Conference on Sustainable Development (UNCSD)?

(A) The Stockholm Conference, 1972
(B) The Rio de Janeiro Conference, 1992
(C) The Johannesburg Summit, 2002
(D) The Rio+ 20 Conference, 2012

Answer: D

Which of the following belongs to the category of geophysical hazards?

(A) Infestation
(B) Avalanches
(C) Invasive species
(D) Diseases

Answer: B

In the formation of Surface Ozone, which of the following do play an important role?

(a) Oxides of nitrogen
(b) Oxides of sulphur
(c) Sunlight
(d) Carbon monoxide

Choose the correct answer from the code given below:
(A) (a), (b), (c)
(B) (b), (c), (d)
(C) (a), (c), (d)
(D) (a), (b), (d)

Answer: C

Brain drain problem which was dominant in the middle of 20th Century in India is indicative of which one of the following aspects?

(i) Lack of adequate facilities for advanced study and research in India.
(ii) The capacity of the developed nations to buy the talent at a price beyond the means of the developing nations.
(iii) Increase of population and under-utilization of human research.

Choose the correct option from below:
(A) Only (i) and (iii)
(B) Only (ii) and (iii)
(C) Only (iii)
(D) Only (i) and (ii)

Answer: D

Which are the examples of potential kinetic energy? Select your answer from the options given below:

(i) Water that is behind a dam
(ii) Radio signals
(iii) An airplane idling on the runway
(iv) A satellite before it is launched
(v) A coiled spring
(vi) Heat harnessed from the oceans

Options:
(A) (i), (iii), (iv) and (v)
(B) (i), (ii), (iii) and (vi)
(C) (ii), (iii), (iv), and (v)
(D) (ii), (iv), (v) and (vi)

Answer: A

Consider the following statements in the context of Millennium Development Goals (MDGs) adopted by the United Nations.

(a) Eradicate extreme poverty and hunger.
(b) Improve maternal health.
(c) Responsible consumption and production.
(d) Reduce child mortality.

Choose the correct answer from the options given below.
(A) (a), (b) and (c)
(B) (b), (c) and (d)
(C) (a), (b) and (d)
(D) (a), (c) and (d)

Answer: C

Assertion โ€œAโ€: Night time temperatures in the central parts of a city are generally higher than those over the surrounding rural areas.
Reason โ€œRโ„ข: Radiation losses over the urban areas are less than that over the rural areas.

Choose the correct answer from the options given below:
(A) Both โ€œAโ€ and โ€œRโ€ are true and โ€œRโ„ข is the correct explanation of โ€œAโ€.
(B) Both โ€œAโ€ and โ€œRโ€ are true but โ€œRโ€ is not the correct explanation of โ€œAโ€.
(C) โ€œAโ€ is true but โ€œRโ€ is false.
(D) โ€œAโ€ is false but โ€œRโ€ is true.

Answer: A

Which of the following disasters belongs to the category of nuclear disasters?

(a) Fukushima disaster
(b) Chernobyl disaster
(c) Three mile Island incident
(d) The love canal disaster

Choose your answer from the options given below:
(A) (a), (b) and (c)
(B) (a), (b) and (d)
(C) (a), (c) and (d)
(D) (b), (c) and (d)

Answer: A

The Paris Agreement aims to limit the temperature rise in this century by how many degrees Celsius above the preindustrial levels?

(A) 1ยฐC
(B) 2ยฐC
(C) 0.5ยฐC
(D) 3ยฐC

Answer: B

Statement I: Most of the member countries in the International solar Alliance are located between the Tropic of Cancer and Tropic of Capricorn.
Statement II: Japan is a member country of the International solar alliance. Which of the above statement is/are correct?

(A) I only
(B) II only
(C) Both I and II
(D) Neither I nor II

Answer: C

The duration for which of the usable compost generation is 3โ€”6 months?

(A) Vermi composting
(B) Semi-Automatic waste converter
(C) Pit composting
(D) Automatic waste converter

Answer: C

One of the main outcomes of the Rio +20 conference was to develop a set of:

(A) Sustainable Development Goals
(B) Millennium Development Goals
(C) National Development Strategy
(D) National Land Restoration Strategy

Answer: A

Oligotrophic lakes are characterized by:

(A) High level of nutrients:
(B) Low level of nutrients
(C) Absence of nutrients
(D) High level of organic matter

Answer: B

With increase in temperature, the dissolved oxygen content in a water body will:

(A) decreases
(B) increases
(C) remain constant
(D) may increase or decrease depending on the temperature range

Answer: A

Which of the following sequences of disposal options for low hazardous solid waste from industrial and urban sources is in order of increasing desirability?

(A) Indiscriminate dumping < Landfill < Incineration < Reuse
(B) Composting< Landfill < Reuse < Incineration
(C) Landfill< Composting < Reuse < Incineration
(D) Incineration < Composting < Reuse < Landfill

Answer: A

Identify the correct sequence of biomass fuels in terms of their energy content per unit mass

(A) Dung(dry)> Coconut Shells > Unsorted domestic refuse
(B) Unsorted domestic refuse > Coconut Shells > Dung(dry)
(C) Wood (dry) > Coconut Shells > Unsorted domestic refuse
(D) Coconut Shells > Dung(dry) > Unsorted domestic refuse

Answer: D

Suppose the concentration of Carbon dioxide, a greenhouse gas responsible for climate change, is 400 ppm. What is its concentration in air in percentage terms?

(A) 0.04%
(B) 0.4%
(C) 0.004%
(D) 4.0%

Answer: A

Identify the correct group of disease caused by polluted water

(A) Cholera, Acute Diarrhoea, Typhoid and Polio
(B) Cholera, Typhoid, Enteritis and Tuberculosis
(C) Typhoid, Enteritis and Tuberculosis
(D) Cholera, Acute Diarrhoea, Typhoid and Tuberculosis

Answer: A

Major sources of oxygen demanding waste in fresh water systems are

(a) Agricultural run-off
(b) Food processing
(c) Ore mining
(d) Sewage cffluent
(e) Landfill sites

Choose the correct option from those given below: โ€“
(A) (a), (d) and (e)
(B) (b), (d) and (e)
(C) (a), (b) and (d)
(D) (a), (b), (c) and (d)

Answer: C

Which among the following natural hazards has relatively slow onset?

(A) Volcanic eruptions
(B) Droughts
(C) Wild fires
(D) Land and slides

Answer: B

Biomass fuels have the potential to from a sustainable carbon-neutral energy source because they

(A) Produce carbon dioxide on combustion as much as they consume when they grow
(B) Produce less amount of carbon dioxide on combustion compared to the amount they use during their growth
(C) Have carbon content same as fossil fuels
(D) Do not produce hazardous emissions on combustion

Answer: A

Under Kyoto protocol, the parties, in the second commitment period, have to reduce their greenhouse gas emissions with reference to 1990 levels by atleast

(A) 10%
(B) 12%
(C) 15%
(D) 18%

Answer: D

Which of the following is a renewable natural resource?

(A) Biological diversity
(B) Tin
(C) Natural gas
(D) Salt

Answer: A

Given below are two statements โ€” one is labelled as Assertion A and the other is labelled as Reason R.

Assertion A: In the afternoon of a hot sunny day during the summer season, the tropospheric ozone levels in a city like Delhi are expected to be lower as compared to morning and evening.
Reasons R: In the afternoon period, atmosphere is unstable and its dilution potential is higher as compared to mornings and evenings.

In the light of the above stated two statements, choose the correct option from the choices given below:
(A) Both A and R are true and R is the correct explanation of A
(B) Both A and R are true and R is not the correct explanation of A
(C) Ais true, but R is false
(D) Ais false, but R is true

Answer: D

Match the following classes of chemicals with their descriptions:

(Chemical Class)
(a) Allergens
(b) Mutagens
(c) Carcinogens
(d) Teratogens
(Description)
(i) Chemicals that alter or damage the genetic material (DNA) cells
(ii) Substances that activate the immune system
(iii) Chemicals that specifically cause abnormalities during embryonics growth and development
(iv) Substances that cause invasive out of control cell growth resulting in malignant tumors

Choose your answer from the following options:
(A) (a)-(iid), (b)-(iv), (ยข)-(@), (d)-(ii)
(B) (a)-(iv), (b)-(iii), (ยข)-(ii), (d)-(1)
(ยฉ) (a)-(ii), (b)-(i), (c)-(iv), (d)-(iii)
(D) (a)-(iii), (b)-(1), (c)~(ib), (d)-(iv)

Answer: C

Spread of forest fires is influenced by

(a) Slope
(b) Soil Moisture
(c) Wind

Choose the correct answer:
(A) (a) and (b) only
(B) (b) and (c) only
(C) (a) and (c) only
(D) (a), (b) and (c)

Answer: D

Which of the following sequence represents the correct order of soil particles arranged in terms of particle size from the smallest to the largest?

(A) Gravel, Silt, Clay, Sand
(B) Silt, Clay, Gravel, Sand
(C) Clay, Silt, Sand, Gravel
(D) Silt, Clay, Sand, Gravel

Answer: C

Which of the following was not a Millennium Development Goal (MDG)?

(A) Eradicate extreme poverty and hunger
(B) Improve maternal health
(C) Ensure healthy lives and promote well being for all at all ages
(D) Ensure environmental sustainability

Answer: C

Match the following soil orders with their descriptions

(a) Histosols
(b) Andisols
(c) Gelisols
(d) Aridsols
(i) Soil order that includes soils formed on volcanic ash
(ii) Soil order of cold regions including soils underlain by permafrost
(iii) Soil order consisting of soils of dry climates
(iv) Soil order consisting of soils with a thick upper layer of organic matter

Choose the correct option from those given below:
(A) (a)-(iv), (b)-(i), (c)-(ii), (d)-(iii)
(B) (a)-(i), (b)-(iv), (c)-(i), (d)-(iii)
(C) (a)-(i), (b)-(ii), (c)-(ii), (d)-(iv)
(D) (a)-(i), (b)-(ii), (c)-(iv), (d)-(iii)

Answer: A

Match the following values with their distinguishing functions:

List I
(a) Sacrifice
(b) Sincerity
(c) Self-control
(d) Altruism
List II
(i) Working in a stipulated time
(ii) Confinement of individual mind on action
(iii) Showing love and affection
(iv) Helping without selfish motives

Choose the correct option:
(A) (a)-(b), (b)-(i), (c)-(iv), (d)-(iii)
(B) (a)-(iv), (b)-(i), (c)-(ii), (d)-(iii)
(C) (a)-(iii), (b)-(iv), (c)-(i), (d)-(ii)
(D) (a)-(i), (b)-(iii), (c)-(ii), (d)-(iv)

Answer: B

Under Green India Mission, how many hectares of degraded forest land is to be brought under afforestation?

(A) 02 million hectares
(B) 06 million hectares
(C) 08 million hectares
(D) 20 million hectares

Answer: B

The themes of some Sustainable Development Goals are

(a) Climate action
(b) Sustainable cities and communities
(c) Peace, Justice and strong institutions
(d) Skill development and decent employment
(e) Green agriculture
(f) Responsible consumption and production

Choose your answer from the following options:
(A) (a), (b), (c) (e), and (f)
(B) (b), (c), (e) and (f)
(C) (b), (c), (d), (e) and (f)
(D) (a), (b), (c) and (f)

Answer: D

The most important pollutants that cause degradation of water quality in rivers and streams are

(a) Bacteria
(b) Nutrients
(c) Metals
(d) Total dissolved solids
(e) Algae

Choose the most appropriate answer from the options given below:
(A) (a), (b) and (c)
(B) (a), (b) and (d)
(C) (a), (b), (d) and (e)
(D) a), (b), (c), (d) and (e)

Answer: A

The most dominant source of Benzene emissions in ambient air is

(A) Cement industry
(B) Cigarettes
(C) Car exhausts
(D) Plants and varnish

Answer: C

Identify from the options given below, the co-benefit of Montreal Protocol

(A) Impetus to development of energy efficient systems
(B) Reduction in carbon dioxide (equivalent)emissions
(C) Convergence of efforts of international community in addressing air pollution
(D) Control of transboundary movement of hazardous waste

Answer: B

Biosphere reserves are:

(i) area comprising terrestrial, marine and coastal ecosystems
(ii) monitored by national governments
(iii) areas where threatened animals and plants are kept in their habitat
(iv) wildlife sanctuaries

Choose the correct statements from the options given below:
(A) (i) and (ii)
(B) (ii) and (iii)
(C) (iii) and (iv)
(D) (i) and (iv)

Answer: A

A green building

(i) uses minimum amount of energy
(ii) generate waste
(iii) consumes a lot water
(iv) conserves natural resources
(v) creates space for healthy living

Choose the correct statements {rom the options given below:
(A) (i), (ii) and (iii)
(B) (1), (iv) and (v)
(C) (ii), (iii) and (v)
(D) (ii), (iv) and (v)

Answer: B

Identify values specially related to organizational ethos from the list given below:

(i) Team work in the approach
(ii) Productivity of the tasks
(iii) Changes in the global contents
(iv) Financial security
(v) Humility in dealing with othersโ€™ concerns
(vi) Technological resources

Choose your answer from the options given below:
(A) (i), (iii) and (iv)
(B) (iii), (iv) and (v)
(C) (ii), (iii) and (iv)
(D) (i), (ii) and (v)

Answer: B

Methane, a greenhouse gas, is emitted from

(a) Landfills
(b) construction debris
(c) wet lands
(d) e-waste

Choose the correct statements from the options given below:
(A) (a), (b) and (d)
(B) (a), (c) and (d)
(C) (a) and (c) only
(D) (a) and (d) only

Answer: A

The share of which of the following sources of energy in electricity generation in India at presents the least?

(A) Thermal
(B) Solar and Wind
(C) Hydro
(D) Nuclear

Answer: B

The government of India has set up the Ministry of Jal Shakti by merging/renaming which of the following:

(A) Ministry of Water Resources and Ministry of Earth sciences
(B) Ministry of Water Resources; Ministry of River development and Ganga Rejuvenation; and Ministry of Rural Development
(C) Ministry of Water Resources; Ministry of River development and Ganga Rejuvenation; and Ministry of Drinking Water and Sanitation
(D) Ministry of Water Resources

Answer: C

Which of the following UN Conferences/Summit adopted the programme of Action for Sustainable Development?

(A) Stockholm Conference
(B) Rio De Janeiro Conference
(C) Johannesburg Summit
(D) Ahmedabad Conference

Answer: B

For encouraging worldwide awareness and action for the protection of our environment, UNEP focused on World Environment Day (2019) on which of the following themes?

Options:-
(A) Conserve energy
(B) Tackle climate change
(C) Beat air pollution
(D) Save water

Answer: C

Identify environmental issues which are local in nature:

(i) Depletion of ozone layer
(ii) Lake pollution
(iii) Soil erosion
(iv) Climate change
(v) water logging
(vi) Solid Waste Management

Select the answer from the options given below:
(A) (ii), (iii), (v) and (vi)
(B) (iii), (iv), (v) and (vi)
(C) (ib), (iii), (iv) and (v)
(D) (i), (ii), (iii) and (iv)

Answer: C

Which of the following indicates the objective of the United Nations Framework Convention on Climate change (UNFCC)?

Options:-
(A) To stabilize greenhouse gases concentration in the atmosphere
(B) To prescribe limits on greenhouse gas emissions for individual countries
(C) To lay down enforcement mechanisms
(D) To prepare guidelines for formulation of Climate Action Plan by member countries

Answer: A

Climate change can have serious consequences for:

a. Water security
b. Food security
c. Erosion of coastal zones
d. Soil fertility
e. Respiratory health

Choose the correct answer from the options given below:
(A) Only a, band c
(B) Only a,b, d and e
(C) Only a, b,c and e
(D) a, b, c, d and e

Answer: A

Photochemical smog in urban areas is known to cause

a) Respiratory effects
b) Eye irritation
c) Nose and throat irritation
d) Carcinogenic effect
e) Reduction in visibility

Choose the correct answer from the options given below
(A) Only a, b and d
(B) Only a, b, c and e
(C) Only b, d and e
(D) a, b, c, d and e

Answer: B

Landslides are of Options:

(A) Chemical origin
(B) Hydrological origin
(C) Hydro-biological origin
(D) Geo-physical origin

Answer: D

The contribution of renewable energy sources including large hydropower projects in the total installed electric power capacity of India at present (June 2019) is approximately

Options:
(A) ~35%
(B) ~23%
(C) ~19%
(D) ~41%

Answer: A

A measure such as building a plinth wall for floods would be termed as Options:

(A) Preparedness
(B) Prevention
(C) Mitigation
(D) Adaptation

Answer: B

At present, in India, the total installed renewable power capacity is accounted for by:

(a) Solar Power
(b) Wind Power
(c) Hydropower
(d) Urban and industrial waste to energy conversion
(e) Biomass power
(f) Geothermal power

Choose the correct answer from the options given below
(A) Only (a), (b), (c), (e) and (f)
(B) Only (a), (b), (c), (d) and (f)
(C) Only (a), (b), (c), (d) and (e)
(D) (a), (b), (c), (d), (e) and (f)

Answer: C

The most relevant effect of particulate matter in air on human health is: Options:

(A) Impaired blood formation
(B) Chest tightness
(C) Aggravation of respiratory disease
(D) Headache and rise in blood pressure level

Answer: C

Given below are two statements one is labelled as Assertion (A) and the other is labelled as Reason R):

Assertion, A: It is crucial to sustain Himalayan ecosystem
Reason, R: Himalayan ecosystem is home to a number of indigenous tribes. In the light of the above two statements

choose the correct answer from the options given below
(A) Both A and R are true and R is the correct explanation of A.
(B) Both A and R are true but R is NOT the correct explanation of A.
(C) Ais true but R is false
(D) A is false but R is true

Answer: B

In developing countries, the fraction of food waste in the composition of municipal solid waste is typically in the range Options:

(A) 10-15%
(B) 20-30%
(C) 25-35%
(D) Greater than 40%

Answer: D

Sustainable Development Goal โ€” โ€˜Life below waterโ€ aims at conserving and sustainably using: Option:

(A) Ground water resources
(B) Rivers, streams, ponds and lake resources
(C) Oceans, sea and marine resources
(D) Wetland resources

Answer: C

From the following list, identify the Greenhouse Gases adopted in the Kyoto Protocol.

(i) Carbon dioxide
(ii) Methane
(iii) Nitrous oxide
(iv) Hydrofluorocarbons
(v) Perfluorocarbons
(vi) Sulphur hexafluoride

Select the most appropriate answer from the options given below: Codes
(A) (i), (ii), (iv), (v) only
(B) (i), (iv), (v) and (vi) only
(C) (ii), (iv), (v) and (vi) only
(D) (i), (ii), (iii), (iv), (v) and (vi)

Answer: D

Climate pledges under Paris Agreement cover what fraction of greenhouse gas emissions reduction needed to limit the global warming below 2ยฐC? Option:

(A) 1/3
(B) 2/3
(C) 1/2
(D) 3/4

Answer: A

The impact of noise pollution on human health is governed by:

(i) Intensity of noise
(ii) Duration of noise
(iii) Socio-economic status of an individual
(iv) Sensitivity of human ear
(v) Frequency range of noise

Choose your answer from the options given below: Codes
(A) (i), (ii), (iv) and (v) only
(B) (i), (ii) and (iii) only
(C) (i), (iii), (iv) and (v) only
(D) (i), (ii), (iii), (iv) and (v)

Answer: A

Polar Stratospheric Clouds are associated with which of the following environmental issues? Option:

(A) Flash floods
(B) Acid rain
(C) Ozone layer depletion
(D) Photo-chemical smog

Answer: C

A vital role in human resource development of a country centered on skilled manpower, productivity and the quality of life of its people is focused on which of the following domains? Option:

(A) Professional education
(B) Formal education
(C) Lifelong education
(D) Technical education

Answer: D

From the following list. identify the sources of solid waste:

i) Residential areas
ii) Oceans
iil) Institutional areas
iv) Agriculture
v) Dust storms

Select the correct answer from the options given below:
(A) i, ii and iii
(B) ii, iii and iv
(C) iii, ivand v
(D) i, iii and iv

Answer: D

Identify the sources of soil pollution from the following list:

i) Industrial effluents
ii) Eutrophication
iii) Unscientific disposal of nuclear waste
iv) Off-shore oil drilling
v) Improper management of septic systems

Choose the correct answer from the options given below:
(A) i, ii and iii
(B) ii, iii and iv
(C) i, iii and v
(D) ii, iv and v

Answer: C

Which of the following statements represents mangrove characteristics as a safety hedge? Option:

(A) The mangrove trees are tall with dense canopies
(B) The mangrove trees provide both food and medicines
(C) The mangrove swamps separate the human settlement from the sea
(D) The mangrove trees protect vulnerable coastlines as they hold soil together

Answer: D

Which of the following statements about the Volatile Organic Compounds (VOCs) are correct?

i) They cause acid rain
ii) They may cause cancer
iii) They play an important role in the formation of photochemical smog

Select the correct answer from the options given below:
(A) iand ii
(B) ii and iii
(C)1i and iii
(D) i. ii, and iii

Answer: B

Which of the following was the central aim of the Paris Agreement? Option:

(A) To reduce the CFCs emissions
(B) To strengthen the global response to the threat of climate change
(C) To address biological diversity issues
(D) To address the problem of ozone layer depletion

Answer: B

Given below are two Statements, one is labelled as Assertion A and other is labelled as Reason R.

Assertion A: Carbon dioxide emissions from bioenergy production have traditionally been excluded from most emission inventories and climate impact studies.
Reason R: Carbon dioxide emissions associated with production of bioenergy are significantly less compared to those from combustion of fossil fuels.

In the light of the above statements, choose the correct answer from the options given below:
(A) Both A and R are correct and R is the correct explanation of A
(B) Both A and R are correct but R is NOT the correct explanation of A
(C) A is correct but R is not correct
(D) A is not correct but R is correct

Answer: B

Wind energy is very sensitive to the wind velocity because wind power is directly proportional to the: Option:

(A) wind velocity
(B) square of the wind velocity
(C) square root of the wind velocity
(D) cube of the wind velocity

Answer: D

During physical treatment process of waste water, what is the correct order of following operations to be followed?

(i) Flocculation
(ii) Filtration
(iii) Screening
(iv) Sedimentation

Choose the correct answer from the options given below:
(A) (iii), (iv), (ii), (i)
(B) (iii), (iv), (i), (ii)
(C) (ii), (iv), (i), (iii)
(D) (ii), (iii), (iv), (i)

Answer: B

Given below are two Statements, one is labelled as Assertion A and other is labelled as Reason R.

Assertion A: Carbon monoxide (CO) is a serious asphyxiate; even a short exposure may have fatal health issues.
Reason R: Hemoglobin present in the blood has greater affinity towards carbon.

In the light of the above statements, choose the correct answer from the options given below:
(A) Both A and R are correct and R is the correct explanation of A
(B) Both A and R are correct but R is NOT the correct explanation of A
(C) A is correct but R is not correct
(D) A is not correct but R is correct

Answer: A

Which of the following explain the concept of values?

(i) Values are beliefs tied inextricably to human emotion and behaviour
(ii) Values refer to desirable goals people strive to attain
(iii) Values are conflict-free
(iv) Values are ordered by importance relative to one another
(v) Values are utilized primarily to effect change in knowledge of a person

Choose the correct answer from the options given below:
(A) (i), (ii), (iii) only
(B) (i), (ii), (iv) only
(C) (ii), (iii), (iv) only
(D) (iii), (iv), (v) only

Answer: B

A multilateral fund for implementation of Montreal Protocol was set up to provide financial and technical assistance to developing countries, parties to Montreal protocol, whose annual per capita consumption and production of ozone depleting substances amounted to less than: Option:

(A) 0.3 kg
(B) 0.5kg
(C) 1.0kg
(D) 1.5 kg

Answer: A

Arrange the Global Warming Potential of a molecule of the following greenhouse gases (relative to CO) in decreasing order:

(i) Methane
(ii) Nitrous Oxide
(iii) CFC โ€“ 12
(iv) CFC -11
(v) Sulphur hexafluoride (SF)

Choose the correct answer from the options given below: Codes
(A) (v) > (iv) > (iii) > (ii) > (i)
(B) (iii) > (v) > (iv) > (ii) > (i)
(C) (iv) > (v) > (iii) > (ii) > (i)
(D) (v) > (iii) > (iv) > (ii) > (i)

Answer: D

Sick Building Syndrome (SBS) is a condition: Option:

(A) when buildings are very old and about to collapse
(B) of human health due to indoor air pollution
(C) of fear due to deserted building
(D) of building devoid of any modern amenities

Answer: B

Given below are two statements:

Statement I: Dioxins and furans are highly toxic substances produced mainly during incineration of municipal solid waste. hazardous and medical wastes.
Statement II: Dioxins and furans play crucial role in photochemical smog formation.

In the light of the above statements, choose the correct answer from the options given below:
(A) Both Statement I and Statement II are true
(B) Both Statement I and Statement II are false
(C) Statement I is correct but Statement II is false
(D) Statement I is incorrect but Statement II is true

Answer: C

A dish solar cooker:

(i) has reflecting material of anodized aluminum sheet.
(ii) has a reflectivity of less than 50%.
(iii) needs to track the sun to deliver power of about 0.6 kW.
(iv) uses a parabolic dish to concentrate the incident solar radiation.
(v) can have the bottom vessel temperature over 1000ยฐC.

Choose the correct answer from the options given below: Codes
(A) (i), (iii), (iv) and (v) only
(B) (i), (iii) and (iv) only
(C) (i), (ii), (iii) and (v) only
(D) (i), (iii) and (v) only

Answer: B

Exposure to excessive noise pollution can cause:

(i) Hearing impairment
(ii) Insomnia
(iii) Rise in blood pressure
(iv) Respiratory disease
(v) Reduced work efficiency

Choose the most appropriate answer from the options given below: Codes
(A) (i), (iii), (v) only
(B) (i), (iii), (iv) only
(C) (ii), (iii) and (v) only
(D) (i), (ii), (iii) and (v) only

Answer: D

In Photo Voltaic system (PV-system) arrange the following from smallest to the largest unit.

(i) Module
(ii) Array
(iii) Solar cell
(iv) Array field

Choose the correct answer from the options given below: Codes
(A) (ii), (iv), (i), (iii)
(B) (ii), (iv), (iii), (i)
(C) (iii), (ii), (iv), (i)
(D) (iii), (i), (ii), (iv)

Answer: D

Which of the following types of particulate matter pollutants are used in calculation of Air Quality Index (AQI)?

(i) TSP (Total Suspended Particles)
(ii) PM 10 (Particulate matters of size 10 micron or less)
(iii) PM 2.5 (Particulate matter of size 2.5 micron or less)
(iv) PM 1 (Particulate matter of size 1 micron or less)

Choose the correct answer from the options given below:
(A) (ii), (iii) and (iv) only
(B) (i) and (ii) only
(C) (ii) and (iii) only
(D) (i), (ii) and (iii) only

Answer: C

Match the following.

List-I (International Agreements)
(a) Montreal Protocol
(b) Kyoto Protocol
(c) Convention on Biodiversity
(d) International Solar Alliance
List-II (Provisions)
(i) Clean affordable energy
(ii) Controlling consumption and production of Halons
(iii) Emission trading
(iv) Clearing House Mechanism

Codes (a) (b) (c) (d)
(A) (iv), (ii), (iii), (i)
(B) (ii), (i), (iii), (iv)
(C) (i), (i), (iv), (iii)
(D) (i), (iii), (iv), (i)

Answer: D

Match the following.

List-I (Toxic and Hazardous substances)
(a) Vinyl Chloride
(b) Polychlorinated biphenyls (PCBs)
(c) Benzo(a)pyrene
(d) Polycyclic Aromatic Hydrocarbons
List-II (Major sources)
(i) Electric Insulations
(ii) Fuel combustion
(iii) Plastic industrial uses
(iv) Waste Incineration

Codes (a) (b) (c) (d)
(A) (i), (iii), (iv), (ii)
(B) (i), (iii), (ii), (iv)
(C) (iii), (iv), (ii), (i)
(D) (iii), (i), (iv), (ii)

Answer: D

Helping individuals and social groups acquire social values, contributes to development of: Option:

(A) Environmental awareness
(B) Environmental knowledge
(C) Environmental attitude
(D) Environmental skills

Answer: C

Match the following.

List-I (Types of Appraisal)
(a) Developmental
(b) Managerial
(c) Laissez-faire
(d) Judgmental
List-II (Characteristics)
(i) Concerned with doing and achieving.
(ii) Concerned with moral, ethical and professional values.
(iii) Concerned with maintenance of social control.
(iv) Concerned with the importance of self-development.

Codes (a) (b) (c) (d)
(A) (iii), (iv), (ii), (i)
(B) (ii), (i), (iv), (iii)
(C) (i), (iii), (ii), (iv)
(D) (iv), (ii), (iii), (i)

Answer: B

MOODLE is abbreviation of: Option:

(A) Modular Object-Oriented Distance Learning Environment
(B) Modular Object-Oriented Dynamic Learning Environment
(C) Modular Object-Oriented Distance Legislative Environment
(D) Modular Object-Oriented Distance Legal Environment

Answer: B

The only UN Treaty related to environmental issues which has been ratified by all 197 UN member states is: Option:

(A) Montreal Protocol
(B) Paris Agreement
(C) Kyoto Protocol
(D) Basel Convention

Answer: A

As of now, which of the following is a leading country in wind energy based electricity generation? Option:

(A) India
(B) China
(C) France
(D) Denmark

Answer: B

โ€˜Solar Water Stillโ€™ is a device to: Option:

(A) Produce potable water by using solar energy
(B) Pump water by using solar energy
(C) Generate electricity by using solar energy
(D) Produce heat using solar energy

Answer: A

Identify the primary air pollutant among the following: Option:

(A) Ozone
(B) Black carbon aerosols
(C) Sulphate aerosols
(D) Peroxy Acetyl Nitrate (PAN)

Answer: B

Given below are 2 Statements:

Statement I: Long stretches of major rivers of India have very low Biochemical Oxygen Demand (BOD) values. Statement II: In rivers, oxygen is added into water by diffusion from air due to its turbulent and rapid flow.

In the light of the above statements, choose the most appropriate answer from the options given below: Codes
(A) Both Statement I and Statement II are correct
(B) Both Statement I and Statement II are incorrect
(C) Statement 1 is correct but Statement II is incorrect
(D) Statement I is incorrect but Statement II is correct

Answer: D

Paravidya, according to the vedic tradition. implies: Option:

(A) Knowledge based on human experience
(B) Knowledge that transcends human experience
(C) Knowledge acquired from teachers
(D) Knowledge acquired from texts

Answer: B

The underlying purpose of environmental education is to:

(i) Adjust to environmental challenges
(ii) Help face environmental hazards
(iii) Promote individualโ€™s critical-thinking about emerging issues
(iv) Increase public awareness and knowledge of environmental issue.
(v) Enhance problem-solving and decision-making skills in respect of handling environmental issues.

Choose the correct answer from the options given below: Codes
(A) (i), (ii), (iii) only
(B) (ii), (iii), (iv) only
(C) (iii), (iv), (v) only
(D) (iv), (v), (i) only

Answer: C

Which of the following countries has not signed/ratified the International Solar Alliance Framework Agreement? Option:

(A) Australia
(B) United Kingdom
(C) Japan
(D) China

Answer: D

Given below are two statements: One is labelled as Assertion A and the other is labelled as Reason R:

Assertion A: Energy production from biomass has traditionally been considered carbon neutral
Reason R: Carbon dioxide emissions from combustion of biomass are sequestered by growing biomass.

In the light of the above statements, choose the most appropriate answer from the options given below:
(A) Both A and R are correct and R is the correct explanation of A
(B) Both A and R are correct but R is NOT the correct explanation of A
(C) A is correct but R is not correct
(D) A is not correct but R is correct

Answer: A

For wind power electricity generation which among the following type of rotors is most commonly used? Option:

(A) Multi-blade rotor
(B) Propeller rotor
(C) Savonius rotor
(D) Darrieus rotor

Answer: B

Given below are two statements:

Statement I: Chronic toxic effects result from single dose of very high toxic substance or continuous exposure of sub lethal dose
Statement II: Acute toxic effects are long lasting and irreversible mainly caused due to prolonged exposure to toxins and survival rate is very low.

In the light of the above statements, choose the most appropriate answer from the options given below:
(A) Both Statement I and Statement II are correct
(B) Both Statement I and Statement II are incorrect
(C) Statement I is correct but Statement II is incorrect
(D) Statement I is incorrect but Statement II is correct

Answer: C

Which of the following goals are related to millennium Development Goals?

I. Improving maternal health
II. Controlling depletion of ozone layer
III. Improving the life of slum dwellers
IV. Creating smart city infrastructure
V. Promoting gender equality and empowerment of women

Choose the correct answer from the options given below:
(A) I, III, V only
(B) II, III, IV, V only
(C) I, II, III, V only
(D) I, II, IV, V, only

Answer: A

Which of the following conventions protocol originated from Rio Earth Summit?

A. UN Framework convention on climate change
B. Ramsar convention on wetlands
C. UN convention to combat desertification
D. Kyoto Protocol
E. Convention on Biological Diversity

Choose the most appropriate answer from the options given below:
(A) A, B and E only
(B) A, C, D and E only
(C) A, Cand E only
(D)B, C,D and E only

Answer: C

Which of the following goals are part of Sustainable Development Goals?

A. Industry, Innovation and Infrastructure
B. Universal higher education
C. Reduced Inequality
D. Partnerships to achieve goals
E. Clean air

Choose the correct answer from the options given below:
(A) A, B,C and D only
(B) A, C and D only
(C) A, B, Cand E only
(D) B, C, D and E only

Answer: B

Arrange the following countries in increasing order of their installed capacities of wind energy (as of 2018):

A. China
B. India
C. USA
D. France
E. Germany

Choose the correct answer from the options given below:
(A) D<E<C<B<A
(B) D<E<B<C<A
(C) D<B<E<C<A
(D) E<D<B<C<A

Answer: C

Given below are two statements:

Statement I: Sunlight is a pre-requisite for the formation of photochemical smog in urban areas.
Statement II: Photochemical smog is a recurring phenomenon in winter and summer seasons in urban areas of India.

In the light of the above statements, choose the most appropriate answer from the options given below:
(A) Both Statement I and Statement II are true
(B) Both Statement I and Statement II are false
(C) Statement I is correct but Statement II is false
(D) Statement I is incorrect but Statement II is true

Answer: C

Given below are two statements: One is labelled as Assertion A and the other is labelled as Reason R:

Assertion A: Global warming potential of a molecule of a greenhouse gas over different time spans of decades to 100 years may vary significantly.
Reason R: Some greenhouse gases have shorter life times compared to carbon dioxide.

In the light of the above statements, choose the most appropriate answer from the options given below:
(A) Both A and R are correct and R is the correct explanation of A
(B) Both A and R are correct but R is NOT the correct explanation of A
(C) A is correct but R is not correct
(D) A is not correct but R is correct

Answer: A

Given below are some metals which are released into environment from a variety of anthropogenic sources. Identify the metals which have been of serious concern from the point of view of pollution of water bodies

I. Aluminium
II. Copper
III. Mercury
IV. Cadmium
V. Lead

Choose the correct answer from the options given below:
(A) II, III, V only
(B) I, II, III, V only
(C) III, IV, V only
(D) I, II, V only

Answer: C

Given below are two statements:

Statement I: Two sounds of same intensity but different frequency characteristics appear to be of different loudness
Statement II: The response of human ear to noise of different frequencies is not uniform

In the light of the above statements, choose the most appropriate answer from the options given below:
(A) Both Statement I and Statement II are correct
(B) Both Statement I and Statement II are incorrect
(C) Statement I is correct but Statement II is incorrect
(D) Statement I is incorrect but Statement II is correct

Answer: A

Given below are two statements: One is labelled as Assertion A and the other is labelled as Reason R.

Assertion A: Long term low emission development strategies by each country are crucial to realizing the goals of Paris Agreement
Reason R: Nationally Determined Contributions (NDCs) declared by each country are not sufficient to keep the rise in global temperature up to 2ยฐC above the pre-industrial level.

In the light of the above statements, choose the most appropriate answer from the options given below:
(A) Both A and R are correct and R is the correct explanation of A
(B) Both A and R are correct but R is NOT the correct explanation of A
(C) A is correct but R is not correct
(D) A is not correct but R is correct

Answer: A

Identify the correct sequence (decreasing order) of lifetimes of the following greenhouse gases in atmosphere:

I. Methane
II. Nitrous oxide
III. Surface ozone
IV. CFC-11

Choose the correct answer from the options given below:
(A) IV>II>I>III
(B) II>IV>I>III
(C) II>IV>III>I
(D) IV>II>III>I

Answer: B

Given below are two statements:

Statement I: Hurricanes and typhoons are essentially the same as tropical cyclones
Statement II: Warm tropical oceans and moist air mass are pre-requisites for the development of tropical cyclones

In the light of the above statements, choose the most appropriate answer from the options given below:
(A) Both Statement I and Statement II are correct
(B) Both Statement I and Statement II are incorrect
(C) Statement [ is correct but Statement II is incorrect
(D) Statement I is incorrect but Statement II is correct

Answer: A

Contamination of aquatic environment by Polychlorinated Biphenyls (PCBs) can originate from

I. Sewage effluent
II. Waste incinerators
III. Toxic dumps
IV. Thermal power plants
V. Transport sector

Choose the correct answer from the options given below:
(A) I, III, V only
(B) I, II, IV, V only
(C) II, III, IV only
(D) I, II, III only

Answer: D

Formulation of long-term emission development strategies by each country was suggested under Option:

(A) Paris Agreement
(B) Montreal Protocol
(C) International Solar Alliance
(D) Kyoto Protocol

Answer: A

Which of the following sources of energy on combustion produces maximum carbon dioxide per unit of energy output or heat content? Option:

(A) Coal (sub-bituminous)
(B) Natural gas
(C) Diesel fuel and heating oil
(D) Gasoline (without ethanol)

Answer: A

Which of the following are objectives of a performance appraisal system?

I. To help an employee in finding out his/her own strengths and weaknesses
II. To help in creating desirable organizational ideas
III. To help in identifying people for the purpose of motivating and training them for new roles
IV. To help in distinguishing the non-performers and performers
V. To enable learners to understand the weakness in the system

Choose the correct answer from the options given below:
(A) I, II and III only
(B) II, III and IV only
(C) III, IV and V only
(D) V, IV and I only

Answer: A

Given below are two statements:

Statement I: Emissions of hydrocarbons and carbon monoxide are inherently very low from diesel vehicles. Statement II: Diesel driven vehicles are inherently more fuel efficient compared to petrol driven vehicles.

In the light of the above statements, choose the most appropriate answer from the options given below:
(A) Both Statement I and Statement II are correct
(B) Both Statement I and Statement II are incorrect
(C) Statement I is correct but Statement II is incorrect
(D) Statement I is incorrect but Statement II is correct

Answer: A

India has huge Thorium reserve, a potential source of nuclear energy. This thorium reserve is mainly confined in the:

(A) rocks of Chhota Nagpur plateau
(B) muds of Sunderban delta
(C) coastal sands of Kerala sea
(D) sands of Thar desert

Answer: C

Identify the objectives specific to sustainable Development Goals among the following:

(i) Responsible consumption and production
(ii) Life on land
(iii) Improving maternal health
(iv) Ensuring environmental sustainability
(v) Reduced inequality

Choose the correct answer from the options given below:
(A) (i), (ii) and (v)
(B) (i), (iii), (iv) and (v)
(C) (ii), (iii), (iv) and (v)
(D) (i), (ii), (iii), (iv) and (v)

Answer: A

Given below are 2 Statements, one is labelled as Assertion A and other is labelled as Reason R.

Assertion A: Emissions from diesel vehicles are relatively less harmful to human health in comparison to petrol vehicles.
Reason R: Emissions of hydrocarbons and carbon monoxide are inherently very low from diesel vehicles.

In the light of the above statements, choose the most appropriate answer from the options given below:
(A) Both A and R are correct and R is the correct explanation of A
(B) Both A and R are correct but R is NOT the correct explanation of A
(C) A is correct but R is not correct
(D) A is not correct but R is correct

Answer: D

Given below are 2 Statements, one is labelled as Assertion A and other is labelled as Reason R.

Assertion A: Solar ponds store solar energy in the form of heat.
Reason R: Solar ponds contain water with definite gradient of salt concentration.

In the light of the above statements, choose the most appropriate answer from the options given below: Codes
(A) Both A and R are correct and R is the correct explanation of A
(B) Both A and R are correct but R is NOT the correct explanation of A
(C) A is correct but R is not correct
(D) A is not correct but R is correct

Answer: A

Which of the following sectors contributed least to the global carbon dioxide emissions in the year 2014 as per IPCC report?

(A) Transportation
(B) Agriculture and forest land use
(C) Industry
(D) Utility (Electricity and Heat Production)

Answer: A

To put the world on a least cost pathway to limiting global warming to 2ยฐC above the pre-industrial level, global greenhouse gas emission by 2030 with reference to 2017, require reduction to the extent of Option:

(A) 25%
(B) 15%
(C) 10%
(D) 20%

Answer: A

Identify the correct sequence in decreasing order of the carbon per unit mass of the following carbon reservoirs in the aquatic environment of oceans

a. Sediments
b. Living organisms
c. Dissolved organic matter
d. Dissolved inorganic matter

Choose the correct answer from the options given below
(A)d,a, b,c
(B)a,d,c,b
(C)b, c,a,d
(D)c,b,a,d

Answer: B

Given below are two statements, one is labelled as Assertion A and the other is labelled as Reason R

Assertion A: Hydrofluorocarbons (HFCs) need to be phased out to protect the Ozone layer
Reason R: HFCโ€™s have a high global warming potential

In light of the above statements, choose the most appropriate answer from the options given below
(A) Both A and R are correct and R is the correct explanation of A
(B) Both A and R are correct but R is NOT the correct explanation of A
(C) A 1s correct but B is not correct
(D) A is not correct but R is correct

Answer: D

Mitigation measure to cope with floods include

a. Elevating or protecting electrical service panels
b. Upsizing culverts to better handle flood surges
c. Protecting facilities with barriers and sandbags
d. Removing trees and debris from the flood plains
e Relocating equipment outside the flood plain

Choose the correct answer from the options given below:
(A) a, c,d and e only
(B) a, b, c and e only
(C) a,b,c,d and e
(D) b, c, d and e only

Answer: B

Identify the correct sequence of the following States Union of India in terms of increasing wind energy potential

a. Gujarat
b. Rajasthan
c. Madhya Pradesh
d. Tamil Nadu

Choose the correct answer from the options given below
(A)d,c,b,a
(B)b,d,c,a
(C)c,b,d,a
(D)b,c,d, a

Answer: C

Dissolved Oxygen (DO), an important parameter of water quality, is essential for survival of

(A) Humans
(B) Animals
(C) Fish
(D) Crops and vegetables

Answer: C

Among the following elements which is typically the most abundant in dried sewage sludge?

(A) Total nitrogen
(B) Total sulphur
(C) Calcium
(D) Total phosphorus

Answer: C

Identify the correct sequence of sectoral global CO, emissions in increasing order as per IPCC (2014) report:

a. Electricity and heat production
b. Buildings
c. Transportation
d. Industry

Choose the correct answer from the options given below:
(A)b, c,d, a
(B)b,d, a, d
(C)b,d,c,a
(D)b,d, a,c

Answer: A

Under Goal 2 of Millennium Development Goals, UN member countries were to ensure that by 2015, children everywhere, boys and girls would be able to complete a full course of:

(A) Primary education
(B) Secondary education
(C) Tertiary education
(D) Skill based education

Answer: A

Identify the correct sequence in increasing order of the carbon content per unit mass of the following reservoirs of carbon in terrestrial environment.

(i) Plants
(ii) Peat
(iii) Soil organic matter
(iv) Fossil fuels

Choose the correct answer from the options given below:
(A) (ii), (i), (iii), (iv)
(B) (i), (iii) (ii), (iv)
(C) (i), (iii), (iv),
(D) (ii), (i), (iv), (iii)

Answer: A

Given below are two statements, one is labelled as Assertion A and other is labelled as Reason R.

Assertion A: Lakes are less prone to getting polluted than rivers.
Reason R: Water in lakes gets replaced through natural processes.

In light of the above statements, choose the most appropriate answer from the options given below: Codes (A) Both A and R are correct and R is the correct explanation of A
(B) Both A and R are correct but R is NOT the correct explanation of A
(C) A is correct but R is not correct
(D) A is not correct but R is correct

Answer: D

Under Millennium Development Goal 5, maternal mortality ratio between 1990 and 2015 was to be reduced by: Option:

(A) 50 %
(B) 75%
(C) 66 %
(D) 40 %

Answer: B

According to the classification of Ministry of New and Renewable Energy and Central Electricity Authority of India, small hydro power plants have capacity in the range: Option:

(A) 1-25 MW
(B) 100-1000 kW
(C) 1-50 MW
(D) 1-100 kW

Answer: A

Given below are two statements, one is labelled as Assertion A and other is labelled as Reason R.

Assertion A: Global emissions of carbon dioxide stagnated during the years 2014-2016.
Reason R: Strong energy cfficiency improvement measures and low carbon technologies were deployed during 2014-2016.

In light of the above statements, choose the most appropriate answer from the options given below: Codes (A) Both A and R are correct and R is the correct explanation of A
(B) Both A and R are correct but R is NOT the correct explanation of A
(C) A is correct but R is not correct
(D) A is not correct but R is correct

Answer: A

Under which of the following objectives of Millennium Development Goals, the progress in the world as a whole has been least satisfactory? Option:

(A) Halting by 2015 and beginning to reverse the spread of HIV/AIDS
(B) Eliminating gender disparity in primary and secondary education by 2015
(C) Reducing the proportion of population below national poverty line by 2015
(D) Reducing by half proportion of people who suffer from hunger by 2015

Answer: D

Identify the correct sequence of Carbon โ€œCโ€™ to Nitrogen โ€œNโ€™ ratio in decreasing order in organic waste material. Option:

(i) Food waste
(ii) Leaves and foliage
(iii) Paper
(iv) Cow manure

Choose the correct answer from the options given below: Codes
(A) (iii), (ii), (iv), (i)
(B) (iii), (i), (iv), (ii)
(C) (ii), (iii), (iv), (i)
(D) (ii), (iv), (iii), (i)

Answer: A

According to Noise Pollution (Regulation and Control) Rules, 2000, the night time Noise standard prescribed for Educational Institutions is: Option:

(A)55dB A
(B)50dB A
(C)45dB A
(D)40dB A

Answer: D

Given below are two statements

Statement I: Volatile Organic Chemicals (VOCs) are contaminants more commonly found in groundwater than in surface water.
Statement II: VOCs are one of the criterion parameters to determine drinking water quality.

In light of the above statements, Choose the correct answer from the options given below: Codes
(A) Both Statement I and Statement II are true
(B) Both Statement I and Statement II are false
(C) Statement I is true but Statement II is false
(D) Statement I is false but Statement II is true

Answer: C

Which of the following is NOT one of the Missions under National Action Plan on Climate Change? Option:

(A) National Water Mission
(B) National Mission on Energy Efficiency Improvement
(C) Green India Mission
(D) National Mission on Solid Waste Management

Answer: D

Identify the correct sequence of countries contributing to global CO, emission in decreasing order from fossil fuel burning, cement manufacturing and gas flaring in the year 2014 Option:

A. USA
B. EU-28
C. China
D. Russia

Choose the correct answer from the options given below
(A)C,A,B,D
(B)A,C,B,D
(C)A,B,C,D
(D)A,C,D,B

Answer: A

Which among the following are used as semiconductor materials to fabricate solar cells

A. Silicon (Si)
B. Tin oxide (SnO)
C. Cadmium Sulphide (CdS)
D. Gallium Arsenide (GaAs)

Choose the correct answer from the options given below:
(A) A and C only
(B) A and B only
(C) B, C and D only
(D) A, B, C and D

Answer: D

Which of the following is not a Sustainable Development Goal? Option:

(A) Gender equity
(B) Climate action
(C) Protection of Ozone layer
(D) Life below water

Answer: C

Which among the following are emitted from a coal based thermal power plant?

A. Carbon monoxide
B. Ozone
C. Sulphur dioxide
D. Methane
E. Particulate matter

Choose the tarred answer from the options given below:
(A) A, B, Cand E only
(B) A, C and E only
(C) A, B and C only
(D) A, C,D and E only

Answer: B

Given below are two statements, one is labelled as Assertion A and the other is labelled as Reason R

Assertion A: Lakes are considered to be more susceptible to pollution than rivers.
Reason R: Water in lakes may take decade(s) to be replaced

In light of the above statement, choose the most appropriate answer from the options given below
(A) Both A and R are correct and R is the correct explanation of A
(B) Both A and R are correct but R is NOT the correct explanation of A
(C) Ais correct but R is not correct
(D) A is not correct but R is correct

Answer: A

Thermal reactors produce energy by fission of the following nuclear fuels

A. Uranium-235 (239U)
B. Uranium-233 (233U)
C. Uranium-238 (238U)
D. Thorium-232 (232Th)
E. Plutonium-239 (238Pu)

Choose the correct answer from the options given below:
(A) A, B, C and E only
(B) A, B, C, D and E
(C) A. B, D and E only
(D) A, B and E only

Answer: D

Noise pollution measured in decibels (dB) with reference to a standard sound intensity of the following magnitude

(A) 1 pico watt per mยฒ
(B) 1 milliwatt per mยฒ
(C) 1 nanowatt per mยฒ
(D) 1 microwatt per mยฒ

Answer: A

Which of the following are Millennium Development Goals?

A. Eradicating extreme poverty and hunger
B. Improving maternal health
C. Addressing climate change
D. Promoting gender equity and empowerment of women
E. Ensuring energy security for all

Choose the correct answer from the options given below:
(A) A, B, C and D only
(B) A, C, D and E only
(C) A, B, C,D and E
(D) A, B and D only

Answer: D

Which of the following are Millennium Development Goals?

A. Eradicating extreme poverty and hunger
B. Improving maternal health
C. Addressing climate change
D. Promoting gender equity and empowerment of women
E. Ensuring energy security for all

Choose the correct answer from the options given below:
(A) A, B, C and D only
(B) A, C, D and E only
(C) A, B, C,D and E
(D) A, B and D only

Answer: D

Identify the correct sequence of energy sources in terms of their heat of combustion in increasing order

A. Methane
B. Natural fats and oils
C. Dry wood
D. Green wood

Choose the correct answer from the options given below
(A)A,C,B,D
(B)D,C,B,A
(C)D,B,C,A
(D)D,C,A,B

Answer: B

Given below are two statements, one is labelled as Assertion A and the other is labelled as Reason R

Assertion A: Polluted rivers with very high values of Biological Oxygen Demand (BOO) may produce a foul smell
Reason R: Anaerobic bacteria produce sulphur dioxide and oxides of nitrogen

In light of the above statements, choose the most appropriate answer from the options given below
(A) Both A and R are correct and R is the correct explanation of A
(B) Both A and R are correct but R is NOT the correct explanation of A
(C) A is correct but R is not correct
(D) A is not correct but R is correct

Answer: C

Given below are two statements, one is labelled as Assertion A and the other is labelled as Reason R

Assertion A: Polluted rivers with very high values of Biological Oxygen Demand (BOO) may produce a foul smell
ReasonR: Anaerobic bacteria produce sulphur dioxide and oxides of nitrogen

In light of the above statements, choose the most appropriate answer from the options given below
(A) Both A and R are correct and R is the correct explanation of A
(B) Both A and R are correct but R is NOT the correct explanation of A
(C) A is correct but R is not correct
(D) A is not correct but R is correct

Answer: C

High hazard wastes may contain

A. Pathogens
B. Radioactive wastes
C. Non-reactive substances
D. Corrosive substances

Choose the correct answer from the options given below
(A) A, B and D only
(B) A, B, C and D
(C) B and D only
(D) A, B and C only

Answer: A

In case of which of the following primary pollutants, the man-made contributions to global emissions (million tonnes per year) is more compared to that from natural sources?

(A) Sulphur Dioxide
(B) Nitric Oxide
(C) Methane
(D) Carbon Dioxide

Answer: A

The phenomenon of ozone depletion is caused by

A. Volatile chlorinated hydrocarbons
B. Emissions from thermal power plants
C. Combustion of urban waster
D. Excessive use of nitrogen-containing fertilizers

Choose the correct answer from the options given below:
(A) A, B and D only
(B) A and D only
(C) A, B and C only
(D) A, C and D only

Answer: B

Which of the following was NOT an issue to be addresses under Millennium Development Goals?Option:

(A) Child Mortality
(B) Environmental sustainability
(C) Human rights
(D) Primary education

Answer: C

Identify the correct sequence of countries in decreasing order of their contributions to per capita emissions of carbon dioxide emissions at present

A. India
B. China
C. USA
D. Japan

Choose the correct answer from the options given below
(A)C,D.B,A
(B)B,C,A,D
(C)D,B,C,A
(D)C,B,D,A

Answer: A

Given below are two statements, one is labelled as Assertion A and the other is labelled as Reason R

Assertion A: In the contemporary environmental discourse, small hydropower plants are preferred over large hydrocarbon plants
Reason R: Installation of a small hydropower plant is less capital intensive compared to a large hydropower plant

In light of the above statements, choose the most appropriate answer from the options given below
(A) Both A and R are correct and R is the correct explanation of A
(B) Both A and R are correct but R is NOT the correct explanation of A
(C) A is correct but R is not correct
(D) A is not correct but R is correct

Answer: B

Day time noise standard prescribed for residential areas in India is Option:

(A) 75dB
(B) 65 dB
(C) 55dB
(D) 50 dB

Answer: C

Tn a polluted urban area, which one of the following has the highest concentration (ppm) in photochemical smog?

(A) Ozone
(B) PAN
(C) Hydrocarbons (without methane)
(D) Carbon monoxide

Answer: D

Under Goal 4 of Millennium Development Goals, the reduction sought to be achieved in under-five child mortality rate between 1990-2015 was to the extent of Option:

(A) Half
(B) Two-third
(C) Three-fourth
(D) One-third

Answer:B

Identify the correct sequence of countries in decreasing order of their contribution to global carbon dioxide emissions:

A. USA
B. China
C. Russia
D. India
E. Japan

Choose the correct answer from the options given below:
(A)A,B,D,C,E
(B)B,A,D,C,E
(C)B,A,D,E,C
(D)A,B,D,E,C

Answer:B

Given below are two statements, one is labelled as Assertion A and the other is labelled as Reason R

Assertion A: A windmill with proper design considerations can be made to operate at efficiencies in the range 60% to 65%
Reason R: The efficiency of the windmill depends on its design parameters

In light of the above statements, choose the most appropriate answer from the options given below:
(A) Both A and R are correct and R is the correct explanation of A
(B) Both A and R are correct but R is NOT the correct explanation of A
(C) A is correct but R is not correct
(D) A is not correct but R is correct

Answer:D

From disaster mitigation measures given below, identify the measures which can be characterized as non-structural:

A. Flood dykes
B. Land-use zoning
C. Raising of homes in flood-prone areas
D. Insurance Programmes
E. Reinforce tornado safe rooms

Choose the correct answer from the options given below:
(A) B and D only
(B) A, B and D only
(C) B, D and E only
(D) A, B, D and E only

Answer:A

Identify the correct sequence of continents in decreasing order of their yearly carbon dioxide emissions, at present

A. Africa
B. Asia
C. Europe
D. North America

Choose the correct answer from the options given below
(A)B,D,A,C
(B)B,D,C,A
(C)D,B,C,A
(D)D,C,B,A

Answer: B

Millennium Development Goals of Global Partnership for development envisaged making available the benefit of new technologies- especially which of the following?

(A) Information and communication
(B) Artificial Intelligence
(C) Nuclear Technology
(D) Solid Waste treatment technology

Answer: A

The Global emissions (by weight) of the following Primary pollutants from natural sources are maximum in case of

(A) Nitric Oxide
(B) Carbon Monoxide
(C) Methane
(D) Carbon dioxide

Answer: D

Match List I and List II which contains water pollutants and sources respectively

List I Water pollutants
A. Oxygen Demanding Wastes
B. Plant Nutrient
C. Acids
D. Polychlorinated biphenyls (PCBs)
List II Sources
I. Mine drainage
II. Modern washing powders and agriculture run off
III. Waste incineration and Toxic dumps
IV. Paper mills and Food Processing units

Choose the correct options given below:
(A) A-IV, B-III, C-II, D-I
(B) A-IV, B-I, C-III, D-II
(C) A-IV, B-II, C-I, D-II
(D) A-IV, B-III, C-I, D-II

Answer: C

Given below are two Statements, one is labelled as Assertion A and the other is labelled as Reason R

Assertion A: Despite increasing vulnerability to natural disasters many communities resist adopting mitigation programmes and measures
Reason R: Mitigation is often perceived by communities as being incompatible with their cultural Practices.

In the light of the above statements, choose the most appropriate answer from the options given below:
(A) Both A and R are correct, and R is Correct Explanation of A
(B) Both A and R are correct, but R is NOT correct explanation of A
(C) A is correct but R is not correct
(D) A is incorrect but R is correct

Answer: C

Which of the following are characteristics/propertics of permanent hardness in water?

A. Presence of bicarbonates of calcium and magnesium
B. Presence of sulphates of calcium and magnesium
C. Forms good lather when comes in contact with soap
D. Presence of chloride of calcium and magnesium
E. Can be removed by boiling the water

Choose the correct answer from the options given below :
(A) B, Cand D only
(B) A and D only
(C) A, B, D and E only
(D) B and D only

Answer: D

The Environment (protection) Act, 1986 was the result of Indiaโ€™s commitment, to take appropriate action for the protection and improvement of environment, at the: Option:

(A) 1985 Vienna convention for protection of ozone layer
(B) Convention on long range trans boundary air pollution, 1983
(C) Convention on international trade in endangered species, 1975 (CITES)
(D) United Nations Conference on the human environment, Stockholm, 1972

Answer: D

To characterize hazardous waste following characteristics are taken into account:

A. Conductivity
B. Ignitability
C. Corrosivity
D. Magnetic susceptibility
E. Reactivity

Choose the correct answer from the options given below:
(A) A, B, C and E only
(B) B, C and E only
(C) A, C and D only
(D) B, C, D and E only

Answer: B

Poly crystalline silicon often used in Photovoltaic cells as semiconductors are: Option:

(A) Many individual crystals with different orientations
(B) Many individual crystals with similar orientations
(C) Amorphous silicon converted to crystal silicon
(D) Crystal silicon grown in laboratory

Answer: A

What is the correct sequence of prescribed limits in residential areas of 24 hourly averaged following pollutants by Central Pollution Control Board of India?

A. PM25
B. PM10
C. SOx
D. Pb

Choose the correct answer from the options given below:
(A) Pb > SOx> PM10 > PM 2.5
(B) PM10 > PM2.5 > SOx > Pb
(C) PM2.5 > PM10> Pb > SOx
(D) PM10 > SOx > PM2.5 > Pb

Answer: D

Indiaโ€™s national-level forest policy dates back to : Option

(A) 1894
(B) 1952
(C) 1988
(D) 1994

Answer: A

Given below are two statements:

Statement I: Amorphous silicon is highly light absorbing material.
Statement II: Amorphous silicon is not suitable material for solar cell development due to its non-crystal properties

In light of the above statements, choose the correct answer from the options given below
(A) Both Statement I and Statement II are true
(B) Both Statement I and Statement II are false
(C) Statement I is true but Statement II is false
(D) Statement I is false but Statement II is true

Answer: C

Which one of the following has, both positive and negative radiative forcing in the climate? Option:

(A) CO2
(B) CH4
(C) NO2
(D) O3

Answer: D

Given below are two statements, one is labelled as Assertion A and the other is labelled as Reason R

Assertion A: Lockdown in 2020 in India was imposed under the Disaster Management Act
Reason R: COVID 19 is the first pan-India biological disaster being handled under the Disaster Management Act

In light of the above statements, choose the correct answer from the options given below :
(A) Both A and R are true and R is the correct explanation of A
(B) Both A and R are true but R is NOT the correct explanation of A
(C) A is true but R is false
(D) A is false but R is true

Answer: A

The faintest detectable noise by a healthy human ear is of: Option:

(A) 0 dB
(B) 1 dB
(C) 10 dB
(D) 20 dB

Answer: A

Given below are two statements :

Statement I: Values are an enduring belief about the way things should be done or about the ends we desire Statement II: The values we form are influenced by our circumstances families, friends, age, gender, environment, tradition and culture

In light of the above statements, choose the most appropriate answer from the options given below:
(A) Both Statement I and Statement IT are correct
(B) Both Statement I and Statement II are incorrect
(C) Statement I is correct but Statement II is incorrect
(D) Statement I is incorrect but Statement II is correct

Answer: A

Given below are two statements

Statement I: Oxidative stress is an imbalance between accumulation of oxygen reactive substances and ability of body to destroy them.
Statement II: Oxidative stress is significantly lowered by the presence of oxidants.

In light of the above statements, choose the correct answer from the options given below
(A) Both Statement I and Statement II are true
(B) Both Statement I and Statement II are false
(C) Statement I is true but Statement II is false
(D) Statement I is false but Statement II is true

Answer: C

What is gasohol? Option:

(A) A type of chemical used for chemical remediation of waste water
(B) An instrument used to detect gas leakages in factories
(C) Mixture of gasoline and ethanol
(D) A type of chemical for the decomposition of stubbles

Answer: C

The target year for achieving the Sustainable Development Goals is Option:

(A) 2020
(B) 2025
(C) 2030
(D) 2035

Answer: C

Match List I with List II

List I (Disaster Management Cycle Components)
A. Prevention
B. Preparedness
C. Recovery
D. Response
List II (Explanation)
I. Land-use planning
II. Rescue
III. Risk assessment
IV. Construction

Choose the correct answer from the options given below:
(A)A-III, B-I, C-II, D-IV
(B)A-II, B-I, C-IV, D-II
(C)A-I, B-II, C-II, D-IV
(D)A-I, B-III, C-II, D-IV

Answer: B

Given below are two statements, one is labelled as Assertion A and the other is labelled as Reason R

Assertion A: The decibel level for a given loudness may be different at different frequencies in the audible range.
Reason R: The human ear has a differential frequency response.

In light of the above statements, choose the correct answer from the options given below
(A) Both A and R are true and R is the correct explanation of A
(B) Both A and R are true and R is NOT the correct explanation of A
(C) A is true but R is false
(D) A is false but R is true

Answer: A

Given below are two statements, one is labelled as Assertion A and the other is labelled as Reason R

Assertion A: Chromium and Iron contamination is found in wastewater of some industries
Reason R: Higher dose of Chromium and Iron can cause toxic effects in our body.

In light of the above statements, choose the most appropriate answer from the options given below
(A) Both A and R are correct and R is the correct explanation of A
(B) Both A and R are correct but R is NOT the correct explanation of A
(C) A is correct but R is not correct
(D) A is not correct but R is correct

Answer: B

Blue baby syndromeโ€™ in infants is mainly caused by presence of Option:

(A) Sulphate in water
(B) Nitrate in water
(C) Fluoride in water
(D) Arsenic in water

Answer: B

Climate changes involve factors, both internal and external. External ones include

A. Feedback between the atmosphere, ocean and land surface
B. Solar variability
C. Earthโ€™s orbit alteration
D. Variability within the climate system
E. Volcanic activity

Choose the correct answer from the options given below:
(A) A, B and C only
(B) B, C and E only
(C) C,D and E only
(D) A, B and E only

Answer: B

Identify the correct sequence of the following noise pollution indices in decreasing order of their magnitude in an urban area

A. Big
B. Logg
C. Lg

Choose the correct answer from the options given below
(A) A, C, B
(B) A, B, C
(C) B, A, C
(D) B, C, A

Answer: A

Given below are two statements, one is labelled as Assertion A and the other is labelled as Reason R

Assertion A: As a rule of thumb, sites of average wind speed 8 m/s or above can be considered very good for electricity generation from wind turbines.
Reason R: For average wind speed < 8 m/s, the power produced is not very significant.

In light of the above statements, choose the correct answer from the options given below (
A) Both A and R are true and R is the correct explanation of A
(B) Both A and R are true but R is NOT the correct explanation of A
(C) A is true but R is false
(D) A is false but R is true

Answer: C

Which of the following are the missions of the National Action Plan on Climate Change?

A. National Solar Mission
B. Green Indian Mission
C. Western Ghat Development Mission
D. National Water Mission
E. Air Pollution Mitigation Mission

Choose the correct answer from the options given below:
(A) A, B and D only
(B) A, B, D and E only
(C) A, B and E only
(D) A, C, D and E only

Answer: A

Nationally Determined Contributions of European Union (EU) countries aim at a reduction in Green House Gas emissions by the year 2030 (in the reference to 1990 levels) to the extent of Option:

(A) 40%
(B) 50%
(C) 35%
(D) 55%

Answer: A

Which of the following contaminants in water can cause adverse human effects such as mental disturbance and impairment of speech, hearing and vision? Option:

(A) Arsenic
(B) Methyl mercury
(C) Lead
(D) Nickel and Chromium

Answer: C

Identify the correct sequence of countries that are the worldโ€™s leading geothermal power (in MW) producers.

A. New Zealand
B. Iceland
C. USA
D. Indonesia

Choose the correct answer from the options given below
(A)C>A>B>D
(B)A>C>B>D
(C)D>C>A>B
(D)C>D>A>B

Answer: D

Match List I with List II

List 1
A. Montreal Protocol
B. Rio Summit
C. Kyoto protocol
D. Paris agreement
List II
I. Environment and sustainable development
II. Reduce the emission of Green House Gases
III. Climate change mitigation and adaptation
IV. Ozone Depletion

Choose the correct answer from the options given below:
(A)A-III, B-I, C-II, D-IV
(B)A-II, B-III, C-IV, D-I
(C)A-IV, B-I, C-II, D-III
(D)A-IV, B-II, C-I, D-II

Answer: C

According to the Sustainable Development Report (2020), the top-ranked country on the basis of the SDG Index is Option:

(A) Bhutan
(B) New Zealand
(C) Sweden
(D) United Kingdom

Answer: C

Which one of the following waterborne diseases does not require direct contact with water but a host to spread? Option:

(A) Cholera
(B) Malaria
(C) Trachoma
(D) Schistosomiasis

Answer: B

Paris agreement entered into force when the instruments of ratification were deposited by at least 55 countries which accounted for, at least Option:

(A) 50% of global CO emissions
(B) 55% of global CO emissions
(C) 60% of global CO emissions
(D) 75% of global CO emissions

Answer: B

Biogas has a calorific value of about Option:

(A) 5000 kcal per cubic meter
(B) 3000 kcal per cubic meter
(C) 2500 kcal per cubic meter
(D) 1800 kcal per cubic meter

Answer: A

Under which of the following agreements was it decided to limit the increase in global average temperature to well below 2ยฐC? Option:

(A) Paris Agreement
(B) Kyoto Protocol
(C) Montreal Protocol
(D) Rio Summit

Answer: A

Identify the correct sequence of BRICS countries in relation to their Sustainable Development Goals Index (from highest to lowest) as per the Sustainable Development Report (2020)

A. China
B. India
C. South Africa
D. Brazil
E. Russian Federation

Choose the correct answer from the options given below:
(A)C>D>E>A>B
(B)E>D>C>A>B
(C)C>E>B>D>A
(D)A>D>E>C>B

Ans: D

Kigali Amendment to the Montreal Protocol envisages reduction in HFCs consumption by late 2040s to the extent of: Option:

(A) 50-35%
(B) 65-70%
(C) 80-85%
(D) 90-95%

Ans: C

According to Red list, the percentage of threatened mammals in the world is about? Option:

(A) 20%
(B) 25%
(C) 40%
(D) 45%

Ans: B

The permissible limit for Arsenic in drinking water, as per Indian Standards [IS:10500] is: Option:

(A) 0.05mg
(B) 0.5mg
(C) 1.0mg
(D) 2.0mg

Ans: A

From the energy security perspective, which of the following energy sources is considered most secure for India? Option:

(A) Geothermal
(B) Solar
(C) Wind
(D) Hydro

Ans: B

The disability characterized by significant limitations in both intellectual functioning and adaptive behaviour as expressed in conceptual, social, and practical adaptive skill, is known as-

A. Articulation disorder
B. Intellectual disability
C. Cognitive impairment
D. General learning disability
E. Hyperactivity

Choose the correct answer from the options given below:
(A) A, C and E only
(B) B. C and D only
(C) A, B and C only
(D) C, D and E only

Answer: B

Which of the following source(s) is/are โ€˜nonpoint source(s)โ€™ of groundwater contamination?

A. Wastewater ponds
B. Refuse piles
C. Landfills
D. Agricultural land
E. Buried storage tanks

Choose the correct answer from the options given below:
(A) A, D and E only
(B) A, Cand D only
(C) D only
(D) B, C and D only

Answer: C

Which of the following are non-structural measures in the context of Disaster Mitigation?

A. Hazard mapping
B. Raising of homes in flood-prone areas
C. Insurance program
D. Relocating equipment outside tlood plains

Choose the correct answer from the options given below:
(A) B and C only
(B) B. C and D only
(C) A, Cand D only
(D) A and C only

Answer: C

In a typical fine continental aerosol, according to its composition, which of the following constituents is most abundant? Option:

(A) Sulphate
(B) Organic Carbon
(C) Elemental Carbon
(D) Nitrate

Answer: A

The main environmental concern associated with geothermal energy is the release of Option:

(A) Carbon monoxide
(B) Methane
(C) Carbon dioxide
(D) Hydrogen sulphide

Answer: D

Identify the correct sequence of countries in increasing order of their present annual carbon dioxide emissions.

A. India
B. China
C. USA
D. Russia
E. Japan

Choose the correct answer from the options given below:
(A)E<D<A<C<B
(B)E<D<A<B<C
(C)D<E<A<C<B
(D)D<E<A<B<C

Answer: A

โ€œTo conserve and sustainably use the oceans, seas and marine resourcesโ€™ โ€” is the aim of sustainable development Goal number: Option:

(A) 10
(B) 12
(C) 14
(D) 16

Answer: C

According to composition (%) of typical fine continental aerosols, identify the correct sequence in decreasing order.

A. Sulphate
B. Ammonium
C. Organic carbon
D. Elemental Carbon

Choose the correct answer from the options given below:
(1)A>C>B>D
(2)A>B>C>D
(3)C>A>B>D
(4)C>D>B>A

Answer: A

Point sources of water pollution are:

A. Underground coal mines
B. Run-off from farm fields
C. Sewage treatment plants
D. Run-off from roads and construction sites
E. Power plants

Choose the correct answer from the options given below:
(A) C, E only
(B) A, C, E only
(C) A, C, D, E only
(D) B, C, E only

Answer: B

The optimal range of wind speeds for wind power generation is: Option:

(A) ~32 โ€” 40 m/s
(B) ~24 โ€” 30m/s
(C) ~15 โ€” 22 m/s
(D) ~4 โ€” 12m/s

Answer: D

In the international Solar Alliance Mission, these entities are corporate Partners from India :

A. National Thermal Power corporation
B. Power grid corporation
C. State Bank of India
D. Rural Electrification Corporation
E. Indian Renewable Energy Development Agency

Choose the correct answer from the options given below:
(A) A, D, E only
(B) A, C, D only
(C) A, B, D only
(D) A, B, D, E only

Answer: D

Under Sustainable Development Goal 3 โ€” โ€˜Good Health and Wellbeingโ€™. by 2030, the premature mortality from non-communicable diseases is to be reduced by Option:

(A) 75%
(B) 60%
(C) 50%
(D) 40%

Answer: C

Major factors for loss of biodiversity are

A. Changes in land and sea use
B. Acid rain
C. Direct exploitation of organisms
D. Climate change
E. Ozone depletion

Choose the correct answer from the options given below:
(A) C, D, E only
(B) A, C, D only
(C) B, C, D only
(D) A, B, C, D only

Answer: B

Identify the Natural Hazards of geophysical origin among the following:

A. Wildfires
B. Floods
C. Subsidence
D. Tsunami
E. Heat wave

Choose the correct answer from the options given below:
(A) A, B, C, D only
(B) B, C, D only
(C) B, C only
(D)B, C, D, E only

Answer: B

Given below are two statements: One is labelled as Assertion A and the other is labelled as Reason R:

Assertion A: In comparison to coal and oil, natural gas contributes much less to global warming per unit energy.
Reasons R: Natural gas has lesser hydrogen per carbon atom than oil or coal. In the light of the above statements. choose the most appropriate answer from the options given below:

(A) Both A and R are correct and R is the correct explanation of A
(B) Both A and R are correct but R is NOT the correct explanation of A
(C) A is correct but R is not correct
(D) A is not correct but R is correct

Answer: C

During second commitment period (2013-2020) for parties under Kyoto Protocol. the target for reduction in Green House emissions with reference to 1990 levels, is Option:

(A) 7.6%
(B) 5.8%
(C) 15%
(D) 18%

Answer: D

By the year 2020. one of the targets in sustainable Development Goals is to reduce global deaths and injuries from road traffic accidents by Option:

(A) 38%
(B) 40%
(C) 50%
(D) 25%

Answer: C

Which of the following is perceived relatively a low risk problem in the context of human welfare? Option:

(A) Radionuclides
(B) Airborne toxins
(C) Stratospheric ozone depletion
(D) Habitat alteration and destruction

Answer: A

According to WHO standards for drinking water quality, the concentration of Arsenic should not exceed: Option:

(A) 10 pgL
(B) 6.0 pgL
(C) 5.0 peL
(D) 1.0 pgL

Answer: A

During a combustion process, the following air pollutants are produced:

A. Nitric oxide
B. Nitrogen dioxide
C. Nitrous oxide
D. Carbon monoxide
E. Ozone

Choose the correct answer from the options given below:
(A) A and C only
(B) A, Cand D only
(C) B.D and E only
(D) A, B and D only

Answer: D

The โ€œVision statementโ€™ of International Solar Alliance is Option:

(A) Let us together make the world a better place
(B) Let us together make the sun brighter
(C) Let us make the world more energy cfficient
(D) Let us have a carbon-neutral world

Answer: B

According to one of the Sustainable Development Goals, the per capita global food waste at the retail and consumer levels is to be reduced to the extent of Option:

(A)1/2
(B)1/3
(C)1/4
(D)1/10

Ans: A

According to WHO standards, the pesticide content (Lindane or DDT) in drinking water should not exceed: Option:

(A) 1 ?gL-1
(B) 2 ?gL-1
(C) 5 ?gL-1
(D) 10 ?gL-1

Ans: B

Which of the following Pollutants has the least residence time in atmosphere? Option:

(A) Nitric Oxide
(B) Nitrous Oxide
(C) Chlorofluorocarbons
(D) Methane

Ans: D

Noise levels (db) are referenced to the human hearing threshold at a frequency of Option:

(A) 50Hz
(B) 100 Hz
(C) 1 KHz
(D) 4 KHz

Ans: C

International Solar Alliance Framework Agreement entered into force in the year Option:

(A) 2015
(B) 2017
(C) 2016
(D) 2018

Ans: B

What will be the resultant of two noise levels of 81 dB and 90 dB generated by two independent sources? Option:

(A) ~96 dB
(B) ~91dB
(C) ~98 dB
(D) ~118 dB

Answer: B

Match List I with List II

List I
A. Kyoto Protocol
B. Montreal Protocol
C. Paris Agreement
D. International Solar Alliance
List II
I. Clean and Affordable Energy
II. Clean Development Mechanism
III. Nationally Determined Contribution
IV. Phase out of chlorinated compounds

Choose the correct answer from the options given below:
(A)A-II, B-IV, C-III, D-I
(B)A-I, B-III, C-IV, D-II
(C)A-II, B-IV, C-I, D-III
(D)A-III, B-IV, C-I, D-II

Answer: A

Given below are two statements :

Statement I: Solar flat plat collectors essentially work on the principle of Green House Effect.
Statement II: Black absorber plat radiates infrared radiations.

In light of the above statements, choose the correct answer from the options given below
(A) Both Statement I and Statement II are true
(B) Both Statement I and Statement II are false
(C) Statement I is true but Statement II is false
(D) Statement I is false but Statement II is true

Answer: A

Identify the correct sequence in decreasing order of percentage of total worldโ€™s water in the following locations:

A. Atmosphere
B. Rivers and streams
C. Ground water
D. Lakes (Fresh water)
E. Ice caps and Glaciers

Choose the correct answer from the options given below:
(A)E,C,B,D,A
(B)E,B,C,D,A
(C)E,D,C,B,A
(D)E,C,D,A,B

Answer: D

By the year 2030. Sustainable Development Goal 7 aims at enhancing the global rate of improvement in energy efficiency by a factor of Option:

(A) Two
(B) Three
(C) Four
(D) Five

Answer: A

Identify the correct sequence for phases of personal growth in the non-directive interview process.

A. Develops insight
B. Integration
C. Release of feelings
D. Action
E. New orientation

Choose the correct answer from the options given below
(A)A,B,C,D,E
(B)B.E,D,A,C
(C)C,A,D,B,E
(D)C,E,B,A,D

Answer: C

Given below are two statements

Statement I: Behaviour theory does not concentrate on observable behaviour.
Statement II: Behaviour theory approach involves continuous inquiry.

In light of the above statements, choose the most appropriate answer from the options given below
(A) Both Statement I and Statement II are correct
(B) Both Statement I and Statement II are incorrect
(C) Statement I is correct but Statement II is incorrect
(D) Statement I is incorrect but Statement II is correct

Answer: D

Match List I with List IT

List I Eriksonโ€™s Stages of Psychological Development
A. Basic trust versus basic mistrust
B. Industry versus inferiority
C. Intimacy versus isolation
D. Generativity versus stagnation
List II Relevant Illustrations
I. Parenting/ mentoring
II. Love relationships
III. School
IV. Feeding

Choose the correct answer from the options given below:
(A) A-I, B-II, C-III, D-IV
(B) A-II, B-I, C-IV, D-II
(C) A-IV, B-III, C-II, D-I
(D) A-III, B-IV, C-I, D-II

Answer: C

Eutrophication in a water body leads to which of the following?

A. Bloom of algae
B. Increase in the level of dissolved oxygen (DO)
C. Increase in turbidity and odor of the water body
D. Difficulty in sustaining normal aquatic life
E. Deficiency of nutrients

Choose the correct answer from the options given below:
(A) A, B and C only
(B) B, C and D only
(C) A, C and D only
(D) C, D and E only

Answer: C

Which of the following has positive radiative forcing or warming effect in the global climate? Option:

(A) Tropospheric Ozone
(B) Stratospheric Ozone
(C) Sulphate Aerosol
(D) Sulphur dioxide

Answer: A

Given below are two statements

Statement I: Methanol (CH;OH) also known as wood alcohol and has a much higher octane rating than gasoline.
Statement II: Methanol burns with higher flame temperature than gasoline or diesel. Thus, it produces a higher amount of NOx.

In light of the above statements, choose the correct answer from the options given below
(A) Both Statement I and Statement II are true
(B) Both Statement I and Statement II are false
(C) Statement I is true but Statement II is false
(D) Statement I is false but Statement II is true

Answer: C

Targets of Goal 7 of the Millennium Development Goals (MDG) are

A. Control global warming
B. Universal access to modern energy
C. Increase global percentage of renewable energy
D. Mitigating air pollution
E. Double the improvement in energy efficiency

Choose the correct answer from the options given below:
(A) A, B, Cand D only
(B) B. C and D only
(C) B, Cand E only
(D) A, C,D and E only

Answer: B

The primary treatment of wastewater involves removal of Option:

(A) Solid
(B) Colloids
(C) Organic water
(D) Microorganisms

Answer: A

NSFQ organises qualification according to series of levels of Option:

(A) Knowledge, Skills, Experience
(B) Skills, Experience, Aptitude
(C) Knowledge, Skills, Aptitude
(D) Skills, Achievement, Aptitude

Answer: C

Convention on Biodiversity was signed during Option:

(A) Montreal Protocol, 1987
(B) Earth Summit at Rio de Janeiro, 1992
(C) Kyoto Protocol, 1997
(D) Human Environment Conference, Stockholm, 1972

Answer:B

Given below are two statements

Statement I: Aerobic digestion of sewage sludge requires lots of energy
Statement II: Aerobic digestion of sewage sludge produces huge amount of methane

In light of the above statements, choose the correct answer from the options given below
(A) Both Statement I and Statement II are true
(B) Both Statement I and Statement II are false
(C) Statement 1 is true but Statement II is false
(D) Statement I is false but Statement II is true

Answer: C

Given below are two statements, one is labelled as Assertion A and the other is labelled as Reason R

Assertion A : Ground level concentration of ozone decreases at night
Reason R : Ultraviolet radiation photolyzes the ozone into other components

In light of the above statements, choose the correct answer from the options given below
(A) Both A and R are true and R is the correct explanation of A
(B) Both A and R are true but R is NOT the correct explanation of A
(C) A is true but R is false
(D) A is false but R is true

Answer: B

Targets of Goal 7 of the Millennium Development Goals (MDG) are

A. Control global warming
B. Universal access to modern energy
C. Increase global percentage of renewable energy
D. Mitigating air pollution
E. Double the improvement in energy efficiency

Choose the correct answer from the options given below:
(A) A, B, Cand D only
(B) B. C and D only
(C) B, Cand E only
(D) A, C,D and E only

Answer: C

โ€˜Solar water stillโ€™ is a device to Option:

(A) Pump water using solar energy
(B) Convert saline water to potable water using solar energy
(C) Heat water using solar energy
(D) Generate electricity using solar energy

Answer: B

Given below are two statements

Statement I: Summer smog is primarily associated with photochemical formation of ozone and other pollutants
Statement II: The Great Smog of London disaster, is a classic example of summer smog

In light of the above statements, choose the correct answer from the options given below
(A) Both Statement I and Statement II are true
(B) Both Statement I and Statement II are false
(C) Statement I is true but Statement II is false
(D) Statement I is false but Statement II is true

Answer: C

Which of the following are the main goals of the Convention on Biological Diversityโ€™s (CBDโ€™s)?

1. Protection of water resources
2. The conservation of biological diversity
3. Promotion of shifting agriculture
4. The sustainable use of biological components
5. Fair and equitable sharing of benefits arising from genetic resources

Choose the correct answer from the options given below:
(A) 1, 2 and 3 only
(B) 1, 3, and 5 only
(C) 2, 4 and 5 only
(D) 1, 4 and 5 only

Answer: C

Given below are two statements

Statement I: In waste water treatment systems, most of the large floating materials are removed in the screen chamber
Statement II: Colloidal and dissolved organic content of waste water are removed in the grit chamber

In light of the above statements, choose the most appropriate answer from the options given below
(A) Both Statement I and Statement II are correct
(B) Both Statement I and Statement II are incorrect
(C) Statement I is correct but Statement II is incorrect
(D) Statement I is incorrect but Statement II is correct

Answer: C

Which one of the following Articles of the Indian Constitution talks about our duty to protect the environment?

(A) Article 51A(a)
(B) Article 51A(c)
(C) Article 51A(e)
(D) Article 51A(g)

Answer: D

Arrange the following naturally occurring Uranium isotopes in the correct sequence as per the proportion of occurrence

1.U-234
2. U-235
3. U-238

Choose the correct answer from the options given below
(A) 1, 2,3
(B) 2, 3, 1
(C) 3, 2, 1
(D) 3, 1, 2

Answer: C

Given below are two statements, one is labelled as Assertion A and the other is labelled as Reason R:-

Assertion A: The in-situ passive soil remediation process is the easiest to implement and the least expensive Reason R: It relies upon several natural processes to destroy the contaminants

In light of the above statements, choose the most appropriate answer from the options given below:
(A) Both A and R are correct and R is the correct explanation of A
(B) Both A and R are correct but R is NOT the correct explanation of A
(C) A is correct but R is not correct
(D) A is not correct but R is correct

Answer: A

Dewatering of sewage sludge can be achieved by:-

A. Gravity thickening
B. Mechanical stirring
C. Osmosis
D. Floatation
E. Screening

Choose the correct answer from the options given below:
(A) A, B and D only
(B) B, C and E only
(C) A, C and D only
(D) C, D and E only

Answer: A

Which one of the following isotopes of Uranium is fissile material and can be used for nuclear reactions? Option:

(A) U-234/U-234
(B) U-235/U-235
(C) U-236/U-236
(D) U-238/U-238

Answer: B

Rain is termed as acid rain when its pH value is:- Option:

(A) 1-3/1 โ€“ 3
(B) Less than 5.6/5.6
(C) 5.6-7.0/5.6 โ€“ 7.0
(D) More than 7/7

Answer: B

Which one among the following diseases is NOT caused due to exposure to ultra-violet radiation?

(A) Bronchitis
(B) Melanoma
(C) Ocular damage
(D) Erythema

Answer: A

Match List I with List II

List I Class of water pollutants
A. Disease-causing agents
B. Oxygen depleting wastes
C. Inorganic plant nutrients
D. Water-soluble inorganic chemicals
List II Example/Explanation
I. Organic wastes decomposed by aerobic bacteria
II. Acids, salts, compounds of toxic metals
III. Bacteria, protozoa, worms etc.
IV. Water-soluble nitrates and phosphate cause cxeessive growth of algac etc

Choose the correct answer from the options given below:
(A) A -I, B-IV, C- II, D โ€“ IIl
(B) A โ€“ II, B- IIl, C-I, D-IV
(C) A- III, B-I,C-IV, D-II
(D) A -IV, B- Il, C- Ill, D-I

Answer: C

Given below are two statements

Statement I: Nitrification results in an increase in effluent ammonia toxicity
Statement II: Denitrification reduces nitrate to nitrogen gas using bacteria

In light of the above statements, choose the most appropriate answer from the options given below:
(A) Both Statement I and Statement II are correct
(B) Both Statement I and Statement II are incorrect
(C) Statement I is correct but Statement I is incorrect
(D) Statement I is incorrect but Statement II is correct

Answer: D

Flotation is a unit operation, used in wastewater treatment to

A. Remove the lighter suspended solids
B. Concentrate biological sludge
C. Remove oil and grease
D. Remove the temporary hardness of the water

Choose the correct answer from the options given below:
(A) A, B and C only
(B) B, C and D only
(C) A, B and D only
(D) A, B, C and D

Answer: A

Match List I and List II:

List I Ozone depleting substance
A. C3HF7
B. C2FH3Cl2
C. C2 Fy1Cl2
D. CF3Br
List II
I. HCFC
II. Halons
III. HFC
IV. CFC

Choose the correct answer from the options given below:
(A) A-I,B-III, C-II D-IV
(B) A- III, B-I, C-IV, D-II
(C)A-II, B-IV, C-III, D-I
(D) A -IV, B-II, C-I, D-III

Answer: B

What are essential for the formation of photochemical smog?

1. Oxides of sulphur
2. Oxides of Nitrogen
3. Volatile Organic Compounds
4. Water Vapour
5. Sunlight

Choose the correct answer from the options given below:
(A) 2, 3 and 5 only
(B) 1, 2 and 3 only
(C) 1, 2, 3 and 4 only
(D) 2, 3, 4 and 5 only

Answer: D

What is the CORRECT sequence of occurrence of various atmospheric layers from earthโ€™s surface?

I. Thermosphere
2. Stratosphere
3. Mesosphere
4. Troposphere

Choose the correct answer from the options given below
(A) 4, 2, 1,3
(B) 4, 2, 3, 1
(C) 2, 4, 3,1
(D) 3, 2, 4, 1

Answer: B

Which one of the following climate change proxies is not the terrestrial biotic climatic proxy? Option:

(A) Boreholes
(B) Leaf physiology
(C) Tree ring
(D) Pollen and spore analysis

Answer: A

A high amount of BOD (Biochemical Oxygen Demand) in a water body represents

1. Highly Polluted Water
2. Presence of a large amount of organic materials
3. Presence of good amount dissolved oxygen
4. Severely limited aquatic life

Choose the correct answer from the options given below:
(A) 1, 2, 3 and 4 only
(B) 2, 3 and 4 only
(C) 1, 3 and 4 only
(D) 1, 2 and 4 only

Answer: D

According to NITI Ayogโ€™s SDG Index (2019-20), which among the following states is the best performer in the attainment of SDG-5 (Gender Equity)?

(A) Kerala
(B) Himachal Pradesh
(C) Sikkim
(D) Goa

Answer: B

Protection from Ultraviolet Radiations as envisaged under the Montreal Protocol also contributes to following SDGs

(A) SDGS 3, 11, 14, 15
(B) SDGs 10, 16, 17
(C) SDGs 4, 5, 11, 17
(D) SDGs 8, 10, 12

Answer: A

Among all recorded species in the world, India accounts for about

(A) 7-8 %
(B) 10 โ€“ 12 %
(C) 15 -18 %
(D) 20 โ€“ 25 %

Answer: A

In respect of Biochemical Oxygen Demand (BOD), the permissible Indian standard for discharge of treated effluents in Inland Surface Waters is

(A) 5.0 mg/L
(B) 6.0 mg/L
(C) 15.0 mg/L
(D) 30.0 mg/L

Answer: D

Which type of power plants uses the least land per unit of GWh energy production?

(A) Wind
(B) Coal
(C) Solar
(D) Geothermal

Answer: D

Given below are two statements

Statement I: Personal competencies determine how we manage ourselves
Statement II: Social competencies determine how we handle relationships

In light of the above statements, choose the most appropriate answer from the options given below:
(A) Both Statement I and Statement II are correct
(B) Both Statement I and Statement II are incorrect
(C) Statement I is correct but Statement II is incorrect
(D) Statement I is incorrect but Statement II is correct

Answer: A

Match List I with List II

List I Thinking Hat System
A. White Hat
B. Red Hat
C. Black Hat
D. Green Hat
List II Concepts and Ideas
I. Judgement and caution-the logical negative
II. Data gathering-facts, figures, information needs and gaps
III. Provocations, alternatives and creativity-proposals
IV. Intuition

Choose the correct answer from the options given below:
(A) A -III, B-II, C-IV , D-I
(B) A-II, B-IV, C-I, D-II
(C)A-IV, B-I, C -III, D-II
(D)A-I, B-III, C-II, D-IV

Answer: B

Which of the following are in the โ€œSix Thinking Hats Systemโ€, a form of parallel thinking?

A. Brown Hat
B. White Hat
C. Violet Hat
D. Red Hat
E. Black Hat

Choose the correct answer from the options given below:
(A) A, B and C only
(B) B, C and D only
(C) C, D and E only
(D) B, D and E only

Answer: D

In agricultural regions, groundwater can have significant concentrations of which pollutant?

(A) Lead
(B) Cadmium
(C) Selenium
(D) Nitrate

Answer: D

Which of the following types of power plants provide electricity consistently running 24 hrs per day, 7 days a week?

(A) Geothermal
(B) Solar
(C) Wind
(D) Hydro

Answer: A

Which of the following Green House bases were the target gases whose emission was to be covered under the first commitment period of the Kyoto protocol?

A. SO
B. CO
C. N20
D. NH3
E. CH

Choose the correct answer from the options given below:
(A) A, B, C and D only
(B) A, B and C only
(C) B, C, D and E only
(D) B, C and E only

Answer: D

In India, according to Niti Ayogโ€™s SDG Index (2019-2020), which is the best performing state on SDG-9 (Industry, Innovation and Infrastructure)

(A) Kerala
(B) Gujrat
(C) Karnataka
(D) Telangana

Answer: A and B

Identify the correct sequence of wind energy potential at 100 m above ground level in decreasing order

A. Karnataka
B. Andhra Pradesh
C. Gujarat
D. Tamil Nadu
E. Andaman and Nicobar Islands

Choose the correct answer from the options given below:
(A) A>B>C>E>D
(B) D>C>A>B>E
(C) C>A>B>D>E
(D) C>E>B>D>A

Answer: C

The waste component of waste categorized as โ€œTrashโ€™ consists of

A. Paper, Cardboard cartons
B. Animal wastes
C. Industrial process wastes
D. Vegetable wastes
E. Wood

Choose the correct answer from the options given below:
(A) A and E only
(B) A, C and E only
(C) A, C, D and E only
(D) A, B, D and E only

Answer: A

Under Sustainable Development Goal 12- Sustainable Consumption and Production, one of the targets is to reduce per capita food waste at the retail and consumer levels by:

(A) 3/4
(B) 1/2
(C) 1/3
(D) 1/4

Answer: B

Small hydro power potential in India is estimated approximately as

(A) ~ 60 MW
(B) ~ 100 MW
(C) ~ 10 GW
(D) ~ 20 GW

Answer: D

Small hydro power potential in India is estimated approximately as

(A) ~ 60 MW
(B) ~ 100 MW
(C) ~ 10 GW
(D) ~ 20 GW

Answer: D

The global environmental issue of Ozone depletion is due to emission of :

A. Sulphur dioxide
B. Chlorinated hydrocarbons
C. Methane
D. Carbone dioxide

Choose the most appropriate answer from the options given below :
(A) A and B only
(B) B only
(C) B and D only
(D) A, B and C only

Answer: B

Match List I with List II

List I (Digital Initiatives in Higher Education)
A. SWAYAM
B. SWAYAM PRABHA
C. Digilocker
D. NAD
List II (Purpose)
I. Online store and access of academic awards namely degrees, mark-sheets etc.
II. Store and access personal c-documents irespective of physical location.
III. MOOCs for high-quality cducation anywhere and anytime.
IV. Free DTH channel for high-quality education.

Choose the correct answer from the options given below:
(A) A-IIL, B-IV, C-II, D-I
(B) A-IV, B-III, C-I, D-II
(C) A-IV, B-Ill, C-II, D-I
(D) A-Il, B-I, C-III, D-IV

Answer: A

A coal fired thermal power plant has an efficiency of 35%. The electricity from the power plant is used entirely for lighting purposes with average efficiency of 20%. What is the overall efficiency of conversion from coal to lighting?

(A) 15%
(B) 55%
(C) 70%
(D) 7%

Answer: D

As per Sustainable Development Goal 3, one of the targets is to reduce under-5 mortality per 1000 live births to at least as low as

(A) 50
(B) 40
(C) 25>
(D) 15

Answer: C

In a polluted urban area, which of the following constituents of photochemical smog has the least concentration (parts per billion by volume)?

(A) CO Carbon monoxide
(B) Nitrogen dioxide
(C) Hydrocarbons (without methane)
(D) Ozone

Answer: D

For drinking and irrigation purposes, the availability of suitable water out of earthโ€™s total water supplies is about less than

(A) 1%
(B) 5%
(C) 6%
(D) 10%

Answer: A

Tidal range (R) is a critical factor in determining whether an estuary would be useful for tidal power generation as energy (E) available per tidal cycle depends on scales with R as according to the following relation:

(A)EaR
(B) Ea R3/2
(C) Ea R2
(D) Ea R3

Answer: C

How many missions are there under the Climate Action Plan of Govt. of India?

(A) 4
(B) 8
(C) 12
(D) 1

Answer: B

According to India Meteorological Department, average sustained wind speed over 3 minutes of a cyclonic storm. is in the range

(A) 40 โ€” 50 km/hr
(B) 50 โ€” 60 km/hr
(C) 63 โ€” 88 km/hr
(D) 200 โ€” 225 km/hr

Answer: C

Given below are two statements:

Statement I: Wind farms are generally set up on exposed hilltops or coasts in remote areas.
Statement II: Wind velocities are steady in these remote locations.

In the light of the above statements, Choose the most appropriate answer from the options given below:
(A) Both Statement I and Statement II are correct
(B) Both Statement I and Statement II are incorrect
(C) Statement I is correct but Statement II is incorrect
(D) Statement I is incorrect but Statement II is correct

Answer: C

Given below are two statements: One is labelled as Assertion A and the other is labelled as Reason R.

Assertion A: Production of electricity from Renewable Energy sources is without adverse environmental impacts.
Reason R: There is no net emission of carbon di-oxide into atmosphere from exploitation of Renewable sources of energy.

In the light of the above statements, Choose the most appropriate answer from the options given below :
(A) Both A and R are true and R is the correct explanation of A
(B) Both A and R are true but R is NOT the correct explanation of A
(C) A is true but R is false
(D) A is false but R is true

Answer: D

โ€˜Wheezingโ€™ is the most relevant Health Effect in case of the air pollutant

(A) Ozone
(B) Carbon monoxide
(C) Nitrogen dioxide
(D) Sulphur dioxide

Answer: D

Given below are two statements:

Statement I : Noise Pollution has an adverse impact on hearing ability as well as psychological well-being of a person.
Statement Il : Human perception of noise levels is linear.

In the light of the above statements, choose the correct answer from the options given below:
(A) Both Statement I and Statement II are true
(B) Both Statement I and Statement II are false
(C) Statement I is true but Statement II is false
(D) Statement I is false but Statement II is true

Answer: C

Which country is in the process of setting up the largest integrated solar-wind energy park of 30 GW capacity?

(A) China
(B) India
(C) Australia
(D) Saudi Arabia

Answer: B

Given below are two statements : One is labeled as Assertion A and the other is labeled as Reason R.

Assertion A: There has to be an integrated approach to achieve the targets of Sustainable Development Goals (SDGs).
Reason R : Many of the SDGs are interconnected.

In the light of the above statements, choose the most appropriate answer from the options given below:
(A) Both A and R are correct and R is the correct explanation of A
(B) Both A and R are correct but R is NOT the correct explanation of A
(C) A is correct but R is not correct
(D) A is not correct but R is correct

Answer: A

Given below are two statements : One is labeled as Assertion A and the other is labeled as Reason R.

Assertion A: Mankindโ€™s interaction with Earth has led to a highly complex and diverse ecosystem to an increasingly degraded environment.
Reason R: Many of the changes taking place in Earthโ€™s environment are irreversible.

In the light of the above statements, choose the most appropriate answer from the options given below:
(A) Both A and R are true and R is the correct explanation of A
(B) Both A and R are true but R is NOT the correct explanation of A
(C) A is true but R is false
(D) A is false but R is true

Answer: A

What percentage of Indiaโ€™s land is vulnerable to earthquakes?

(A) ~ 57%
(B) ~ 42%
(C) ~ 37%
(D) ~ 28%

Answer: A

Which of the following is an ozone depleting substance?

(A) Sulphur dioxide
(B) Methyl chloride
(C) Carbon dioxide
(D) Methane

Answer: B

Which of the following sources of energy is not a form of solar power?

(A) Tidal Power
(B) Wind Power
(C) Coal
(D) Hydro Power

Answer: C

Which of the following natural hazards is characterized by progressive occurrence and multicausality?

(A) Earthquake
(B) Volcanic eruption
(C) Epidemic
(D) Storm

Answer: C

The abbreviation NMSHE, in the context of Climate Action Plan of Govt. of India, stands for

(A) National Mission on Solar-Hydro Energy
(B) National Mission on Sustaining Himalayan Ecosystem
(C) National Mission on Sustainable Hydrogen Energy
(D) National Mission on Survey of Himalayan Ecosystem

Answer: B

Which of the following pollutants have adverse implications for human health as well as climate change?

A. Nitrogen dioxide
B. Particulate matter (PM2.5)
C. Ozone
D. Methane
E. Nitrous oxide

Choose the correct answer from the options given below:
(A) B, C and D only
(B) A, B and C only
(C) B and D only
(D) B and C only

Answer: D

Formation of photochemical smog involves the following

A. Carbon dioxide
B. Volatile Organic Carbon
C. Sunlight
D. Methane
E. Nitrogen dioxide

Choose the correct answer from the options given below:
1. A, B and C only
2. B, C and E only
3. A, C and D only
4. A, B and D only

Ans: B

Dioxins are produced as a byproduct of

A. Forest fires
B. Incineration of plastics
C. Chemical processing
D. Volcanic eruptions
E. Diesel fuel combustion

Choose the correct answer from the options given below:
1. A, C and D only
2. B, C and D only
3. C, D and E only
4. B, C and E only

Ans: C

The Indian Government has recently pledged to reduce the projected carbon emissions by

1. 1 billion tons by 2050
2. 1 billion tons by 2030
3. 2.5 billion tons by 2050
4. 100 million tons by 2030

Ans: C

During nuclear fission process, the speed of neutrons is reduced to a critical value by using following moderators:

A. Deuterium
B. Graphite
C. Monazite
D. Tritium
E. Heavy Water

Choose the correct answer from the options given below:
1. A, C and D only
2. B and E only
3. B, C and D only
4. D and E only

Ans: D

Which one of the following isotopes of Uranium is used as fissile material in nuclear thermal power plants?

1. U234
2. U235
3. U238
4. U239

Answer: 4

Which one among the following rotors ig mostly used in wind turbines for electricity generation?

1. Multiblade rotor
2. Propeller rotor
3. Savonious rotor
4. Darrieus rotor

Answer: 2

What is the correct chronological order of the followimeg from older to newer?

(A) Paris agreement
(B) Convention on biodiversity
(C) Kyoto Protocol
(D) Montreal Protocol
(E) Stockholm Conference

Choose the correct answer from the options siven below :
1. B, D, C, E A
2. B, C D, E, A
3. E, D, C, B, A
4. E, D, B, C, A

Answer: 4

Given below are two statements โ€“

Statement I : Total absence of fluoride in drinking water causes dental caries.
Statement II : With presence of fluoride in drinking water, all the dental problems can be avoided.

In the light of the above statements. choose the most appropriate answer from the options given below:
1. Both Statement I and Statement II are correct
2. Both Statement I and Statement I are incorrect
3. Statement l is correct but Statement II is incorrect
4. Statement I is incorrect but Statement II is correct

Answer: 3

According to Indian National Ambient Atv Quality standards an Aw Quality Index (AQT) value of 200 indicates the quality of air as

1. Very Poor
2. Moderate
3. Poor
4. Satisfactory

Answer: 3

Which of the followimg pesticides is expected to persist in the environment for longer period as compared to others?

1. Dieldrin
2. Parathion
3. Malathion
4. Carbryl

Answer: C

Avrange the following terms chronologically according to the time these were first introduced.

(A) Principle of common but differentiated responsibility.
(B) Sustainable Development
(C) Nationally Determined Contributions
(D) Clean Development Mechanism

Choose the correct answer from the options given below :
1. A B C D
2. B A C D
3. B A D C
4. A B D C

Answer: D

Identify the nuclear disasters among the following:

(A) Love cannal
(B) Fukushima
(C) Kyshtym
(D) Chernobyl

1. A B and C Only
2. B C and D Only
3. A B and D Only
4. D Only

Answer: B

Given below are two statements : One is labelled as Assertion A and the other is labelled as Reason R,

Assertion(A): Energy of most cvelones dissipates fast after landfall.
Reasons (R): Surface roughness of land areas is higher than that of oceans.

In the light of the above statements, choose the most appropriate answer from the options given below :
1. Both (A) and (R) are true and (R) is the correct explanation of (A)
2. Both (A) and (R) are true but (R) is NOT the correct explanation of (A)
3. (A) is correct but (R) is not correct
4. (A) is not correct but (R) is correct

Answer: B

Given below are two statements :

Statement I : Understanding the concept of sustaimable development is the pre-requisite for developing the values needed for it,
Statement II : Sustainable development is defined as development that meets the needs of the present generation without comproniusing the ability of future generations to meet their own need.

1. Both Statement I and Statement II are true
2. Both Statement I and Statenient II are false
3. Statement I is true but Statement II is false
4. Statement I is false but Statement II is true

Answer: C

Ambient noise levels on a site located in vicinity of a busy road are expected to be in the range:

A. 35 to 45 dB
B. 50 to 60 dB
C. 75 to 85 dB
D. 110 to 120 dB

Ans: C

Given below are two statements:

Statement I : Ozone hole over the Antarctica was discovered in 1987
Statement II : The Montreal Protocol envisaged 50% reduction in the use of CFCs, in the developed countries by 1998 with reference to 1986 consumption levels.

In the light of the above statements, choose the correct answer from the options given below :
A. Both Statement I and Statement II are true
B. Both Statement I and Statement II are false
C. Statement I is true but Statement II is false
D. Statement I is false but Statement II is true

Ans: D

Which of the following statements about Worldโ€™s forests are correct?

(A) Forests act as a huge carbon sink.
(B) Most of the Worldโ€™s forests are concentrated in the subtropical belt in the northern and southern hemisphere.
(C) Forests are home to much of worldโ€™s biodiversity.
(D) Moisture released from forests also contributes to rainfall.

Choose the correct answer from the options given below :
A. A and B only
B. B C D only
C. A C D only
D. A B C only

Ans: C

Given below are two statements:

Statement I : International Solar Alliance was formed in 2010
Statemet II: International Solar Alliance is a group of twenty leading countries of the World involved in collaborative research for developing solar technologies.

In the light of the above statements, choose the most appropriate answer from the options given below:
A. Both Statement I and Statement II are true
B. Both Statement I and Statement II arรฉ false
C. Statement I is true but Statement II is false
D. Statement I is false but Statement II is true

Ans: B

Given below are two statements: one is labelled as Assertion A and the other is labelled as Reason R

Assertion (A): Large dams on rivers tend to reduce the fertility of land areas in the river flood plain
Reasons (R) : Dams reduce the deposition of silt over land areas in the river flood plain.

In the light of the above statements, choose the most appropriate answer from the options given below:
A. Both (A) and (R) are correct and R is the correct explanation of A
B. Both (A) and (R) are correct but R is NOT the correct explanation of A
C. (A) is correct but (R) is not correct
D. (A) is not correct but (R) is correct

Ans: A

Carbon monoxide pollution may cause loss of consciousness or death in the exposed individual. For such an event to occur, the likely minimum percentage saturation level of carboxyhaemoglobin in the human blood would be:

A. 5%
B. 15%
C. 30%
D. 60%

Ans: D

At a typical urban site, the ground level ozone concentration would be maximum at what time of the day?

A. Morning
B. Afternoon
C. Evening
D. Night

Ans: B

Identify the biodiversity hot-spot regions from the following โ€“

(A) Western Ghats
(B) Central India
(C) Sundaland
(D) Indo-Burma

Choose the correct answer from the options given below :
A. (A) and (B) only
B. (B) and (C) only
C. (A), (C) and (D) only
D. (A), (B) and (D) only

Ans: C

Given below are two statements :

Statement I : National Action Plan on Climate Change was launched in 2014,
Statement II : National Mission for Smart Cities is one of the missions under the National Action Plan on Climate Change.

In the light of the above statements, choose the correct answer from the options given below :
A. Both Statement I and Statement II are true
B. Both Statement I and Statement II are false
C. Statement I is true but Statement II is false
D. Statement I is false but Statement II is true

Ans: B

A 20 dB increase in noise level represents an enhancement in intensity of sound by a factor of

A. 2
B. 5
C. 100
D. 20

Answer: C

An earthquake in magnitude on the Richter scale in the range 7ยญ7.3, is termed as

A. destructive
B. ruinous
C. disatrous
D. catastrophic

Answer: C

Given below are two statements

Statement I: Dioxins are chlorinated compounds.
Statement II: Dioxins cause deaths of birds and other stray animals due to their widespread use as herbicides and pesticides.

In light of the above statements, choose the correct answer from the options given below:
A. Both Statement I and Statement II are true
B. Both Statement I and Statement II are false
C. Statement I is true but Statement II is false
D. Statement I is false but Statement II is true

Answer: A

โ€˜Joint Implementationโ€™ was one of the mechanisms suggested in meeting the objectives of

A. Agenda 21
B. Montreal Protocol
C. Kyoto Protocol
D. Paris Agreement

Answer: C

Which of the following ststements are true about the International Solar Alliance?

A. It was founded in 2012.
B. India is one of the founding countries.
C. Its headquarters are in Geneva.
D. It was signed by over 100 countries.
E. Mostly sunshine countries are members of this alliance.

Choose the correct answer from the options given below:
A. A, C and D only
B. A, D and E only
C. B, C and D only
D. B, D and E only

Answer: D

Given below are two statements

Statement I: Concentration of Volatile Organic Chemicals is generally 100-1000 times higher in sutface water than underground water,
Statement Il: The most commen method of treatment of Volatile Organic Chemicals is to aerate them to atmosphere.

In light of the above statements, choose the correct answer from the eptions given below:
1. Both Statement I and Statement II are true.
2. Both Statement I and Statement II are false.
3. Statement I is true but Statement II is-false.
4, Statement I is false but Statement II is true,

Ans: D

Pyrolysis helps in addressing the Problem of

1. Air Pollution
2. Water Pollution
3. Municipal Solid Wastes
4. Electronic Wastes

Ans: C

The period of Sendai framework for disaster risk reduction is:

1. 2015 โ€“ 2030
2. 2020 โ€“ 2040
3. 2015 โ€“ 2025
4. 2020 โ€“ 2030

Ans: A

Given below are two statements

Statement I: To address problem of climate change it is essential for the world to have โ€˜Net Zeroโ€™ carbon emissions by the year 2050.
Statement II: There is zero carbon emission associated with-the production of gray-hydrogen.

In light of the above statements, choose the correct answer from the eptions given below:
1. Both Statement I and Statement II are true.
2. Both Statement I and Statement II are false.
3. Statement I is true but Statement II is-false.
4. Statement I is false but Statement ll is true.

Ans: C

Given below are two statements

Statement I: Acute exposure to a chemical is.a long term continuous exposure by that chemical
Statement II: Chronic exposure to a chemical is a short term exposure to that chemical

In light of the above statements, choose the most appropriate answer from the options given below
1. Both Statement I and Statement Il are correct
2, Both Statement I and Statement II are incorrect
3. Statement I is correct but Statement II is Incorrect
4. Statement I is incorrect but Statement II is correct

Ans: B

In 2018. the Original Framework Agreement on establishment of International Solar Alliance was amended to extend the scope of its membership to

(1) All member states of United Nations
(2) The SAARC countries
(3) Countries in the Middle-East
(4) Countries in the Sub-Saharan Africa

Ans: A

Given below are two statements :

Statement (I): Atmospheric stability is important because 1t determines the ability of pollutants to disperse vertically in the atmosphere.
Statement (II): In summer season in India. the atmosphere is generally stable.

In the light of the above statements, choose the correct answer from the options given below :
(1) Both Statement (I) and Statement (I) are true
(2) Both Statement (I) and Statement (II) are false
(3) Statement (I) is true but Statement (II) is false
{4) Statement (I) is false but Statement (II) is true

Ans: C

Arsenic pollution in ground water Is due to

(1) Oxides of arsenic
(2) Sulphates of arsenic
(3) Acids of arsenic
(4) Nitrates of arsenic

Ans: C

Identify the correct sequence of the followingโ€™ countries in respect of them ranking in Sustainable Development Index 2020.

(A) China
(B) UsA
(C) Brazil
(D) Russia
(E) India

Choose the correct answer from the options given below :
(1) D>B>A>E>C
(2) B>A>C>D>E
(3) B>A>D>C>E
(4) D>B>A>C>E

Ans: B

What is the correct sequence of various components of biogas in terms of their abundance?

(A) Ammonia (NH3)
(B) Hydrogen Sulfide (H2S)
(C) Methane (CH4)
(D) Carbon dioxide (COs)

Choose the correct answer from the options given below :
(1) B>D>A>C
(2) C>D>A>B
(3) D>C>B>A
(4) C>D>B>A

Ans: D

Hardness of water is caused by the presence of

1, Solid particles in the water
2, Pathogens In the water
3, Toxic metalsin the water
4, Cations in the water

Ans: B

Reducing the intensity of noise by a factor of 1O will mean areduction in noise level by

1. 10 dB
2. 20 dB
3. 3 dB
4. 6 dB

Ans: A

Given below-are two statements

Statement I: Photovoltaic cells are often made Up of erystal silicon,
Statement II; Photovoltaic cells are also made up of inexpensive amorphous silicon, which is like ordinary glass and has no crystal properties.

Inlight of the above statements, choose the most appropriate answer from the options giver below
1, Both Statement| and Statement || are correct
2, Both Statement| and Statement| are incorract
3, Staterment | is correct but Statement Il is incorrect
4, Statement | isincorract but Statement Il ls correct

Ans: C

Given below are two statemerits, one is labelled as Assertion A and the otheris labelled as Reason R

Assertion A: Biomass is corisidered as carbon meutral,
Reason R: The arriount of carbon they emitis equal to the amount of carbor they have consumed during their life-tinne,

In light of the above statements; choose the mest appropriate arisvrer from the options given below
1, Both Aand R are correct and Ris the correct explanation of A
2, Both A-arid R are correct but Ris NOT the correct explanation of A
3, Ais correct but Ris not correct
4, Ais notcorrect but Ris correct

Ans: B

Which of the following pollutant causes aggravation of respiratory disease, arid atmospheric discolouration?

1, Carbon monoxide
2, Lead
3, Nitrogen dioxide
4, Ozone

Ans: A

Identify the correct sequence of average noise levels in increasing order of their magnitude from different sources.

(A) City street corner
(B) Conversational speech
(C) Highway
(D) Aircraft noise during take off
(E) Typical office

Choose the correct answer from the options given below :
(1) (E), (A). (B). (C). (D)
(2) (E), (B), (A), (C), (D)
(3) (B), (E). (A), (C). (D)
(4) (B), (E), (A), (D), (C)

Ans: B

Given below are two statements : One is labelled as Assertion (A) and the other is labeled as Reason (R).

Assertion (A): Energy produced from biomass is considered โ€˜carbon neutralโ€™.
Reason (R): Biomass does not add CO> to the atmosphere.

In the light of the above statements. choose the most appropriate answer from the options given below :
(1) Both (A) and (R) are correct and (R) is the correct explanation of (A)
(2) Both (A) and (R) are correct but (R) is NOT the correct explanation of (A)
(3) (A) 1s correct but (R) is not correct
(4) (A) is not correct but (R) is correct

Ans: C

What is the correct order of Glohal Warming Potential (GWP) of following atmospheric chemical species?

(A) Chlorofiure Carbon (CFC)
(B) Methane (CH)
(C) Carbon dioxide (COs)
(D) Nitrous oxide (NoO)

Choose the correct answer from the options given below :
(1) D>B>C>A
(2) G<B<D<A
(3) A>B>C>D
(4) A<C<D<B

Ans: B

Given below are two statements :

Statement I : Classical smog is formed when oxides of nitrogen combine with particulate matter especially in summer season.
Statement II: Classical smog reduces atmospheric visibility to a great extent.

In the light of the above statements. choose the correct answer from the options given below :
(1) Both Statement I and Statement II are true
(2) Both Statement I and Statement II are false
(3) Statement I is true but Statement II is false
(4) Statement I is false but Statement II is true

Ans: D

Source of arsenic pollution in the ground water is

(1) Industrial ettluents
(2) Deep underground formations
(3) Dyeing industries
(4) Agricultural wastes

Ans: A and B

Which of the following are true about Joint Forest Management (JFM)?

(A) Its concept was introduced through the Environment (Protection) Act. 1986
(B) Tt involves both state forest department and local community
(C) Women also participate in this
(D) It varies from state to state and is known by different names in different languages

Choose the most appropriate answer from the options given below:
(1) (A), (B) and (C) only
(2) (B). (C) and (D) only
(3) (A). (C) and (D) only
(4) (A), (B), (C) and (D)

Ans: B

Disaster management plan was launched by ministry of Home affair in

(1) 2005
(2) 2011
(3) 2016
(4) 2020

Ans: C

Environmental problems associated with soils are

A. Nutrient leaching
B. Metal contamination
C. Salinity and Alkalinity
D. Ozone depletion
E. Fine aerosols in ambient air

Choose the correct answer from the options given below :
1. (A). (B), (C). (E) only
2. (B). (C), (BE) only
3. (A). (B). (C). (D) only
4. (A), (B), (C) only

Ans: D

Given below are two statements : One is labelled as Assertion (A) and the other is labelled as Reason (R).

Assertion (A) : Geothermal energy is not a renewable source of energy.
Reason (R): The processes for extracting and using geothermal power are more akin to those for centralized fossil-fuels than the dispersed renewable supplies of the natural environment.

In the light of the above statements. choose the correct answer from the options given below :
1. Both (A) and (R) are true and (R) is the correct explanation of (A)
2. Both (A) and (R) are true but (R) is NOT the correct explanation of (A)
3. (A) is true but (R) is false
4. (A) is false but (R) is true

Ans: D

What is the correct sequence of concentration of various components of producer gas?

(A) CH4
(B) CO2
(C) CO
(D) N2

Choose the correct answer from the options given below :
(1) (D) > (C) > (A) > (B)
(2) (D) > (C) > (B) > (A)
(3) (C) > (D) > (A) > (B)
(4) (C) > (D) > (B) > (A)

Ans: B

1 mm layer of pure ozone gas around earth at normal temperature and pressure is equivalent to:

(1) 1 Dobson Unit (DU)
(2) 10 DU
(3) 100 DU
{4) 0.1 DU

Ans: C

Match List I with List II:

List I
(A) Mist
(B) Smoke
(C) Fume
(D) Smog
List II
I. Aerosols consisting of solid particles
II. Aerosols produced by condensation of hot vapours of metals
III. Photochemical products of VOCs and NOx
IV. Aerosols consisting of liquid draplets

Choose the correct answer from the options given below:
1. A-IV B-I C-II D-III
2. A-III B-II C-IV D-I
3. A-II B-IV C-III D-I
4. A-IV B-II C-I D-III

Ans: A

Given below are two statements: One 1s labeled as Assertion A and the other is labeled as Reason R.

Assertion A: Bottom ash generated into the boilers of the thermal power planta are major cause of nearby river pollution.
Reason R : The hotwater generated from thermal power plants are discharged into the nearby rivers.

In the hght of the above statements. choose the correct answer from the options given below:
(1) Both A and R are true and R is the correct explanation of A
(2) Both A and R are true but R is NOT the correct explanation of A
(3) A is true but R is false
(4) A is false but R is true

Ans: B

Given below are two statements:

Statement I: Chronic toxicity caused due to hazardous wastes is not difficult to determine.
Statement II: Chronic effects may be seen immediately after the exposure.

In the light of the above statements, choose the most appropriate answer from the options given below:
(1) Both Statement I and Statement II are correct
(2) Both Statement I and Statement II are incorrect
(3) Statement I is correct but Statement II is incorrect
(4) Statement I is incorrect but Statement II is correct

Ans: B

What is the correct sequence of preparing vermi-compost?

A. Introduction of culture of worms in to the pit.
B. Lining up with straw or dried grass and leaves
C. Water the pit once or twice a week
D. Construction of pit of half square meter area and one meter deep
E. Putting into pit the organic wastes as and when generated

Choose the correct answer from the options given below:
1. (D). (E). (B). (A). (C)
2. (A), (C), (D), (E), (B)
3. (E), (A), (B), (C). (D)
4. (D), (B). (E), (A). (C)

Ans: D

Over the past 100 years; the estimated rise of global sea level is by 10 โ€“ 25 cm or more. Which one of the following has maximum contribution in it?

1. Thermal expansion of ocรฉarn waters
2, Glacier and ice- cap melting
3. Greenland ice sheet thinning
4, Terrestrial water storage changes due to human activities

Ans: A

Aeration process is efficiently used to: remove

1, Persistant Organic Pollutants
2. Volatile Organic Compounds
3. Metals
4. |narganic ions

Ans: B

What is the correct order of Risk assessment due to exposure of some chemicals to the humans

A, Dose Response Assessment
B. Exposure Assessment
C. Risk Characterization
D, Hazard Identification

Choose the correct answer from the options given below
1. A, B, C, D
2. D, B, C, A
3. C, D, A, B
4. D, A, B, C

Ans: D

National Disaster Management Authority (NDMA) works under the aegis of

1. Ministry of Earth Science (MoES)
2. Ministry of Home Affairs (MHA)
3. Ministry of Environmental Forest and Climatic Change (MoEFCC)
4. Department of Space- Indian Space Research Organisation (ISRO)

Ans: B

Which of the following statements are correct in this context of energy crops ?

A, They are fast growing plants
B, They require specific sรฉasarito grow
C, They necessarily require fertile land to grow
D. They can be converted into gaseous and liquid fuel
E, They generally have high calorific value

Choose the correct answer from the options given below,
1. A, D.and E only
2, A, B,C and D only
3. B, C, D and E only
4. C, D and E only

Ans: A

Given below are two statements : One is labelled as Assertion (A) and the other is labelled as Reason (R) :

Assertion (A): Geothermal fluids are rich in salts and other contaminants such as hydrogen sulphide (H:8) and oxides of nitrogen (NOx)
Reasons (R) : They are re-injected into ground rather than being dumped to river or ponds.

In the light of the above statements. choose the most appropriate answer from the options given below:
(1) Both (A) and (R) are correct and (R) is the correct explanation of (A)
(2) Both (A) and (R) are correct but (R) is NOT the correct explanation of (A)
(3) (A) is correct but (R) is not correct
(4) (A) is not correct but (R) 1s correct

Ans: B

Which one among the following is not a unit of measurement of surface ozone?

(1) Microgram per meter cube
(2) Nanogram per meter cube
(3) Dobson unit
(4) Parts per million

Ans: C

Match List I with List II

List I Hazardous Waste
(A) F-list
(B) E-list
(C) P-list
{D) U-Hst
List II Explanation
(I) Hazardous wastes from specific industrial processes
(II) Discarded chemicals exceeding 25 kilograms per month
(IIl) Hazardous wastes from non-specific sources
(IV) Discarded chemicals exceeding 1 kilogram per month

Choose the correct answer from the options given below :
1. A-III B-I C-IV D-II
2. A-II B-III C-I D-IV
3. A-IV B-II C-III D-I
4. A-I B-IV C-II D-III

Ans: A

The substance that slows down the neutrons to have controlled chain reaction during nuclear energy production Is*called as

1. Controller
2. Inhibiter
3. Moderator
4. Reducer

Ans: D

Which of the fallawing pollutants is cansidered as surrogate for eye irritation?

1. Nitrogen dioxide
2, Sulphur dioxide
3, Carbon monoxide
4 Ozone

Ans: C

Given below are two statements

Statement I: Pesticides such as DDT are particularly disruptive to food chains.
Statement II: Pesticides are very persistent and soluble in water.

In light of the above statements, choose the most appropriate answer from the options given beiow
1. Both Statement I and Statement II are correct
2. Both Statement I and Statement II are incorrect
3. Statement I is correct but Statement Il is Incorrect
4. Statement I is incorrect but Statement II is correct

Ans: A

Given below are two statements

Statement I: Global warming could lead to increased release of the greenhouse gas, Methane.
Statement Il: There is a large amount of Methane currently frozen in the permafrost in the far northern regions of the world.

In light of the above statements, choose the most appropriate answer from the options given beiow
1, Both Statement | and Statement II are correct
2, Both Statement | and Statement II are incorrect
3, Statement | is correct but Statement II is incorrect
4, Statement | is incorrect but Statement II is correct

Ans: C

Given below are two statements

Statement I: Biodiesel can reduce Sulphur emissions significantly in comparison to gasoline,
Statement II: Energy density of biodiesel is much less than that of ethanol.

In light of the above statements, choose the correct answer from the options given belaw
1, Both Statement | and Statement II are true
2, Both Statement | and Staternent II are false
3, Statement | is true but Statement II is-false
4, Statement | is false but Statement || is true

Ans: C

Which of the following are examples of secondary air pollutants?

(A) Sulphate
(B) Nitric Oxide (NO)
(C) Photochemical smog
(D) Peroxy Acetyl Nitrate (PAN)
(E) Polycyclic Aromatic Hydrocarbons (PAH)

Choose the most appropriate answer from the options given below :
1. (A), (B), (C) and (D) only
2. (C). (D). and (E) only
3. (B), (C) and (D) only
4. (A), (C) and (D) only

Ans: D

Which of the following are common in both Millinium Development Goals (MDGs) and Sustainable Development Goals (SDGs)?

(A) Zero hunger
(B) Life below water
(C) No poverty
(D) Life on earth
(E) Gender equality

Choose the correct answer from the options given below :
1. (A), (B), and (C) only
2. (B). (C), and (D) only
3. (A). (D) and (E) only
4. (A), (C) and (E) only

Ans: D

Given below arรฉ two statements :

Statement I : An increase in CQยป is expected to enhance global plant growth.
Statement II: Deforestation can increase the biosphereโ€™s capacity.

In the light of the above statements. choose the correct answer from the options given below :
1. Both Statement I and Statement II are true
2. Both Statement I and Statement II are false
3. Statement I is true but Statement II is false
4. Statement I is false but Statement II is true

Ans: C

Green hydrogen is produced by electrolysis powered by

1. Energy from thermal power plant
2. Renewable sources of energy
3. Nuclear energy
4. Lead battery

Ans: B

Given below are two statements :

Statement I : Bioconcentration in humans can occur due to consumption of fish.
Statement II : Bioconcentration is the tendency of hazardous substances to accumulate in human tissues

In the Light of the above statements, choose the correct answer from the options given below :
1. Both Statement I and Statement II are true
2. Both Statement Iand Statement II are false
3. Statement Tis true but Statement [I is false
4. Statenient Tis false but Statement I] is true

Ans: C

Given below are two statements ; One is labelled as Assertion A and the other is labelled as Reason R,

Assertion(A): Large seale dams also cause the Green House Gas emissions.
Reasons (R): When submerged vegetation decays. It produces carbon dioxide which is a sreen house gas.

In the Light of the above statements. choose the most appropriate answer from the options given below :
1. Both (A) and (R) are true and (R) is the correct explanation of (A)
2. Both (A) and (R) are true but (R) is NOT the correct explanation of (A)
3. (A) is true but (R) is not false
4. (A) is not false but (R) is true

Ans: D

Wind enerey

{A) an Intermittent energy
(B) is extremely useful at remote and isolated places
(C) does not depend upon weather
(D) is a clean enerey
(E) does not have any enviornmental impact at all

Choose the most appropriate answer from the options given below :
1. A B C only
2. A B D only
3. B C D only
4. C D E only

Ans: C

Indian National Ambient Ar Quality standards stipulate the maximum permissible limit of annual mean concentration of PM2.5 to be

1. 60?g/m3
2. 45?g/m3
3. 40?g/m3
4. 25?g/m3

Ans: A

Which of the following are sustainable Development Goals (SDGs)?

(A) No poverty
(B) Zero hunger
(C) Employment to everyone
(D) Quality education
(E) Gender equality

Choose the correct answer from the options given below :
1. A B C only
2. A B C D only
3. A B D E only
4. C D E only

Ans: D

Geothermal field requires combination of following three conditions

(A) A natural underground source of water
(B) A mountain in the vicinity
(C) An impermeable laver
(D) A coal mine in the vicinity
(E) Alarge mass of hot rock in the vicinity

Choose the most appropriate answer from the options given below :
1. A C E only
2. A C D only
3. B C D only
4. B D E only

Ans: C

Given below are two statements :

Statement I : Natural erosion of the soil is gradual removal of the top soil by natural processes.
Statement II : Accelerated erosion of the soil is caused due to man made activities. The rate of accelerated erosion is equal to the rate of soil formation.

In the light of the above statements, choose the correct answer from the options given below :
1. Both Statement I and Statement II are true
2. Both Statement I and Statement II are false
3. Statement I is true but Statement II is false
4. Statement I is false but Statement II is true

Ans: D

Giilorinein drinking water is used for

(A) Disinfection
(B) Removal of hardness
(C) Odor treatment
(D) turbidity control
(E) Removal of iron and manganese

Choose the correct answer from the options given below :
1. A C E only
2. A B C only
3. A D E only
4. C D E only

Ans: B

Given below are two statements :

Statement I : Volatile Organic Chemicals (VOCs) are among the most commonly found contaminants in ground water.
Statement II : The concentration of VOCs in ground water is much less compared to that in surface waters.

In the light of the above statements. choose the correct answer from the options given below :
1. Both Statement I and Statement II are true
2. Both Statement I and Statement II are false
3. Statement I is true but Statement II is false
4. Statement I is false but Statement II is true

Ans: A

Dissolved Oxygen (DO) is important for

1. Drinking water
2, Rain water
3, Underground water
4, River water

Ans: D

The only energy source which ts not in anyway related to Sur is

1, Hydro
2, Tidal
3, Wind
4, Geothermal

Ans: D

Solar ponds

A. produce potable water
B, are thermal energy collectors
C, use salt gradient-to inhibit heat loss
D. are used for swimming In cold regions
E, produce heated water

Choose the correct answer from the options given below;
1. A, B and C only
2, B, C and E only
3. A, B, C and D only
4, B, C, D and E only

Ans: B

Which of the following are Green House Gases?

A. Carbon dioxide
B, Nitrogen dioxide
C, Methane
D. Chicrofluorocarbons
E, Benzene

Choose the correct answer from the options given below;
1. A, B and C only
2, A, B, C and D only
3. A, C and D anly
4,.C, D and E only

Ans: C

Given below are two statements:

Statement I: Millennium Development Goals (MDGs) had a total of 31 targets-te be achieved by 2015. Statement II: There are 8 goals in MDGs and 17 in SDGs,

In light of the above statements, choose the correct answer from the options given below:
1, Both Statement | and Statement II are true
2, Both Statement | and Staternent II are false
3, Statement | is true but Statement II is-false
4, Statement | is false but Statement Il is true

Ans: D

Qrganochlorine pesticides such as DDTโ€™s human toxicity is comparatively lower. It is their impact on food chain that leads to acute human toxicity. Their properties that cause them to be impactful on food chain are

(A) Persistence
(B) Insolubihity
(C) Solubility
(D) Polarity

Choose the most appropriate answer from the options given below:
1. A B only
2. A C only
3. A B D only
4. A C D only

Ans: B

Which of the following are responsible for global cooling?

(A) Halocarbons
(B) Stratospheric ozone
(C) Tropospheric ozone
(D) Sulphate aerosols
(E) Black carbon

Choose the most appropriate answer from the options given below :
1. A B D only
2. B D only
3. B D E only
4. C D only

Ans: B

Which of the following are Sustainable Development Goals (SDGs)?

(A) Life below water
(B) Life on land
(C) Zero deforestation
(D) Clean water
(E) Clean Air

Choose the correct answer from the options given below :
1. A C D only
2. B C E only
3. A B D only
4. A B D E only

Ans: B

Indiaโ€™s disaster management plan is based on

1. Sendai Framework
2. Kyoto protocol
3. Rio summit
4. Montreal protocol

Ans: A

Blue hydrogen is produced by process of

1. Electrolysis
2. Steam reforming of natural gas or methane
3. Steam reforming of Bituminous coal
4. Steam reforming of henite

Ans: B

Given below are two statements

Statement I: Prolonged (chronic) exposure to high noise levels (90-110 dB) causes only a temporary shift in the hearing threshold of an exposed person.
Statement II: Noise may adversely impact the psychological as well as physiological well being of the exposed person.

In light of the above statements, choose the correct answer from the options given below
(1) Both Statement | and Statement II are true
(2) Both Statement | and Statement II are false
(3) Statement | is true but Statement II is false
(4) Statement | is false but Statement II is true

Ans: D

Given below are two statements

Statement I: Millennium Development Goals were envisaged to reduce extreme poverty by 2020.
Statement II: There were eight Millennium Development Goals.

In light of the above statements, choose the correct answer from the options given below
(1) Both Statement | and Statement II are true
(2) Both Statement | and Statement II are false
(3) Statement | is true but Statement Il is false
(4) Statement | is false but Statement Il is true

Ans: D

Identify the hazards that occur only due to natural reasons.

A. Forest fires
B. Volcanoes
C. Cyclones
D. Floods

Choose the correct answer from the options given below:
(1) A and B only
(2) B and C only
(3) C and D only
(4) A and D only

Ans: B

Arrange the following gases in an increasing order of their contribution towards global warming.

A. Methane
B. Chlorofluorocarbons
C. Nitrous oxide
D. Carbondioxide

Choose the correct answer from the options given below
(1) A,B,C D
(2) C B, A D
(3) C A B,D
(4) A C B, D

Ans: B

Given below are two statements, one is labelled as Assertion A and the other is labelled as Reason R

Assertion A: All the member countries of the International Solar Alliance are located between the Tropics of Cancer and Capricorn.
Reason R: Intensity of solar radiation in the tropical region is high as compared to temperate and polar regions.

In light of the above statements, choose the most appropriate answer from the options given below
(1) Both A and R are correct and R is the correct explanation of A
(2) Both A and R are correct but R is NOT the correct explanation of A
(3) A is correct but R is not correct
(4) A is not correct but R is correct

Ans: D

Which of the following air pollutants is known to be a human carcinogen?

(1) Carbon monoxide
(2) Asbestos fiber
(3) Nitrogen dioxide
(4) Sulphur dioxide

Ans: B

Which region of India experiences the maximum number of cyclones?

(1) East Coast
(2) West Coast
(3) Central India
(4) Northern India

Ans: A

Which of the following statements about biodiversity are correct?

A. Biodiversity can support ecosystem stability.
B. Human activities have sharply increased species extinctions.
C. Habitat fragmentation has no adverse impact on biodiversity.
D. About 70% of all known species are invertebrates.

Choose the correct answer from the options given below:
(1) A, B and C only
(2) A, B and D only
(3) B, C and D only
(4) A, C and D only

Ans: B

Identify the major sources of methane emissions:

A. Landfills
B. Mountain rivers
C. Paddy fields
D. Wetlands

Choose the correct answer from the options given below:
(1) A, B and C only
(2) B, C and D only
(3) A, B and D only
(4) A, Cand D only

Ans: D

Which one of the following is a chemical treatment process of waste water treatment?

(1) Sedimentation
(2) Neutralization
(3) Aeration
(4) Adsorption

Ans: B

Virus responsible for the spread of COVID 19 is

(1) SARS COV 19
(2) SARS COV II
(3) CORONA 19
(4) SARS COV1

Ans: B

โ€˜Gobardhanโ€™ scheme aimed at Cattle and other biodegradable waste in rural India is being implemented by Department of Drinking Water and Sanitation in collaboration with the following organisations:

(A) Ministry of New and Renewable Energy
(B) Ministry of Petroleum and Natural Gas
(C) Department of Rural Development
(D) Ministry of Enviroment, Forest and Climate change
(E) Department of Animal Husbandry and Dairying

Choose the correct answer from the options given below :
(1) (A), (B), (C). (D) only
(2) (A), (C), (D), (E) only
(3) (A), (B), (C), (E) only
(4) (B), (C), (D), (E) only

Ans: C

Given below are two statements : One is labelled as Assertion (A) and the other is labelled as Reason (R) :

Assertion (A) : Photochemical smog causes eye irritation.
Reason (R) : Photochemical smog has ozone as one of its constituents.

In the light of the above statements, choose the most appropriate answer from the options given helow :
(1) Both (A) and (R) are correct and (R) is the correct explanation of (A)
(2) Both (A) and (R) are correct but (R) is NOT the correct explanation of (A)
(3) (A) is correct but (R) is not correct
(4) (A) is not correct but (R) is correct

Ans: B

Hurricanes are tropical cyclone with surface wind speeds in excess of about

(1) 119 km/hour
(2) 71 km/hour
(3) 51 km/hour
(4) 47 km/hour

Ans: A

Global warming is attributed to the presence of the following gases in the atmosphere:

(A) Methane
(B) Sulphur dioxide
(C) Surface Ozone
(D) Nitrogen dioxide
(E) Carbon dioxide

Choose the correct answer from the options given below :
(1) (A). (B), (C). (E) only
(2) (A), (C), (E) only
(3) (A), (C). (D), (E) only
(4) (A), (B), (D), (E) only

Ans: B

Given below are two statements : One is labelled as Assertion (A) and the other is labelled as Reason (R) :

Assertion (A) : In vitro testing forms the basis for hazard identification.
Reason (R) : In vitro studies are performed within the same biological entities.

In the light of the above statements, choose the correct answer from the options given below :
(1) Both (A) and (R) are true and (R) is the correct explanation of (A)
(2) Both (A) and (R) are true but (R) is NOT the correct explanation of (A)
(3) (A)is true but (R) is false
(4) (A)is false but (R) is true

Ans: C

Gray Water consists of

(A) Kitchen waste water
(B) Waste water containing fecal matter
(C) Waste water from industrial units
(D) Bathroom waste water
(E) Water from agricultural run-off

Choose the correct answer from the options given below :
(1) (A), (B),(C), (E) only
(2) (A), (C), D), (E) only
(3) (C) and (E) only
(4) (A) and (D) only

Ans: D

Concept behind fuel cell vehicles (FCVs) is

(1) Using solar energy to run cars
(2) Using rechargeable batteries to run cars
(3) Using hydrogen and oxygen to run cars
(4) Using improvised CNG to run cars

Ans: C

In helping to protect prople from exposure to UV radiatiors. Montreal Protocol contributes also in attainment of the following Sustainable Development Goals (SDGs):

(1) SDGs 3,11,14 and 15
(2) SDGs 3,7,11 and 12
(3) SDGs 2, 3, 5 and 11
(4) SDGs 3, 5, 7 and 12

Ans: A

Identify the non-recyclable wastes among the following:

(A) Paper
(B) Plastic bottles
(C) Thermocol
(D) Tetra packs
(E) Metal

Choose the correct answer from the options given below :
(1) (B). (C). (D). (E) only
(2) (B). (D). (E) only
(3) (C). (D) only
(4) (A). (C). (E) only

Ans: Drop

A key component in Swatchh Bharat Mission (Grameen). โ€˜SLWNM stands for

(1) Solid and Liquid Waste Management
(2) Sanitation and Liquid Waste Management
(3) State Level Waste Management
(4) State Level Water Management

Ans: A

Arrange the following fossil fuels in increasing order of their carbon intensities (gC/MJ):

(A) Natural gas
(B) Coal
(C) Petroleum

Choose the correct answer from the options given below :
1. A<C<B
2. C<A<B
3. B<A<C
4. A<B<C

Ans: A

Given below are two statements : One is labelled as Assertion (A) and the other is labelled as Reason (R) :

Assertion (A): Soil Pollution due to detergents affects the root growth of the plants and depresses the growth of soil micro-organisms.
Reason (R) : Presence of detergents in soil makes the soil more acidic.

In the hght of the above statements. choose the most appropriate answer from the options given below :
(1) Both (A) and (R) are correct and (R) is the correct explanation of (A)
(2) Both (A) and (R) are correct but (R) is NOT the correct explanation of (A)
(3) (A) is correct but (R) is not correct
(4) (A) is not correct but (R) is correct

Ans: C

Examples of Persistant Organic Pollutants (POPs) are:

(A) Dioxins
(B) PAN
(C) VOC
(D) FURANS
(E) PCB

Choose the correct answer from the options given below :
(1) (A), (D). (E) only
(2) (A). (B), (D) only
(3) (B). (C). (E) only
(4) (C). (D). (E) only

Ans: A

In the first commitment period of Kyoto Protocol, how many green house gases were covered for reducing their emissions?

1. 4
2. 5
3. 6
4. 7

Ans: C

Given below are two statements : One is labelled as Assertion (A) and the other is labelled as Reason (R):

Assertion (A): Green hydrogen production will be critical for world community to achieve carbon neutrality by the year 2050.
Reason (R): The energy content of hydrogen gas is much higher than that of coal.

In the light of the above statements. choose the correct answer from the options given below :
(1) Both (A) and (R) are true and (R) is the correct explanation of (A)
(2) Both (A) and (R) are true but (R) is NOT the correct explanation of (A)
(3) (A) is true but (R) is false
(4) (A) is false but (R) is true

Ans: B

Since the start of industrial revolution, the acidity of oceanโ€™s surface water has

(1) increased by ~10%
(2) remained constant
(3) decreased by ~25%
(4) increased by ~30%

Ans: D

Sustainable Development Goals (SDGs) are also known as

1. Agenda 30
2. Agenda 35
3. Agenda 17
4. Agenda 10

Ans: A

Given below are two statements:

Statement |: Most soils are stratified into horizontal layers called as soil profile.
Statement Il: A cross sectional view of the horizons in a soil is called as soil horizon.

In light of the above statements, choose the most appropriate answer from the options given below:
1. Both Statement | and Statement II are true.
2. Both Statement | and Statement II are false.
3. Statement | is true but Statement II is false.
4, Statement | is false but Statement II is true.

Ans: A

A general term for downhill slides of earth is known as?

1. Mass wasting
2. Earth wasting
3. Mud wasting
4. Hill wasting

Ans: A

Given below are two statements:

Statement |: Allergens are substances that activate the immune system.
Statement Il: Concentration of allergens in indoor environment can be thousand times higher than in the air outside.

In light of the above statements, choose the most appropriate answer from the options given below:
1. Both Statement | and Statement II are true.
2. Both Statement | and Statement II are false.
3. Statement | is true but Statement II is false.
4. Statement | is false but Statement II is true.

Ans: A

Which of the following Volatile Organic Compound (VOC), a kind of air pollutant, is emitted from natural sources such as plants, bogs etc.?

1. Formaldehyde
2. Benzene
3. Terpene
4. Toluene

Ans: C

Chlorination of water is done to

1. Remove microbes
2. Remove hardness
3. Increase salt contents
4, Decrease turbidity

Ans: A

What is the average thickness of Ozone layer ?

1. > 1 cm
2. < 1 cm
3. 1 cm to 1 meter
4.> 1 meter

Ans: B

Given below are two statements :

Statement-I: Ion exchange is an example of tertiary waste water treatment process.
Statement-II: Ion exchange is used for the removal of residual suspended solids.

In the light of the above statements, choose the correct answer from the options given below:
1. Both Statement I and Statement II are true.
2. Both Statement I and Statement II are false.
3. Statement I is true but Statement II is false.
4, Statement I is false but Statement II is true.

Ans: C

Given below are two statements :

Statement-I : Montreal Protocol was agreed in 1987 and came into force in 1990.
Statement-II : Kyoto protocol was agreed in 1997 and came into force in 2005,

In the light of the above statements, choose the correct answer from the options given below:
1. Both Statement I and Statement II are true.
2. Both Statement I and Statement II are false.
3. Statement I is true but Statement II is false.
4, Statement I is false but Statement II is true.

Ans: D

Given below are two statements :

Statement-I: Some pollutants such as carbon tetrachloride (CC1,). pesticides and heavy metals cause liver damage.
Statement-II: Toxic chemicals that cause liver damage are called Hematotoxins.

In the light of the above statements, choose the correct answer from the options given below:
1. Both Statement I and Statement II are true.
2. Both Statement I and Statement II are false.
3. Statement I is true but Statement II is false.
4. Statement I is false but Statement II is true.

Ans: C

Hydrogen produced using solar energy is called as

1. Grey hydrogen
2. Green hydrogen
3. White hydrogen
4. Torquoise hydrogen

Ans: B

Which of the following are removed through secondary waste water treatment process?

1. Polythene, tetra pack, etc
2. Large suspended solids
3. Colloidal and dissolved organic matter
4. Settleable organic matters

Ans: C

Given below are two statements:

Statement I: Article 48A: It is the duty of citizens of India to protect and improve the natural environment. Statement II: Article 51: Directive Principles of state policy are to ensure protection and improvement of the natural Environment.

In light of the above statements, choose the correct answer from the options given below:
1. Both Statement | and Statement Il are true.
2. Both Statement | and Statement |l are false.
3. Statement | is true but Statement Il is false.
4. Statement | is false but Statement Il is true.

Ans: B

How many national missions are there in the National Action Plan on climate change?

1. 6
2. 7
3. 8
4. 9

Ans: C

Given below are two statements:

Statement-I: Hydro chlorofluorocarbons (HCFCs) were introduced as ozone-safe replacement for chlorofluorocarbons (CFCs)
Statement-II: Hydro chlorofluorocarbons (HCFCs) do not contribute to green house warming.

In the light of the above statements, choose the most appropriate answer from the options given below:
1. Both Statement I and Statement II are correct.
2. Both Statement I and Statement II are incorrect.
3. Statement I is correct but Statement II is incorrect.
4. Statement I is incorrect but Statement II is correct.

Ans: C

Which of the following play a role during formation of photochemical smog?

A.NO, (Nitrogen dioxide)
B. Black Carbon
C. Sun light
D. VOCs (Volatile Organic Compounds)
E. CO (Carbon monoxide)

Choose the correct answer from the options given below:
1.A, B and C only
2.A, CandD only
3.A.Dand E only
4.B,C.Dand E only

Ans: B

In which of the following types of electricity power plants, turbines are not required?

1. Geothermal
2. Nuclear
3. Hydro
4. Solar

Ans: D

Given below are two statements: one is labelled Assertion A and other is labelled as Reason R.

Assertion-A : Rise in water temperature from normal levels, due to thermal pollution, can adversely affect โ€˜water quality and aquatic life.
Reason-R : Oxygen solubility in water increases as temperature increases.

In the light of the above statements, choose the most appropriate answer from the options given below:
1. Both A and R are correct and R is the correct explanation of A.
2.Both A and R are correct but R is NOT the correct explanation of A.
3. Ais correct but R is not correct.
4. Ais not correct but R is correct.

Ans: C

Which of the following Millenium Deveopment Goals (MDGs) had the highest number of targets?

1. Goal 5: Improve Maternal Health
2. Goal 6: Combat HIV / AIDS
3. Goal 7 : Ensure Environmental Sustainability
4. Goal 8: Develop a Global Partnership for Development

Ans: D

Which of the following diseases are caused by atmospheric particulate matters?

A. Astharna
B. Melanoma
C. Dry Eye Syndrome
D. Bronchitis
E. Chronic Obstructive Pulmonary Diseases (COPD)

Choose the correct answer from the options given below:
1.4, B, Cand D only
2.B, C, Dand E only
3.4, C, Dand E only
4.4, B, D and E only

Ans: C

Given below are two statements:

Statement |: Amount of oxygen dissolved in water is a good indicator of water quality and of the kinds of life it will suppport.
Statement II: Chemical Oxygen Demand (COD) is a measure of dissolved oxygen consumed by aquatic microorganisms.

In light of the above statements, choose the correct answer from the options given below:
1. Both Statement | and Statement II are true.
2. Both Statement | and Statement II are false.
3. Statement | is true but Statement II is false.
4. Statement | is false but Statement II is true.

Ans: C

Given below are two statements:

Statement |: Reactor is the heart of any nuclear power plant.
Statement II: In case of nuclear power plants, except the reactor, the technology is exactly similar to any other fossil fueled power plant.

In light of the above statements, choose the correct answer from the options given below:
1. Both Statement | and Statement II are true.
2. Both Statement | and Statement II are false.
3. Statement | is true but Statement II is false.
4. Statement | is false but Statement II is true.

Ans: A

Which one of the following pollutants is responsible for acid rain formation?

1. CH4 (Methane)
2. C6H6 (Benzene)
3. SO2 (Sulphur dioxide)
4, CO ( Carbon monoxide)

Ans: C

Given below are two statements:

Statement-I: United Nations General Assembly declared 2010 as International Year of Biodiversity.
Statement II : 2010 was also the target year to achieve the halt in the decline of biodiversity.

In the light of the above statements, choose the most appropriate answer from the options given below:
1. Both Statement I and Statement II are correct.
2. Both Statement I and Statement II are incorrect.
3. Statement I is correct but statement II is incorrect.
4, Statement I is incorrect but Statement II is correct.

Ans: A

Given below are two statements:

Statement-I Vision of International Solar Alliance; Every home no matter how far away, will have a light at home.
Statement II : Mission of International Solar Alliance: Let us together make sun brighter.

In the light of the above statements, choose the most appropriate answer from the options given below:
1. Both Statement I and Statement II are correct.
2. Both Statement I and Statement II are incorrect.
3, Statement I is correct but statement II is incorrect.
4, Statement I is incorrect but Statement II is correct.

Ans: B

Full form of IPCC, a nodal agency in the context of climate change, is:

1. Intergovernmental Policy on Climate change
2. Intergovernmental Panel on Climate Change
3. International Policy on Climate Change
4. International Panel on Climate Change

Ans: B

Which of the following different sized particulate matters are required for the determination of Air Quality Index (AQI) in India?

A. PM1 (Particulate Matter Size equal or less than 1 micron)
B. PM 2.5 (Particulate matter size equal or less than 2.5 micron)
C. PM 10 (Particulate matter size equal or less than 10 micron)
D. TSPM (Total Suspended Particulate Matters)

Choose the most appropriate answer from the options given below:
1. Aand B only
2. Band C only
3. Aand C only
4. B, C and D only

Ans: B

Given below are two statements:

Statement-1 Most of the worldโ€™s geothermal sites are located in the circum pacific belt known as โ€˜rim of fireโ€™. Statement II : Himalayan geothermal fields in India are also part of โ€˜rim of fireโ€™.

In the light of the above statements, choose the most appropriate answer from the options given below: . 1.Both Statement I and Statement II are correct.
2. Both Statement I and Statement II are incorrect.
3. Statement I is correct but statement II is incorrect.
4, Statement I is incorrect but Statement II is correct.

Ans: C

Arrange the following Earthโ€™s water compartments in increasing order of the amount of the water they hold.

A. Rivers and streams
B. Fresh lakes
C. Atmosphere
D. Ice and Snow

Choose the correct answer from the following options given below :
1. A C B D
2. C A B D
3. C B A D
4. A B D C

Ans: A

Given below are two statements :

Statement : I Municipal solid waste includes Garbage and Rubbish.
Statement II : Garbage consists of both combustible and non combustible materials.

In the light of the above statements, choose the most appropriate answer from the options given below :
1. Both Statement I and Statement II are true.
2. Both Statement I and Statement II are false.
3. Statement I is true but Statement II is false.
4. Statement I is false but Statement II is true.

Ans: B

Given below are two statements :

Statement : I In many cases, both organisms and human infrastructure will not be able to move or adapt quickly enough to changing climate.
Statement II : Infectious diseases are likely to increase as a result of global warming.

In the light of the above statements, choose the most appropriate answer from the options given below :
1. Both Statement I and Statement II are true.
2. Both Statement I and Statement II are false.
3. Statement I is true but Statement II is false.
4. Statement I is false but Statement IT is true.

Ans: C

Major contributor to carbon monoxide emissions in metropolitan areas of India is/are:

1. Industrial Sector
2. Domestic Sources
3. Transport Scctor
4. Agriculture Sector

Ans: A

Given below are two statements : One is labelled as Assertion A and the other is labelled as Reason R.

Assertion A : Although internal-combustion engines used in todayโ€™s vehicles can be adapted to bum hydrogen, hydrogen powered combustion engines probably are not the future of hydrogen fuelled vehicles. Reason R : It is more efficient to convert hydrogen energy directly to electricity using fuel cells.

In the light of the above statements, choose the correct answer from the options given below :
1. Both A and R are true and R is the correct explanation of A.
2. Both A and R are true but R is not the correct explanation of A.
3. A is true but R is false.
4. A is false but R is true.

Ans: C

What is the correct sequence of steps in Risk assessment related to environmental hazards ?

A. Risk characterization
B. Exposure assessment
C. Hazard Identification
D. Risk Management

Choose the correct answer from the options given below :
1. C,B,A,D
2. C A B D
3. A C B D
4. B C D A

Ans: A

National Solar Mission, one of the missions of National Action Plan on Climate Change (NAPCC) was launched in the year

1. 2009
2. 2010
3. 2011
4. 2012

Ans: C

Which one of the following has contributed maximum in sea level rise so far in the context of climate change ?

1. Melting of Antarctic Ice-sheets
2. Melting of Glaciers
3. Thermal expansion of oceanic waters
4. Melting of Greenland Ice sheets

Ans: A

Out of different oxides of nitrogen which of the following are considered as Air Pollutants and known as NOx

A. NO (Nitric Oxide)
B. NO, (Nitrogen dioxide)
C. NO; (Nitrogen trioxide)
D. N,O (Nitrous oxide)

Choose the correct answer from the options given below :
1. A and B Only
2. B and C Only
3. C and D Only
4. A, B and D Only

Ans: C

The advantages of biodiesel over conventional petro diesel are :

A. Biodiesel produces significantly lesser CO, and SO).
B. It is non-renewable
C. It has higher octane number
D. It can be used in any ratio with petro-diesel
E. It has lower flash point.

Choose the most appropriate answer from the options given below :
1. A, B and C Only
2. B, C and D Only
3. A, C and D Only
4. C, D and E Only

Ans: A

Give below are two statements:

Statement I : The Montreal Protocol to protect the Earthโ€™s ozone layer is to date the only United Nations environment agreement to be ratified by every country in the world.
Statement II : Paris agreement establishes voluntary commitments by all parties to prepare, communicate and maintain a Nationally Determined Contribution (NDC).

In the light of the above statements, choose the correct answer from the options given below:
1. Both Statement I and Statement II are true.
2. Both Statement I and Statement II are false.
3. Statement I is true but Statement II is false.
4. Statement I is false but Statement II is true.

Ans: C

Give below are two statements:

Statement I : Thermal pollution, usually effluent from cooling systems of power plants or other industries, alters water temperature in a lake or stream.
Statement II : Humans also cause thermal pollution by altering vegetation cover and run-off patterns.

In the light of the above statements. choose the correct answer from the options given below:
1. Both Statement I and Statement II are true.
2. Both Statement I and Statement II are false.
3. Statement I is true but Statement II is false.
4. Statement I is false but Statement II is true.

Ans: A

Which of the following substances has the least Ozone Depleting Potential?

1. Chlorofluorocarbons
2. Hydro chlorofluorocarbons
3. Carbon tetrachloride
4. Halons

Ans: B

Potential regions of geothermal sites are:

A. Circum pacific belt
B. Himalayan Region
C. Delta
D. Subduction zones
E. Rann of Kachchh

Choose the correct answer from the options given below:
1. A B C only
2. B C D only
3. A B D only
4. A D E only

Ans: C

International Solar Alliance (ISA) has its head quarter in:

1. United States
2. Switzerland
3. India
4. France

Ans: C

Given below are two statements :

Statement I : Most of the hazardous landslides are induced by earthquakes.
Statement II : All landslides involve falling of earth material under shear stress.

In the light of the above statements, choose the correct answer from the options given below :
1. Both statement I and Statement II are true
2. Both Statement I and Statement II are false
3. Statement I is true but Statement II is false
4. Statement I is false but Statement II is true

Ans: D

Given below are two statements :

Statement I : It is relatively easier to monitor and regulate water pollution caused by nonpoint sources than point sources.
Statement II : Nonpoint sources of water pollution may be fairly uniform and predictable throughout the year.

In the light of the above statements, choose the correct answer from the options given below :
1. Both statement I and Statement II are true
2. Both Statement I and Statement II are false
3. Statement I is true but Statement II is false
4. Statement I is false but Statement II is true

Ans: B

Given below are two statements :

Statement I : By 2025, two-thirds of worldโ€™s people will be living in water stressed countries.
Statement II : Growth of water withdrawal and consumption in Industrial sector is going to overtake that in Agriculture sector by 2025.

In the light of the above statements, choose the correct answer from the options given below :
1. Both statement I and Statement II are true
2. Both Statement I and Statement II are false
3. Statement I is true but Statement II is false
4. Statement I is false but Statement II is true

Ans: C

Given below are two statements : One is labelled as Assertion A and the other is labelled as Reason R:

Assertion A : Kyoto Protocol exempted countries such as India and China from rolling back their Greenhouse Gas emissions by about 5% below 1990 levels by 2012.
Reason R : Exemption was given to India and China as their contribution to total Green House Gas emissions of the world was insignificant in 1997.

In the light of the above statements, choose the correct answer from the options given low :
1. Both A and R are true and R is the correct explanation of A
2. Both A and R are true but R is not the correct explanation of A
3. Ais true but R is false
4. Ais false but R is true

Ans: C

Sources of heat of geothermal energy are

A. Radioactive decay of unstable elements
B. Nuclear fusion
C. Magma
D. Temperature gradient inside earth
E. Oceanic circulation

Choose the correct answer from the options given below :
1. A, B and C only
2. B, C and D only
3. A, C and D only
4. C, D and E only

Ans: C

Which one of the following is not a constituent of photochemical smog?

1. Ozone (Oz)
2. Peroxyacetyl Nitrate (PAN)
3. Dioxins
4. Formaldehyde (HCHO)

Ans: C

What is the correct increasing order of Global Warming Potential of following air pollutants?

A. CFC-11 (CCIE)
B. Nitrous Oxide (N20)
C. Carbon dioxide (CO3)
D. Methane (CHa)

Choose the correct answer from the options given below:
1.D,C,B,A
2.A,C, D, B
3.C,D, B, A
4.C,B,D,A

Ans: C

Lead (Pb), one of the important criteria air pollutants, was widely used in petrol due to its efficacy in?

1. Antiknocking
2. Knocking
3. Catalytic Conversion
4. Lubrication

Ans: A

Which one of the following wind energy rotors is mostly used for the electricity power generation?

1. Multiblade rotor
2. Propeller rotor
3. Savonious rotor
4, Darrieus rotor

Ans: B

The Secretariat of Convention on Biodiversity (CBD) is based in

1. Montreal, Canada
2. Rio de Janeiro, Brazil
3. Geneva, Switzerland
4, New York, United States

Ans: A

Which of the following Sustainable Development Goals (SDGs) pertains to climate action?

1.SDG 11
2.SDG 12
3.5DG 13
4.5DG 14

Ans: C

Given below are two statements:

Statement I: International Solar Alliance (ISA) is an alliance of countries most of which lie between tropic of cancer and pole.
Statement II: Presently, ISA is headed by India.

In light of the above statements, choose the most appropriate answer from the options given below:
1. Both Statement | and Statement Il are correct.
2. Both Statement | and Statement Il are incorrect.
3. Statement | is correct but Statement Il is incorrect.
4. Statement | is incorrect but Statement Il is correct.

Ans: D

Halons, kinds of Ozone Depletings Substances(ODS), were essentially and widely used in

1. Refrigerators
2. Air conditioners
3. Aerosol spray propellents
4. Fire Extinguishers

Ans: D

Kyoto Protocol(1997) was about

1. Protection of ozone layer
2. Reduction in Green House Gas emissions
3. Conservation of Biodiversity
4. Limiting the rise in global temperature to 2ยฐC above pre-industrial times.

Ans: B

Schistosomiasis, a water associated disease, is caused by

1. Ingestion of contaminated water
2. Contact with contaminated water
3. Lack of sufficient water to maintain cleanliness
4. Vectors grown on contaminated water

Ans: B

Given below are two statements:

Statement I: Wind energy has high energy density.
Statement II: Wind energy is an intermittant source of energy.

In light of the above statements, choose the most appropriate answer from the options given below:
1. Both Statement | and Statement Il are correct.
2. Both Statement | and Statement Il are incorrect.
3. Statement | is correct but Statement Il is incorrect.
4. Statement | is incorrect but Statement Il is correct.

Ans: D

Ozone layer is in

1. Thermosphere
2. Mesosphere
3. Stratosphere
4. Troposphere

Ans: C

Given below are two statements:

Statement I: Millennium Development Goals (MDGs) were set to be achieved by 2030.
Statement II: Sustainable Development Goals (SDGs) were set to be achieved by 2015.

In light of the above statements, choose the most appropriate answer from the options given below:
1. Both Statement | and Statement Il are correct.
2. Both Statement | and Statement Il are incorrect.
3. Statement | is correct but Statement Il is incorrect.
4. Statement | is incorrect but Statement Il is correct.

Ans: B

Official languages for International Solar Alliance are

A. Hindi
B. English
C. French
D. Spanish
E. Chinese

Choose the correct answer from the options given below:
1.A, B and Conly
2.8,D and E only
3.B,Cand D only
4.B,Cand E only

Ans: A

Given below are two statements:

Statement I : Some microorganisms, especially protozoan, are able to survive conventional disinfection process.
Statement II : Some carcinogens, such as chloroform, are created during chlorination process.

In the light of the above statements, choose the most appropriate answer from the options given below:
1. Both Statement I and Statement II are correct.
2. Both Statement I and Statement II are incorrect.
3. Statement I is correct but Statement II is incorrect.
4. Statement I is incorrect but Statement II is correct.

Ans: A

The framework agreement of International Solar Alliance (ISA) was opened for signature of member countries in

1. New Delhi, India
2. Paris, France
3. Geneva. Switzerland
4, Marrakesh, Morocco

Ans: D

Reducing Child Mortality was one of the goals of:

1. Sustainable Development Goals (SDGs)
2. Millennium Development Goals (MDGs)
3. Both SDGs and MDGs
4. Neither SDGs nor MDGs

Ans: B

Dioxins are emitted during combustion of:

1. Diesel
2. Coal
3. Plastic
4, Biomass

Ans: Drop

Given below are two statements:

Statement I: Flat-plate solar collector has an optical concentrator.
Statement II : In concentrating type solar collector the area receiving the solar radiation is several times greater than the absorber area.

In the light of the above statements, choose the Correct answer from the options given below:
1. Both Statement I and Statement II are true.
2. Both Statement I and Statement II are false.
3. Statement I is true but Statement II is false.
4, Statement I is false but Statement II is true.

Ans: D

Which of the following air pollutants make rain water acidic ?

A. Nitric oxide (NO)
B. Nitrogen dioxide (NO2)
C. Nitrous oxide (N20)
D. Carbon monoxide (CO)
E. Sulphur dioxide (S02)

Choose the most appropriate answer from the options given below :
1.A.B and E only
2.B.C and D only
3.A,C andE only
4.A,B and C

Ans: A

Which among the following are Green House Gases (GHG) ?

A. Surface ozone (O3)
B. Ammonia (NH3)
C. Methane (CH4)
D. Carbon monoxide (CO)
E. Nitrous oxide (N20)

Choose the most appropriate answer from the options given below :
1.A,Band C only
2. A, Cand E only
3.B,Cand D only
4.C.DandE only

Ans: B

Given below are two statements:

Statement I : Passive solar systems pump a heat absorbing medium rather than storing heat in stationary medium.
Statement II : Solar energy can produce high temperatures even in the range of 1000ยฐC or more.

In the light of the above statements, choose the most appropriate answer from the options given below :
1. Both Statement I and Statement II are correct.
2. Both Statement I and Statement II are incorrect.
3. Statement I is correct but Statement II is incorrect.
4. Statement I is incorrect but Statement II is correct.

Ans: D

Sick Building Syndrome (SBS) is associated with

1. Outdoor Air Quality
2. Indoor Air Quality
3. Water Quality
4. E-waste

Ans: B

BOD (Biochemical Oxygen Demand) is the measure of :

1. air pollution
2. water contamination
3. soil contamination
4. dissolved oxygen of river water

Ans: B

Which one of the following oxides of nitrogen is a Green House Gas (GHG)?

1. NO2 (Nitrogen dioxide)
2. N2O (Nitrous oxide)
3. NO (Nitric Oxide)
4. N2O3 (Dinitrogen trioxide)

Ans: B

Which of the following Particulate Matter (PM) type is also commonly known as Respirable Suspended Particulate Matter (RSPM)?

1. PM1
2.PM25
3.PM5
4.PM10

Ans: D

Given below are two statements:

Statement I: Formaldehyde, widely used in wood products and fabrics is a very strong allergen.
Statement II: Formaldehyde concentrations in indoor air can be thousands times higher than the outdoor air.

In light of the above statements, choose the most appropriate answer from the options given below:
1. Both Statement | and Statement Il are correct.
2. Both Statement | and Statement Il are incorrect.
3. Statement | is correct but Statement Il is incorrect.
4. Statement | is incorrect but Statement Il is correct.

Ans: A

Thermal Pollution

1. is a pollution of hot springs
2. is a pollution of water body
3. is a pollution of air.
4. is increase of temperature in urban environment in comparison to surrounding suburban environment.

Ans: B

Given below are two statements:

Statement I: Grid-connected photovoltaic system suppliments the grid power during the daytime.
Statement II: Grid connected photovoltaic system does not require additional equipment to control voltage etc.

In light of the above statements, choose the most appropriate answer from the options given below:
1. Both Statement | and Statement Il are correct.
2. Both Statement | and Statement Il are incorrect.
3. Statement | is correct but Statement Il is incorrect.
4. Statement | is incorrect but Statement Il is correct.

Ans: C

Which of the following are water borne bacterial diseases ?

A. Cholera
B. Hepatitis
C. Typhoid
D. Schistosomiasis
E. Dysentery

Choose the most appropriate answer from the options given below :
1.A,Band C Only
2.B. Cand D Only
3.A,CandE Only
4.A,Dand E Only

Ans: C

Given below are two statements : One is labelled as Assertion A and the other is labelled as Reason R.

Assertion A : Beneath the surface horizons, the subsoil, usually have a lower organic content and higher concentration of mineral particles.
Reason R : Subsoil is generally made up of weathered rock fragments

In the light of the above statements, choose the most appropriate answer from the options given below :
1. Both A and R are correct and R is the correct explanation of A.
2. Both A and R are correct but R is NOT the correct explanation of A.
3. Aiis correct but R is not correct.
4. Ais not correct but R is correct.

Ans: A

Which one of the following is a secondary acrosol ?

1. Seasalt
2. Mineral particles
3. Black carbon
4. Sulphate particles

Ans: D

Hazardous waste is liquid or solid that contains substances known to be

A. Fatal to humans or animals only at high doses
B. Toxic, carcinogenic or mutagenic to humans
C. Ignitable even at temperatures less than 60ยฐC
D. Non-reactive
E. Corrosive

Choose the most appropriate answer from the options given below :
1.A. Cand D Only
2.B.Cand E Only
3.B.Cand D Only
4.C.and D

Ans: B

An Eutrophic lake

A. is rich in nutrients
B. is poor in organic matters
C. has high algal blooms
D. s rich in dissolved oxygen
E. has no issue with survival of fish and other aqualic life

Choose the most appropriate answer from the options given below :
1.A.Band C Only
2.C.Dand E Only
3.Aand C Only
4.B and D only

Ans: C

Solar ponds store solar thermal energy by ?

1. Photovoltaic cells
2. Creating salt gradient
3. Using solar thermal collectors
4. Covering major portion of pond by plastic sheets

Ans: B

โ€˜Which of the following are responsible for formation and destruction of ozone (Os) in upper atmosphere?

A, Solar radiation
B. Oxides of nitrogen (NOx)
C. Black Carbon
D. Chlorine (Cl)
E. Ammonia (NH3)

Choose the most appropriate answer from the options given below:
1.A,Band D only
2.B,Cand D only
3.C,Dand E only
4.A.CandE only

Ans: A

Which of the following are primary acrosols in atmosphere?

A. Sea salt
B. Black carbon
C. Sulphate
D. Nitrate
E. Mineral particles

Choose the most appropriate answer from the options given below:
1.A,Band C only
2.B,Cand D only
3.A,Band E only
4.C,DandE only

Ans: C

Given below are two statements:

Statement I: Dose-response assessment is the process of determining whether or not a particular chemical is causally linked to particular health effect.
Statement II : Hazard identification is the process of characterizing the relationship between the dose of an toxic agent received and the incidence of an adverse health effect.

In the light of the above statements, choose the most appropriate answer from the options given below:
1. Both Statement I and Statement II are correct.
2. Both Statement I and Statement II are incorrect.
3. Statement I is correct but Statement II is incorrect.
4. Statement I is incorrect but Statement II is correct.

Ans: B

Lower level of Dissolved Oxygen (DO) in water

A. is bad for human consumption.
B. may have been caused due to presence of oxygen demanding wastes.
C. threatens the fish and aquatic lifc.
D. cannot be used for irrigation purposes.
E. may cause undesirable taste and odour.

Choose the most appropriate answer from the options given below:
1.A, B and C only
2.A,Band D only
3.B,Cand E only
4.C.DandE

Ans: C

Which of the following gases has maximum Global Warming Potential (GWP)?

1. Chlorofluorocarbans (CFCs)
2. Carbon dioxide(CO2)
3. Sulphur hexafluoride(SFg)
4. Methane (CHs)

Ans: C

Identify the health affects related to Noise pollution

A. Speech interference
B. Hearing loss
C. Hypertension
D. Respiratory disease
E. Insomnia

Choose the correct answer from the options given below:
1.A Band E only
2.A,B,Cand D only
3.A B C Dand E
4.A,B,Cand E only

Ans: D

Which of the following are core missions of National Action Plan on Climate Change?

A. National Solar Mission
B. National Mission on dessert
C. National Water Mission
D. National mission for sustainable agriculture
E. National Mission on Indian Ocean

Choose the correct answer from the options given below:
1.A, Band Conly
2.B,Cand D only
3.A Cand D only
4.C,Dand Eonly

Ans: C

Photo-electro-chemical decomposition of water is done to

1. To produce hydrogen
2. To remove pathogens from water
3. To remove metals from water
4. To remove oil and grease from water

Ans: A

Given below are two statements, one is labelled as Assertion (A) and the other is labelled as Reason (R).

Assertion (A) : Carbon monoxide (CO) has direct green house effect.
Reason (R) : Carbon monoxide in atmosphere can form carbon dioxide (CO2) and can also affect the concentration of methane (CH4)

In light of the above statements, choose the correct answer from the options given below:
1. Both (A) and (R) are true and (R) is the correct explanation of (A).
2. Both (A) and (R) are true but (R) is NOT the correct explanation of (A).
3. (A) is true but (R) is false.
4. (A) is false but (R) is true.

Ans: D

Which of the following chemicals replaced chlorofluorocarbons (CFCs) after their phase out due to Montreal Protocol?

1. Perfluorocarbons (PFCs)
2. Hydrochlorofluorocarbons (HCFCs)
3. Sulphur hexafluoride (SFg)
4. Halons

Ans: B

Given below are two statements:

Statement I: Millennium Development Goals (MDGs) were established during millennium summit of the United Nations in the year 2000.
Statement II: Sustainable Development Goals (SDGs) were adopted in the year 2015.

In light of the above statements, choose the most appropriate answer from the options given below:
1. Both Statement | and Statement Il are correct.
2. Both Statement | and Statement Il are incorrect.
3. Statement | is correct but Statement Il is incorrect.
4. Statement | is incorrect but Statement Il is correct.

Ans: A

Which sector contributes maximum to the Indiaโ€™s total electricity production.

1. Nuclear
2. Thermal
3. Renewable
4. Hydro

Ans: B

Given below are two statements :

Statement I : Turbulent and rapidly flowing waters are generally depleted in oxygen.
Statement II : Adding organic materials, such as sewage or paper pulp, to water stimulates activity and oxygen consumption by decomposers

In the light of the above statements, choose the correct answer from the options given below :
1. Both Statement I and Statement II are Correct
2. Both Statement I and Statement II are Incorrect
3. Statement I is Correct but Statement II is Incorrect
4. Statement I is Incorrect but Statement II is Correct

Ans: D

Meaning of Tsunami is ?

1. Oceanic wave
2. Harbour Wave
3. Coastal Wave
4. Oceanic carthquake

Ans: B

Given below are two statements :

Statement I : International Solar Alliance (ISA) was launched on the side-lines of COP24.
Statement II : ISA was launched by Prime Minister Narendra Modi and President of France, Emanuel Macron.

In the light of the above statements, choose the correct answer from the options given below :
1. Both Statement I and Statement II are True.
2. Both Statement I and Statement II are False.
3. Statement I is true but Statement II is False.
4. Statement I is false but Statement II is True.

Ans: B

Which of the following Volatile Organic Compounds (VOC) are emitted from anthropogenic sources only ?

A Tsoprene
B. Benzene
C. Terpene
D. Methane
E. Toluene

Choose the correct answer from the options given below :
1.A,Band C Only
2.Band E Only
3. Aand C Only
4.C.Dand E Only

Ans: Drop

Given below are two statements, one is labelled as Assertion (A) and the other is labelled as Reason (R).

Assertion (A): Since pre-industrial times carbon dioxide has contributed maximum to the global warming. Reason (R): Carbon dioxide has highest Global Warming Potential (GWP).

In light of the above statements, choose the most appropriate answer from the options given below:
1. Both (A) and (R) are correct and (R) is the correct explanation of (A).
2. Both (A) and (R) are correct and (R) is NOT the correct explanation of (A).
3. (A) is correct but (R) is not correct.
4. (A) is not correct but (R) is correct.

Ans: C

Given below are two statements:

Statement I: Dioxins and other chlorinated hydrocarbons have been shown to accumulate to dangerous levels in fish and fish eating birds.
Statement II: Dioxins are highly toxic; even exposure to its extremely low concentration can cause birth defects, genetic disorder and cancer.

In light of the above statements, choose the most appropriate answer from the options given below:
1. Both Statement | and Statement Il are correct.
2. Both Statement | and Statement Il are incorrect.
3. Statement | is correct but Statement Il is incorrect.
4. Statement | is incorrect but Statement Il is correct.

Ans: A

Given below are two statements:

Statement I: In a breeder reactor more fissile material is produced than it is consumed.
Statement II: In a breeder reactor, breeding works best with slow neutrons and hence moderators are required.

In light of the above statements, choose the most appropriate answer from the options given below:
1. Both Statement | and Statement Il are correct.
2. Both Statement | and Statement Il are incorrect.
3. Statement | is correct but Statement Il is incorrect.
4. Statement | is incorrect but Statement Il is correct.

Ans: C

Given below are two statements, one is labelled as Assertion (A) and the other is labelled as Reason (R).

Assertion (A) : Acidity in water with pH level above 5 has detrimental effect on aquatic life.
Reason (R) : Water with pH value between 5 to less than 7 is of acidic nature.

In light of the above statements, choose the correct answer from the options given below:
1. Both (A) and (R) are true and (R) is the correct explanation of (A).
2. Both (A) and (R) are true but (R) is NOT the correct explanation of (A).
3.(A) is true but (R) is false.
4. (A) is false but (R) is true.

Ans: D

Full form of PAN, a type of air pollutant, is โ€“

1. Peroxyacetyl nitrates
2. Peroxyacety nitrites
3. Polycyclic acyl nitrates
4. Polycyclic acyl nitrites

Ans: A

Non-point sources of water pollution are:

A. from specific location
B. diffuse
C. episodic
D. identifiable
E. difficult to monitor

Choose the correct answer from the options given below:
1.B. C and E only
2.A,Band Conly
3.B.C and D only
4.C.Dand E only

Ans: A

Which of the following are biofuels?

A. Gasoline
B. Methanol
C. Methane
D. Ethanol

Choose the correct answer from the options given below:
1. A, Band C only
2.B.Cand D only
3.Band D only
4.Aand D only

Ans: C

Given below are two statements:

Statement-I: International Solar Allianceโ€™s present post of President is held by India.
Statement-II: The United States of America is not a member of International Solar Alliance.

In the light of the above statements, choose the correct answer from the options given below:
1. Both Statement I and Statement II are true.
2. Both Statement I and Statement I are false.
3. Statement I is true but Statement I is false.
4. Statement I is false but Statement IT is true.

Ans: C

Given below are two statements:

Statement-I: United Nations Framework Convention on Climate Change (UNFCC) was framed during Kyoto Protocol, 1997.
Statement-II: Kyoto Protocol set up different limits of carbon emissions for individual nations, depending on their output before 1990.

In the light of the above statements, choose the correct answer from the options given below:
1. Both Statement I and Statement II are true
2. Both Statement I and Statement II are false
3. Statement I is true but Statement I is false
4. Statement I is false but Statement II is true

Ans: D

The total power output in a hydroelectric installation depends upon

A. the total water flow
B. the electrical conductivity of water
C. atmospheric temperature
D. aertical distance through which water falls

Choose the correct answer from the options given below:
1. A.Band Conly
2. Band C only
3. Aand D only
4. A.BandD only

Ans: Dropped

In the context of climate change which of the following observations are correct for 20th century with reference to 1750?

A. The global average surface temperature has increased by 0.6ยฐC.
B. Greater increase in daytime temperature than night time has been observed.
C. Global average sea level ose 0.1-0.2 m largely due to thermal expansion.
D. The atmosphere concentration of Carbon dioxide (CO,) has increased by 31%.
E. Precipitation in the northern hemisphere has decreased by 1% per decade.

Choose the correct answer from the options given below:
1. A. B. Dand E only
2. A. C and D only
3. B. C and D only
4. A. B. C

Ans: 2

At pH values above 7.5, the disinfectant capability of chlorine is

1. greatly reduced
2. greatly enhanced
3. slightly enhanced
4. not changed

Ans: 1

Given below are two statements:

Statements I: Fuel oxides of nitrogen (NO) is created when nitrogen and oxygen in the combustion chamber are heated to as high as 1000ยฐ C temperature.
Statement II: Thermal NO, is often the dominant source in the total NO, emissions.

In the light of the above statements. choose the most appropriate answer from the options given below:
1. Both Statement I and Statement II are true
2. Both Statement I and Statement II are false
3. Statement I is true but Statement II is false
4. Statement I is false but Statement IT is true

Ans: 2

โ€˜When was the Environmental Protection Act (1986) last amended .

1. 1990
2. 1991
3. 1992
4. 1993

Ans: 2

Given below are two statements:

Statement I: Stratospheric ozone is known as โ€˜good ozoneโ€™.
Statement II: Ozone at the surface level is a serious health hazard and known as pollutant.

In the light of the above statements. choose the most appropriate answer from the options given below.
1. Both Statement I and Statement II are correct
2. Both Statement I and Statement II are incorrect
3. Statement I is correct but Statement II is incorrect
4. Statement I is incorrect but Statement IT is correct

Ans: 1

Many nations blend petrol with ethanol to reduce the emission of CO. The mixture is known as

1. Gasol
2. Petrohol
3. Gasohol
4. Ethopetrol

Ans: 3

Given below are two statements:

Statement I:Screen chamber in wastewater treatment is used to remove small and heavy suspended solids. Statement II: Grit chamber in waste water treatment is used to remove large floating materials.

In the light of the above statements. choose the most appropriate answer from the options given below.
1. Both Statement I and Statement IT are correct
2. Both Statement I and Statement II are incorrect
3. Statement I is correct but Statement II is incorrect
4. Statement I is incorrect but Statement II is correct

Ans: 2

Which one of the following gases has the maximum Global Warming Potential(GWP)

1. Methane (CHy)
2. Nitrous Oxide (N,0)
3. Carbon dioxide (CO,)
4. Carbon Monoxide (CO)

Ans: 2